Absolute Allergy and Immunology Board Review (Oct 29, 2022)_(3031128664)_(‎Springer) 9783031128660, 9783031128677


105 35

English Pages [329] Year 2022

Report DMCA / Copyright

DOWNLOAD PDF FILE

Table of contents :
Preface
Contents
Innate and Adaptive Immunity
1 Introduction
2 Innate Immunity
3 Components of the Innate Immune System
3.1 Phagocytes
3.2 Neutrophils and Neutrophil Extracellular Traps (NETs)
3.3 Monocytes and Macrophages
3.4 Innate Lymphoid Cells
3.5 NK and NK-T Cells
3.6 Epithelial Cells
3.7 Complement
4 Adaptive Immunity
5 Humoral Immunity
5.1 How Does the Adaptive Immune System Achieve Its Goals of Strength, Specificity, and Diversification?
6 Cell-Mediated Immunity
7 Components of the Adaptive Immune System
7.1 B Lymphocytes
7.2 T Lymphocytes
7.3 Antigen-Presenting Cells
7.4 The Function of CD4+ and CD8+ T Cells
8 Summary and Conclusions
Bibliography
Immune Cells and Functions
Bibliography
HistocompatIbility Antigens (HLA) and Transplantation
1 Introduction
1.1 Indications for Hematopoietic Stem Cell Transplant in Inborn Errors of Immunity
1.1.1 Indications for Hematopoietic Stem Cell Transplant in Inborn Errors of Immunity
Severe Combined Immunodeficiency
Wiskott-Aldrich Syndrome
Chronic Granulomatous Disease
1.2 Donor Selection for Hematopoietic Stem Cell Transplant in Inborn Errors of Immunity
1.2.1 Matched Related Donors
Question 3What are the most important molecular markers used to identify an adequate donor for HSCT in IEI?
1.2.2 Haploidentical Donors
1.2.3 Unrelated Donors
1.2.4 Bone Marrow
1.2.5 Peripheral Blood Stem Cells
1.2.6 Cord Blood
1.2.7 Conditioning Intensity and Donor Chimerism
Donor Chimerism
Conditioning Intensity
Examples of Disease-Specific Considerations
Concluding Thoughts and Future Directions
Pathophysiology of Acute and Chronic GVHD
Staging and Grading of Acute and Chronic GVHD
Treatment of Acute and Chronic GVHD
Severe Combined Immunodeficiency
Other Inborn Errors of Immunity
Bacterial Infections
Fungal Infections
Viral Infections
1.3 Noninfectious Complications
1.3.1 Graft Failure
1.3.2 Pulmonary Complications
1.3.3 Autoimmunity
1.4 Late Effects and Survivorship
1.4.1 General Considerations
Table 10 Specific organ/system evaluations post-HSCT
Bibliography
Allergic Diseases of the Eye
1 Introduction
2 Allergic Conjunctivitis: Seasonal or Perennial
3 Pathogenesis
4 Clinical Features of Allergic Conjunctivitis
5 Diagnosing Allergic Conjunctivitis
6 Additional Factors Affecting Allergic Conjunctivitis
7 Vernal Keratoconjunctivitis
8 Pathogenesis
9 Clinical Features and Diagnosis
10 Atopic Keratoconjunctivitis (AKC)
11 Giant Papillary Conjunctivitis
12 Differential Diagnosis of Ocular Allergy
13 Management of Ocular Allergy
14 Emerging Therapies
Bibliography
Allergic Rhinitis
1 Allergic Rhinitis
1.1 Classification
1.2 Testing
1.3 Pathophysiology of Allergic Rhinitis
1.4 Treatment
1.5 Subpopulations with Special Considerations
1.6 Conclusion
2 Allergen Immunotherapy
2.1 Immune Changes Associated with AIT
2.2 Indications for Initiation and Efficacy of AIT
2.3 AIT Vial Preparation
2.4 Dosing Administration, Safety, Adverse Effects
2.5 Clinical Considerations When Administering AIT
2.6 Conclusion
Bibliography
Otitis Media
Bibliography
Asthma
1 Introduction
Table 1 Asthma severity classification
Case 1
uses ashort-acting B agonist (SABA) once a day and wakes withsymptoms 4 nights per week. Her forced expiratory volumein 1 min (FEV1) was 80% of the predicted value.
How would you treat her?
Table 2 Asthma treatment recommendations according to severity and age
Asthma treatment: > = 12 years old
Asthma treatment: 5–11 years old
persisted uncontrolled with an asthma control test scoreof 12
The patient is still uncontrolled despite her treatment with an ICS/LABA.
IgE was 323, and her eosinophil count was 125 μg/ml. She weighs 80 kg. Assuming the patient is compliant with treatment, uses adequate technique, and that comorbidities are controlled
a long-term pulmonary change seen in asthma?
Case 2
risk factors for persistent wheezing in children?
an inciting factor for asthma?
increased nasal rhinorrhea and a low-grade fever. The next day he started wheezing.
the role of fractional exhaled nitric oxide (FeNO) in the evaluation of asthma?
2 Conclusion
Bibliography
Cough and Allergic Diseases
1 Introduction
Table 1 Etiologies of chronic cough in adults and children
Case Presentation 1
2 Conclusion
Bibliography
Hypersensitivity Pneumonitis
1 Introduction
2 Past Medical History
3 Social History
4 Physical Exam
5 Investigations
5.1 Clinical Presentation of Nonfibrotic HP
5.2 Diagnosis of Nonfibrotic HP
5.3 Exposure History
5.4 Pathophysiology of HP
5.5 Treatment of Nonfibrotic HP
6 Past Medical History
7 Social History
8 Physical Exam
9 Investigations
9.1 Clinical Presentation of Fibrotic HP
9.2 Differential Diagnoses
9.3 Pathophysiology of HP
9.4 Diagnosis of HP
9.5 Management and Prognosis of HP
Bibliography
Allergic Bronchopulmonary Aspergillosis
1 Introduction
2 Definition
3 Risk Factor for ABPA
4 Pathogenesis (Fig. 1)
5 Clinical Features
6 Radiographic Abnormalities Seen in ABPA
7 Differential Diagnosis
8 Diagnosis of ABPA
9 Diagnostic Criteria of ABPA
10 Staging
10.1 Clinical Staging
10.2 Radiological Staging
11 Management
11.1 Treatment
Bibliography
Food Allergy
Bibliography
Eosinophilic Disorders
1 Introduction
2 Eosinophil Biology
3 Eosinophilia
4 Secondary Versus Primary Eosinophilic Disorders
5 Eosinophilic Gastrointestinal Disorders
6 Eosinophilic Gastritis, Eosinophilic Gastroenteritis, and Eosinophilic Colitis
7 Epidemiology and Risk Factors for Eosinophilic Esophagitis
8 Eosinophilic Esophagitis
9 Clinical Presentation of Eosinophilic Esophagitis
10 Diagnosis of Eosinophilic Esophagitis
11 Pathogenesis of Eosinophilic Esophagitis
12 Treatment of Eosinophilic Esophagitis
13 Prognosis of Eosinophilic Esophagitis
14 Hypereosinophilia and Hypereosinophilic Syndromes
15 Hypereosinophilic Syndrome Variants
16 Approach to Patients with Hypereosinophilia and HES
Bibliography
A Case-Based Board Review of Angioedema and Urticaria
1 Introduction
Bibliography
Atopic Dermatitis
1 Introduction
Bibliography
Allergic Contact Dermatitis
1 Introduction
2 Presentations of Allergic Contact Dermatitis
3 Diagnosis of Allergic Contact Dermatitis
4 Summary
Further Reading
Insect Allergy
1 Summary
Bibliography
Latex Allergy
1 Introduction
2 Cases
3 Conclusion
Bibliography
Drug Allergy
Bibliography
Autoimmunity
1 Introduction
Bibliography
Vasculitis
1 Introduction
Bibliography
B Cell Deficiency
1 Introduction
2 X-Linked Agammaglobulinemia
2.1 Etiology and Pathophysiology of XLA
2.2 Clinical Manifestation of XLA
2.3 Infections in XLA
2.4 Autoimmune Disorders in XLA
2.5 Malignancies in XLA
2.6 Diagnosis of XLA
2.7 Differential Diagnosis
2.7.1 Transient Hypogammaglobulinemia of Infancy (THI)
2.7.2 Common Variable Immunodeficiency (CVID)
2.7.3 Autosomal Recessive Agammaglobulinemia (ARA)
2.8 Management of XLA
2.9 Complications Related to XLA
2.10 Prognosis of XLA
3 Common Variable Immunodeficiency
3.1 Etiology and Pathophysiology of CVID
3.2 Clinical Manifestation of CVID
3.2.1 Pulmonary Disease
3.2.2 Autoimmune Disease
3.2.3 Gastrointestinal Disease
3.2.4 Lymphoma and Cancers
3.3 Diagnosis of CVID
3.3.1 Differential Diagnosis of CVID
3.3.2 Secondary Hypogammaglobulinemia
3.3.3 Primary Hypogammaglobulinemia
3.3.4 Specific Antibody Deficiency
3.3.5 IgG Subclass Deficiency
3.3.6 Hyperimmunoglobulin M Syndrome (HIGM)
3.3.7 Combined Immunodeficiencies
3.4 Management of CVID
3.5 Prognosis
4 Summary
Bibliography
T Cell Deficiency
1 Introduction
2 Severe Combined Immunodeficiency (SCID)
2.1 Clinical Manifestations of SCID
2.2 Etiology of SCID
2.3 Laboratory Findings in SCID
2.4 Newborn Screening for SCID and TREC Analysis
2.5 Management of SCID
3 DiGeorge Syndrome
3.1 Etiology of DiGeorge Syndrome
3.2 Clinical Characteristics of DiGeorge Syndrome
3.3 Diagnosis of DGS
3.4 Management of DGS
4 Summary
Bibliography
Combined Immunodeficiency Disorders
1 X-Linked Hyper-IgM
1.1 Classification, Epidemiology, and Genetics of the Hyper-IgM Syndromes
1.2 Pathogenesis of Hyper-IgM Syndromes
1.3 Clinical Manifestations of X-Linked Hyper-IgM
1.4 Diagnosis and Laboratory Abnormalities in Hyper-IgM Syndrome
1.5 Treatment and Prognosis of X-Linked Hyper-IgM
2 Wiskott-Aldrich Syndrome
2.1 Genetics and Pathogenesis of Wiskott-Aldrich Syndrome
2.2 Clinical Manifestations and Diagnosis in Wiskott-Aldrich Syndrome
2.3 Laboratory Abnormalities in Wiskott-Aldrich Syndrome
2.4 Treatment of Wiskott-Aldrich Syndrome
2.5 Prognosis of Wiskott-Aldrich Syndrome
3 Summary
Bibliography
Human Immunodeficiency Virus (HIV) Infection and AIDS
1 HIV Epidemiology, Transmission, and Natural History
2 HIV Screening and Diagnosis
3 HIV Treatment Principles
4 Opportunistic Infections
5 HIV Prevention
6 Differential Diagnosis of Lung Lesions in a Patient with HIV
7 Conclusions
Bibliography
Anaphylaxis
1 Introduction
2 Pathophysiology of Anaphylaxis
3 Diagnosis of Anaphylaxis
4 Presentation of Anaphylaxis
5 Laboratory Testing in Anaphylaxis
6 Treatment and Prevention of Anaphylaxis
7 Differential Diagnosis of Anaphylaxis
8 Perioperative Anaphylaxis
9 Incidence, Etiology, and Prognosis of Anaphylaxis
10 Summary
Bibliography
Diagnostic Testing in Allergic Diseases
1 Introduction
2 Indication of Allergy Testing
3 Types of Allergy Testing
4 Mast Cells and Histamine
5 Utility of Allergy Testing in Allergic Diseases
Bibliography
Antihistamines and Corticosteroids
Bibliography
Bronchodilators
1 Introduction
Bibliography
New Asthma Therapeutics
1 Introduction
Bibliography
Vaccination
1 Introduction
Bibliography
Allergen Immunotherapy
Bibliography
Index
Recommend Papers

Absolute Allergy and Immunology Board Review (Oct 29, 2022)_(3031128664)_(‎Springer)
 9783031128660, 9783031128677

  • 0 0 0
  • Like this paper and download? You can publish your own PDF file online for free in a few minutes! Sign Up
File loading please wait...
Citation preview

Massoud Mahmoudi Editor

Absolute Allergy and Immunology Board Review

123

Absolute Allergy and Immunology Board Review

Massoud Mahmoudi Editor

Absolute Allergy and Immunology Board Review

Editor Massoud Mahmoudi Department of Medicine University of California San Francisco San Francisco, CA, USA

ISBN 978-3-031-12866-0    ISBN 978-3-031-12867-7 (eBook) https://doi.org/10.1007/978-3-031-12867-7 © The Editor(s) (if applicable) and The Author(s), under exclusive license to Springer Nature Switzerland AG 2022 This work is subject to copyright. All rights are solely and exclusively licensed by the Publisher, whether the whole or part of the material is concerned, specifically the rights of translation, reprinting, reuse of illustrations, recitation, broadcasting, reproduction on microfilms or in any other physical way, and transmission or information storage and retrieval, electronic adaptation, computer software, or by similar or dissimilar methodology now known or hereafter developed. The use of general descriptive names, registered names, trademarks, service marks, etc. in this publication does not imply, even in the absence of a specific statement, that such names are exempt from the relevant protective laws and regulations and therefore free for general use. The publisher, the authors, and the editors are safe to assume that the advice and information in this book are believed to be true and accurate at the date of publication. Neither the publisher nor the authors or the editors give a warranty, expressed or implied, with respect to the material contained herein or for any errors or omissions that may have been made. The publisher remains neutral with regard to jurisdictional claims in published maps and institutional affiliations. This Springer imprint is published by the registered company Springer Nature Switzerland AG The registered company address is: Gewerbestrasse 11, 6330 Cham, Switzerland

To the memory of my father, Mohammad H, Mahmoudi, and to my mother, Zohreh, my wife, Lily, and my sons, Sam and Sina, for their continuous support and encouragement.

Preface

Every year hundreds of allergy and immunology fellows in training and practicing allergists take board examination. My personal search for finding prep books led to disappointment as there were limited resources available for such an important task. This prompted me to search for a distinguished team of academicians to help in preparing the present collection. I have been fortunate to gather over 70 national experts in the field which comprises allergy and immunology fellowship training directors and academicians from various regional academic institutes and hospitals. This book consists of 31 chapters. Each chapter presents two cases and each case is followed by five questions/answers to stimulate the reader’s thought process and discussion about the topic. We understand that preparation for the board examination requires years of didactic preparation and hands-on clinical practice. We, the authors, collectively believe that the present book will help to serve as a brief review of common allergy and immunology topics and to serve as a medium to inspire the thought process for preparation of the board examination. Preparation of this book would not have been possible without the help and support of the publisher. I would like to express my thanks to Michelle Tam for overseeing this project and the editorial and production team at Springer who helped me at all stages of the preparation. Finally, I would like to thank Richard Lansing, Editorial Director of Clinical Medicine at Springer, who has been supportive in all my projects throughout the years with the publisher. I appreciate hearing your comments and suggestions for use in what I hope will be the second edition of this title. Please contact me at [email protected]. San Francisco, CA

Massoud Mahmoudi

vii

Contents

Innate and Adaptive Immunity�����������������������������������������������������������������������������������������   1 Kranthi Nomula, Hanadys Ale, and Christopher Chang  Immune Cells and Functions���������������������������������������������������������������������������������������������  13 Kenneth Paris and Andrew M. Abreo HistocompatIbility Antigens (HLA) and Transplantation ���������������������������������������������  21 Michell M. Lozano Chinga, David Buchbinder, and Jolan E. Walter  Allergic Diseases of the Eye�����������������������������������������������������������������������������������������������  43 Matthew R. Norris and Neeti Bhardwaj Allergic Rhinitis �����������������������������������������������������������������������������������������������������������������  57 Meera R. Gupta and Jessica Palmieri Otitis Media�������������������������������������������������������������������������������������������������������������������������  69 Elizabeth L. Wisner and Kenneth Paris Asthma���������������������������������������������������������������������������������������������������������������������������������  75 Sylvette Nazario Cough and Allergic Diseases ���������������������������������������������������������������������������������������������  83 Satoshi Yoshida Hypersensitivity Pneumonitis �������������������������������������������������������������������������������������������  95 Ria Gripaldo and Avanthika Thanushi Wynn Allergic Bronchopulmonary Aspergillosis ����������������������������������������������������������������������� 107 Mariam Majzoub and Rauno Joks Food Allergy������������������������������������������������������������������������������������������������������������������������� 113 Catherine A. Popadiuk and Doerthe A. Andreae Eosinophilic Disorders������������������������������������������������������������������������������������������������������� 123 Taha Al-Shaikhly, Matthew R. Norris, Heather Stern, and Gisoo Ghaffari  Case-Based Board Review of Angioedema and Urticaria������������������������������������������� 137 A Marisa Riley, Sally Ng, and Timothy Craig Atopic Dermatitis ��������������������������������������������������������������������������������������������������������������� 145 Iona Malinow Allergic Contact Dermatitis����������������������������������������������������������������������������������������������� 151 Ryan Steele Insect Allergy����������������������������������������������������������������������������������������������������������������������� 159 Donya Salmasinia Imanirad and Dennis Ledford Latex Allergy����������������������������������������������������������������������������������������������������������������������� 169 Abeer Siddiqi and Anu Mallapaty ix

x

Drug Allergy ����������������������������������������������������������������������������������������������������������������������� 175 Schuman Tam Autoimmunity��������������������������������������������������������������������������������������������������������������������� 185 Jack Jeskey, Lauren Fill, Madiha Huq, Sandeep Sarkaria, Remie Saab, and Robert Hostoffer Vasculitis ����������������������������������������������������������������������������������������������������������������������������� 193 Nikita Patel and Stratos Christianakis B Cell Deficiency����������������������������������������������������������������������������������������������������������������� 203 Aishwarya Navalpakam and Pavadee Poowuttikul T Cell Deficiency����������������������������������������������������������������������������������������������������������������� 223 Divya Seth and Pavadee Poowuttikul Combined Immunodeficiency Disorders ������������������������������������������������������������������������� 235 Jenny Huang and Pavadee Poowuttikul  Human Immunodeficiency Virus (HIV) Infection and AIDS����������������������������������������� 251 Claudiu Georgescu Anaphylaxis������������������������������������������������������������������������������������������������������������������������� 263 Jeffrey Kepes and Pavadee Poowuttikul Diagnostic Testing in Allergic Diseases����������������������������������������������������������������������������� 277 Massoud Mahmoudi Antihistamines and Corticosteroids ��������������������������������������������������������������������������������� 287 Tara Carr Bronchodilators������������������������������������������������������������������������������������������������������������������� 293 Sumeet Sandhu and Maria-Anna Vastardi New Asthma Therapeutics������������������������������������������������������������������������������������������������� 297 Katheryn Birch, Maaz Jalil, Craig Sewell, Marija Rowane, Haig Tcheurekdjian, Devi Jhaveri, Theodore Sher, and Robert Hostoffer Vaccination ������������������������������������������������������������������������������������������������������������������������� 307 Dhami Jaspreet, Wang Vivian, Wang Ziwei, Pham Brittney, Yabuno Jamie, and Joseph Yusin Allergen Immunotherapy��������������������������������������������������������������������������������������������������� 317 Karla E. Adams and James M. Quinn Index������������������������������������������������������������������������������������������������������������������������������������� 329

Contents

Innate and Adaptive Immunity Kranthi Nomula, Hanadys Ale, and Christopher Chang

1 Introduction The human immune system is a complex system of cells and molecules that resist and protect the body from infections, toxins, invasion by other foreign bodies, and tumors. Conceptually, the reason why organisms need an immune system is to ward off dangerous elements that pose a threat to their health and well-being. These dangerous elements may be exogenous or may originate from the host. The immune system must be programmed to be able to recognize what is dangerous and what is benign, otherwise our bodies will reject essential elements in our body and the surrounding environment that are necessary for our survival as a species. Examples of dangerous and non-dangerous exposures can be seen in Table 1. The immune system can be broadly divided into innate and adaptive immunity. It is important to recognize that this is an artificial distinction. In fact, there are many parts of the immune system that span both innate and adaptive immunity. Other elements may bridge innate and adaptive immunity in order to allow the immune system to function to its most efficient capability. The major differences between the innate and adaptive immunity are mentioned in Table 2.

Table 1  Dangerous and non-dangerous entities

Non-­ dangerous Dangerous

Endogenous (self, etc.) Fetuses, commensal bacteria Tumor cells

Exogeneous (non-self, external) Food, pets, clothes, medications

Pathogens (viruses, bacteria, fungal, mycobacterium)

Table 2  Differences between innate and adaptive immunity Antigen dependency Immunologic memory Components

Specificity

Diversity Response time after exposure

Innate immunity Antigen-independent

Adaptive immunity Antigen-dependent

Not present

Present

Skin, epithelium, complement system, phagocytes, NK cells Nonspecific. Mounts the same response for molecules shared by related microbes and molecules Limited diversity Immediate. Develops within minutes to few hours

Lymphocytes

Specific. Mounts response specific to the antigens Large diversity Delayed. Develops within hours to days

2 Innate Immunity Case 1 A 3-month-old baby develops recurrent skin infections. Culture of the lesions shows Burkholderia cepacia. K. Nomula Department of Pediatrics, Joe DiMaggio Children’s Hospital, Memorial Healthcare System, Hollywood, FL, USA e-mail: [email protected]

Question 1 The test that would most likely reveal the diagnosis is

H. Ale · C. Chang (*) Division of Immunology, Allergy and Pediatric Rheumatology, Joe DiMaggio Children’s Hospital, Memorial Healthcare System, Hollywood, FL, USA e-mail: [email protected]

A. Lymphocyte subset panel B. Natural Killer cell function C. Neutrophil oxidative burst assay D. Toll-like receptors

© The Author(s), under exclusive license to Springer Nature Switzerland AG 2022 M. Mahmoudi (ed.), Absolute Allergy and Immunology Board Review, https://doi.org/10.1007/978-3-031-12867-7_1

1

2

K. Nomula et al.

Answer and Explanation Answer: B While answers A, C, and D are all components of the innate immune system, unusual infections such as Burkholderia and Nocardia are commonly seen when phagocytes are unable to kill microorganisms. This is generally due to a defect in nicotinamide adenine dinucleotide phosphate (NADPH) oxidase, which generates reactive oxygen species (ROS) to kill pathogens. Chronic granulomatous disease is a severe primary immunodeficiency in which variants in genes coding for NADPH oxidase render the enzyme ineffective, leading to infections that would normally not be seen in healthy individuals. Question 2 Which of the following is a feature of innate immunity? A. B. C. D.

Diversity Rapidity Specificity Strength

Answer and Explanation Answer: B The advantage of the innate immune system is that it relies on the presence of highly conserved moieties in pathogens or other dangerous external threats to mount an initial rapid response. What it does not have is a finely tuned specific response to a particular antigen. It also does not have the ability to remember previous exposures so that a subsequent exposure will result in an amplified response. One way to think of the difference between innate and adaptive immunity is that innate immunity is akin to a shotgun, whereas adaptive immunity can be represented by a precision scoped rifle operated by a skilled marksman to kill its target. Innate immunity is an immediate and first line of defense against microorganisms and other dangerous entities. This form of immunity is genetically determined since it uses germline-encoded receptors that recognize microbial products and other molecules. It is dependent on the immune system’s “innate” ability to recognize molecular characteristics that are present in a wide variety of organisms or other dangerous entities. The advantage of this response is that it can be generated quickly, usually within minutes to a few hours after exposure to a pathogen. However, speed of response comes at a price, and innate immune responses are weaker and more temporary than those of the adaptive immune system. The innate immune system consists of multiple components that will be covered in this chapter.

Question 3 Which TLR is paired correctly with its ligand? A. TLR-1/2 and CpG B. TLR-3 and Poly I:C C. TLR-5 and Lipopolysaccharide D. TLR-8 and bacterial flagellin Answer and Explanation Answer: B Lipopolysaccharide was the PAMP that Bruce Beutler studied which eventually won him the Nobel Prize in Physiology and Medicine in 2011 for his work on the activation of innate immunity. Lipopolysaccharide is the ligand for TLR-4. The ligand for TLR-5 is bacterial flagellin while the ligand for TLR-9 is CpG.  The TLRs and their ligands are shown in Table 4. Polyinosinic:polycytidylic acid (Poly I:C) is an immunostimulant that can simulate viral infections. It is structurally similar to double-stranded RNA, the natural stimulant of TLR3. The response is mounted after recognition of pathogen-­ associated molecular patterns (PAMPs) or danger (or damage)-associated molecular patterns (DAMPs) by corresponding molecular entities on the immune cell known as pathogen recognition receptors (PRR). PAMPs are conserved molecular sequences shared by microbes that mount an innate immune response. Examples of PAMPs include lipopolysaccharides, teichoic acid, double-stranded RNA, and flagellin. Examples of PRRs include toll-like receptors (TLRs), nucleotide oligomerization domain (NOD)-like receptors, retinoic acid-inducible gene (RIG)-like receptors, C-type lectin receptors (CLRs), and scavenger receptors (SRs). The structure and function of these receptor classes are shown in Table 3. The most well-known group of PRRs are the toll-like receptors which bind to specific molecules that are part of the PAMP. Some of these entities cross-react with synthetic molecules which allow for measurement of the function of TLRs in individual patients. Some of these molecules along with recognized ligands of TLRs are illustrated in Table 4. The mechanisms of innate immunity can be categorized under anatomic, physiologic, phagocytic, and inflammatory responses. Table 5 shows the various barriers and their role in the immune system. PRRs upon binding to PAMPs lead to the production of pro-inflammatory cytokines which contribute to the removal or destruction of the pathogens. These pro-inflammatory cytokines are often responsible for the clinical symptoms or signs that occur when the body is mounting a defense against a pathogen. The reaction can be

Innate and Adaptive Immunity

3

Table 3  Pattern recognition receptors Class of pattern recognition receptor C-type lectin receptors (CLR)

Nucleotide oligomerization domain (NOD)-like receptors

Retinoic acid-inducible gene (RIG)-like receptors

Toll-like receptors

Scavenger receptors

Structural characteristics Contains a highly conserved domain for the recognition of carbohydrates in a calcium iondependent manner Cytoplasmic PRRs consist of a NOD domain, a domain for initiation of signaling (e.g., caspase activation and recruitment domain (CARD)), pyrin domains, or baculovirus inhibitor of apoptosis repeat (BIR) domains Members of DEAD-­box (SF2) helicases. Binds viral RNA through the C-terminal domain. Catalytic helicase core consists of nine highly conserved sequence motifs which function to unwind RNA Type I integral transmembrane proteins usually with an N-terminal domain external to the membrane, a middle single helix transmembrane domain within the membrane, and a C-terminal domain in the cytoplasm. The N-terminal domain contains the ligand recognition site, whereas the C-terminal domain contains effector moieties for downstream signal transduction Consist of an N-terminal cytoplasmic tail, a C-terminal scavenger receptor cysteine-rich (SRCR) domain, as well as a spacer region, collagenous domain, and an α-helical coiled-coil domain

Table 4  Toll-like receptors, NOD-like receptors, and their ligands Pattern recognition receptor TLR-1 TLR-2 (CD282) TLR-3 TLR-4

TLR-5 TLR-6 TLR-7

TLR-8

TLR-9 NOD-1 (CARD4) NOD-2 (CARD15)

Ligand(s) Triacyl lipopeptides, PAM3CSK4a Glycolipids, proteolipids, lipopeptides, lipoteichoic acid, HSP70, Zymosanb, PAM3CSK4a Double-stranded RNA, poly I:C Lipopolysaccharide (LPS), heat shock proteins, fibrinogen, heparin sulfate, hyaluronic acid, nickel, opioid drugs Bacterial flagellin, profiling Diacyl lipopeptides, Zymosanb Single-stranded RNA, imidazoquinolone (CLO97)c, resiquimod, bropiramine, loxoribine Single-stranded viral RNA, small synthetic compounds, imidazoquinolone (CLO97)c Unmethylated CpG oligodeoxynucleotide DNA Peptidoglycan iE-DAP dipeptide

Adapter protein MyD88/MAL MyD88/MAL

Anatomic Physiologic

Removal of foreign substances and waste in the body, plays a role in homeostasis, apoptosis, inflammation, and clearance of pathogens

Barrier Skin and mucous membranes Temperature

Low pH Complement system Chemical mediators  Lysozyme

MyD88 MyD88/MAL MyD88

MyD88 Phagocytic MyD88

Compounds used to test for TLR function are in red a  TLR1/TLR2 ligand b  TLR2/TLR6 ligand c  CLO is a water-soluble derivative of the imidazoquinolone R848; (Resiquimod) and is a TLR7/TLR8 ligand

Detection of intracytoplasmic bacterial products, such as bacterial peptidoglycans including mesodiaminopimelic acid (NOD1) and muramyl dipeptide (NOD2), also plays a role in the activation of the inflammasome Key sensors of viral infection detect atypical nucleic acids and plays a role in host antiviral responses by mediating transcriptional induction of type 1 interferons Binds various ligands on the surface of viral and bacterial pathogens (see Table 4)

Table 5  Mechanism of innate immunity

TRIF MyD88/ MAL,TRIF, TRAM

Peptidoglycan—Muramyl dipeptide (MDP)

Function Binds sugar molecules and plays a role in recognizing carbohydrate structures on the surface of pathogens

Inflammation

 Beta lysin  Lactic acid  Lactoperoxidase Macrophages, dendritic cells, mast cells

Function Protect against invasion of foreign organisms High temperature destroys infectious organisms. Also, produces interferons and helps recovery from infections High acidity of stomach kills microorganisms Bactericidal action against pathogenic bacteria Breaks down the bacterial cell wall Antibacterial action Antibacterial action Antibacterial action Destroy microorganisms by phagocytic action Tissue injury leads to mobilization of phagocytic cells to the damaged area by means of inflammation

4 Fig. 1  Components of the innate and adaptive immune systems. (Source: Bioactive Nanoparticles for Cancer Immunotherapy - Scientific Figure on ResearchGate. Available from: https://www. researchgate.net/figure/ Cells-­and-­components-­ involved-­in-­the-­innate-­and-­ adaptive-­immune-­system_ fig1_329405685)

K. Nomula et al. Innate Immunity

Adaptive Immunity

Bridges between innate and adaptive immunity

Neutrophils

Cytokines

T cells

Memory Cytotoxic Effector Helper Regulatory

Basophils

Dendritic cells Eosinophils

Bcells

Macrophages

Complement

Memory Transitional Plasma

Bridges between innate and adaptive immunity

local or systemic. The hallmarks of this inflammation include monocytes form the immune component of phagocytic cells. heat, erythema, swelling, and pain, but when pro-­ Epithelial cells, dendritic cells, fibroblasts, and complements inflammatory cytokines are produced unchecked, the reac- are some of the nonimmune phagocytic cells. The functions tion could be systemic, which can result in destruction and and their role in the immune system of some of these cells failure of host organ systems, leading to significant morbid- will be discussed below. ity or even death. The innate immune response also primes the adaptive immune system, which allows for a much stronger, longer 3.2 Neutrophils and Neutrophil lasting, and more specific response. Components of the Extracellular Traps (NETs) innate and adaptive immunity are shown in Fig. 1. Neutrophils are one of the most efficient cells of the innate immune system. Neutrophils migrate to the site of infection 3 Components of the Innate Immune or inflammation by the process of chemotaxis. Neutrophils System recognize pathogen patterns unique to microorganisms known as PAMPs, because of the expression of toll-like 3.1 Phagocytes receptors (TLRs) and begin the process of phagocytosis. Neutrophils exhibit their phagocytic action by both, engulfCells that have a phagocytic function are known as phago- ment and secretion of neutrophil extracellular traps (NETs). cytes. Phagocytic cells can be broadly classified as immune Neutrophils have another interesting function by which they cells and nonimmune cells. Neutrophils, macrophages, and eradicate the microbes from the body, they extrude their

Innate and Adaptive Immunity

nuclear contents, such as their DNA and histones by undergoing cell death in a process called NETosis. These contents are then referred to as neutrophil extracellular traps. As the name indicates, they trap and eliminate the pathogen. Question 4 Which cytokine released by macrophages drives interferon-γ production by Th1 cells? A. B. C. D.

IL-2 IL-4 IL-12 IL-17

Answer and Explanation Answer: C IL-12 is a heterodimer that is secreted by macrophages, dendritic cells, and neutrophils upon stimulation by antigens. IL-12 stimulates the priming of CD4+ T cells and induces the production of interferon-γ, leading to the proliferation of Th1 cells. IL-2 is a T cell activator, and it will activate all subsets of T cells, but is obligatory for T regulatory cell proliferation. IL-4 is a Th2 cytokine which stimulates allergic reactions, while IL-17 is secreted by Th17 cells. A Th17/ Treg cell imbalance that is skewed towards Th17 plays a role in the development of autoimmune diseases.

3.3 Monocytes and Macrophages Monocytes are large white blood cells when migrated to the tissues are called macrophages. The major function of macrophages is phagocytosis and antigen presentation. Macrophages express Major Histocompatibility Complex (MHC) via which antigens are presented for recognition by lymphocytes. Another function of macrophages that plays an important role in the immune system is the production of cytokines. The production and resolution of cytokines are very intricately woven and any disruption in this leads to hyperinflammatory syndromes, such as macrophage-­activating syndrome (MAS) or hemophagocytic lymphohistiocytosis (HLH).

3.4 Innate Lymphoid Cells Innate lymphoid cells are bone marrow-derived cells with lymphocyte morphology that are predominantly residential to the lymphoid and nonlymphoid tissues and rarely found in circulation. They are present in large quantities at the mucosal surfaces. They do not have antigen-specific receptors, but they express receptors that are able to recognize cytokines. Although their overall numbers are not massive, their role in

5

immunity is still significant. Three major types of innate lymphoid cells-ILC1, ILC2, and ILC3 have been identified. Imbalance in the function of innate lymphoid cells has been shown to cause respiratory infections, asthma, allergic rhinitis, chronic obstructive pulmonary disease, cancer, and gastrointestinal infections among others.

3.5 NK and NK-T Cells Natural killer cells or NK cells are large granular lymphocytes that have protective action against virally infected cells and tumor cells. NK cells recognize antigens presented by MHC I and eliminate the infected cells by apoptosis. NK cells activity is measured by granzyme and perforin activity and CD107a expression. NK cells are not to be confused with NK-T cells, which possess T cell receptors but lack cytoplasmic granules. However, both NK cells and NK-T cells have a role in autoimmunity.

3.6 Epithelial Cells Epithelial cells have an important role in the instigation and maintenance of both innate and adaptive immunity. The function of epithelial cells is of utmost importance in the innate immune system in the maintenance of protective barriers such as the skin. Like innate lymphoid cells, disruption of the normal functioning of epithelial cells has been shown to cause respiratory infections, asthma, allergic rhinitis, chronic obstructive pulmonary disease, cancer, and gastrointestinal infections among others. Question 5 Deficiencies in which complement factor can lead to Neisseria meningitides infection A. B. C. D.

C2b C4 Factor B Properdin

Answer and Explanation Answer: D In general, complement deficiencies are rare. The same can be said of properdin deficiency. Properdin is the only positive regulator of the alternative complement pathway. Properdin is released by activated neutrophils, macrophages, and T cells and functions to stabilize the C3 convertase by binding with C3b thereby preventing cleavage by Factor H and Factor I. Properdin deficiency has been associated with severe Neisseria meningitidis infection. Deficiency of C4 is

6

K. Nomula et al.

associated with autoimmune disease, especially systemic lupus erythematosus (SLE), while Factor B deficiency has even more rarely been reported. C2b is a cleavage product of C2 and is a component of the C3 convertase of the classical complement pathway.

of two pneumonias per year for the past 2 years and a history of recurrent ear infections which required pressure equalizer (PE) tube surgery. Ear infections resolved after the surgery. On immune evaluation, she was noted to have sub-protective pneumococcal antibody titers for 18 of 23 serotypes. Her immunizations are up-to-date.

3.7 Complement

Question 1 What is the most appropriate next step?

The complement system consists of several heat-labile proteins that represent an important branch of innate immunity. These unique set of plasma proteins act as a bridge between innate and adaptive immunity. The Complement pathways as the name suggests complement the innate immune system and help in the destruction of pathological microorganisms via opsonization, phagocyte recruitment, and direct killing of microbes. Classical, alternative, and lectin pathways are the major pathways of the complement system. Although the names and the initiating mechanisms of the three pathways are different, they lead to the production of similar effector molecules. Opsonization, the process by which cells are rendered favorable for phagocytosis is mediated by the complement system. Byproducts of the complement pathways act as chemoattractants and amplify the immune response. The terminal complement is formed by a complex of proteins (C6, C7, C8, and C9) that are assembled into a membrane pore called the membrane attack complex (MAC) and damage bacterial cells by cell lysis. Question 6 Delayed separation of the umbilical cord should raise suspicions of a defect in A. B. C. D.

CD18 CD25 CD40 CD146

Answer and Explanation Answer: A Delayed separation of the umbilical cord has long been accepted as a sign of leukocyte adhesion defect (LAD). There are two main forms of LAD, LAD1 and LAD2. LAD1 results from defects in the ITGB2 gene, which encodes for CD18. CD18, along with CD11b is a component of complement receptor 3 (CR3).

4 Adaptive Immunity Case 2 A 4-year-old female has a history of recurrent upper respiratory infections since age 6 months. She also has had a history

A. Administer pneumococcal polysaccharide vaccine and repeat titers after 4–6 weeks B. Obtain genetic testing C. Tonsillectomy/adenoidectomy D. Treat with prophylactic antibiotics Answer and Explanation Answer: A If a 4-year-old patient has a history of recurrent infections and pneumococcal titers are low, the first step would be to test humoral immunity function. We can do this by testing the patient’s response to a vaccine. One would expect that a patient with a functioning humoral immune system would be able to produce specific antibodies to the majority of pneumococcal serotypes following vaccination. The generally accepted threshold for protection for most laboratories is 1.3 μg/mL, although this is somewhat of an arbitrary standard. In addition, there is some disagreement as to how many of the serotypes would be required to show a response. Most immunologists would agree that at least 60% of the serotypes should respond. Another way to interpret the results is that a response is acceptable when there is a fourfold rise in titers, but this may not always be present especially if titers of some of the serotypes are high to begin with. For this patient, it is probably too early in the evaluation to treat with prophylactic antibiotics or to obtain genetic testing. Surgical intervention is also not indicated in this patient. Adaptive immunity is a more sophisticated type of immune response. It takes over from the innate immune system to generate a stronger and more specific response which takes longer to develop. Adaptive immunity is antigen-­ dependent and is mediated by lymphocytes. Lymphocytes are primarily divided into B lymphocytes and T lymphocytes. Other subsets of lymphocytes include Natural Killer (NK)-T lymphocytes which are T lymphocytes that have innate effector functions. These lymphocytes recognize glycolipid molecules in the context of MHC class I-related glycoprotein CD1d. Lymphocytes help the body eliminate pathogens by developing immunologic memory to mount rapid response if reinfected, and recognize self-antigens from nonself-­ antigens. The two facets of adaptive immunity are humoral

Innate and Adaptive Immunity

7

Table 6  Differences between humoral and cell-mediated immunity

Major mediator Other cells involved Intracellular vs extracellular microbe

Humoral Immunity Antibodies B lymphocytes, macrophages Works against extracellular microbes and the toxins produced by them Unprocessed antigens

Cell-mediated Immunity T lymphocytes Macrophages and natural killer cells Works against intracellular microbes

while the infant begins to produce his or her own immunoglobulin G around 6 months of age.

5.1 How Does the Adaptive Immune System Achieve Its Goals of Strength, Specificity, and Diversification?

The immune repertoire can refer to both humoral and cellular immunity. This term refers to the diversity of the immune system in recognizing and distinguishing millions of different epitopes on various molecules and organisms. The immune repertoire develops over time. Indeed, it is estimated that over 1600 genes are involved in the development of the immunity and cell-mediated immunity. Like innate and human immune system, and even before birth, our immune adaptive immunity, humoral and cell-mediated immunity are systems have the ability to recognize innumerable antigenic also codependent and function in unison to mount appropri- epitopes by virtue of various physiologic functions involved ate immune response. Table 6 shows the major differences in the development of tolerance. Through positive and negabetween humoral and cell-mediated immunity. tive selection, our immune system gradually learns to repel dangerous antigens while accepting safe ones, including self-antigens and maternal and environmental exposures. 5 Humoral Immunity The ability to enhance immune responses to certain antigens through positive and negative selection, and the mechanisms Question 2 of central and peripheral tolerance eventually shape an indiWhich receptor facilitates active transport of immunoglobu- vidual’s immune repertoire. lin across the placenta As our immune system matures and we are gradually exposed to new antigens, our cells differentiate into special A. FcγR1 ized cells that can recall previous exposures (as in memory B. FcγRIII cells), and cells that possess effector function. A broad range C. FcRn of antigens can ultimately be recognized by our immune sys D. FcεR1 tem through biochemical processes such as V(D)J recombination and somatic hypermutation. V(D)J recombination Answer and Explanation contributes to the assembly of the variable domain of immuAnswer: C noglobulin and T cell receptor genes. This process is reguIgG is the only immunoglobulin class with the ability to lated by precise DNA cleavage at short conserved sequences cross the placenta. It does so through an active transport under the control of Recombination-activating gene (RAG) mechanism facilitated by the neonatal Fc receptor (FcRn). proteins. Further modification of immunoglobulin genes Humoral immunity or antibody-mediated immunity is takes place through somatic hypermutation which generates mediated by B lymphocytes. Receptors on B lymphocytes, mutations in variable gene segments leading to affinity matuB cell receptors (BCRs) recognize antigens on antigen-­ ration. This satisfies the property of amplification, generatpresenting cells (APCs). Cytokines help in proliferation of ing a stronger response, in adaptive immunity. the B cells and help in the transformation of B cells into Isotype class switching, on the other hand, alters the plasma cells. Plasma cells are antibody powerhouses that heavy chain immunoglobulin constant region to produce bind to the extracellular pathogens or toxins produced by antibodies other than IgM. This process dictates the effector the pathogens. The binding of antigen and antibody function of the antibody molecule. Somatic hypermutation enhances the process of phagocytosis. This is called and heavy chain isotype (class) switching are under the conopsonization. trol of activation-induced cytidine deaminase (AID) which B cells produce five types of immunoglobulins: IgG, IgA, causes DNA deamination of transcribed target DNA. All of IgM, IgD, and IgE. Table 7 provides an overview of the prop- these processes contribute to generate the characteristics of erties and functions of these immunoglobulins. In the latter adaptive immunity—a stronger response with tremendous parts of pregnancy, IgG is actively transported across the pla- diversity and specificity. The molecular details of how this centa to provide passive immunity to the fetus and newborn happens are beyond the scope of this chapter. Processed vs unprocessed antigens

Antigens are processed and presented by MHC complex

K. Nomula et al.

8 Table 7  Properties and functions of the immunoglobulins Property Molecular weight (KDa) Subunits

IgG 153 Monomeric

Concentration in serum (varies with age and lab) Subclasses

560–1800 mg/dl 39–283 mg/dl

Complement binding Function

Serum half-life (days)

IgG1, IgG2, IgG3, IgG4 Yes Sustained immune response ~23

IgM 950 Pentameric

IgA 162 Monomeric in serum, dimer in secretions 82–470 mg/dl

IgD 185 Monomeric

~100kU/L, but varies with age IgE

Very small amounts (0.25% of immunoglobulins) IgD No Signal for B cell activation, activator of mast cells and basophils 2.8 days (secreted IgD)

IgM

IgA1, IgA2

Yes Early response to pathogens

No Mucosal immunity

No Allergic diseases, parasitic infections

5–6

5–6

2–3

6 Cell-Mediated Immunity Cell-mediated immunity is mediated by T lymphocytes. T lymphocytes do not produce antibodies. Instead, the receptors on T cells, or T cell receptors (TCRs), recognize the proteins presented by the major histocompatibility complex (MHC) and modulate the immune response by stimulating B cells to produce antibodies. The interaction between antigen-­ presenting cells and T lymphocytes is a critical step in the completion of an immune response. In addition to the effects on B cells, T lymphocytes also drive the production of cytokines for an amplified immune response and can lead to inflammatory responses in the same way that the innate immune system drives these responses. These inflammatory responses when not regulated appropriately can lead to a hyperinflammatory state that can result in multiorgan system damage and failure. In addition, cytotoxic T lymphocytes may also induce apoptosis of pathogen-infected cells or tumor cells. Other functions of T lymphocytes include macrophage activation which can lead to phagocytosis and oxygen-­radical induced killing of microbes. Cell-mediated immunity is effective in the destruction of not just intracellular pathogens, but also of virus-infected cells and tumor cells.

7 Components of the Adaptive Immune System Question 3 Which mitogen stimulates proliferation of both B and T lymphocytes A. Concanavalin A B. Lipopolysaccharide

IgE 190 Monomeric

C. Phytohemagglutinin A D. Pokeweed Mitogen Answer and Explanation Answer: D Concanavalin A and Phytohemagglutinin A are T cell mitogens. Lipopolysaccharide is a non-lectin mitogen for human B cells. Pokeweed Mitogen can stimulate both human T and B cells to proliferate.

7.1 B Lymphocytes B lymphocytes are generated and mature in the bone marrow. After B lymphocytes are released into the peripheral circulation, the process of maturation does not begin until they encounter an antigen. The functions of B cells are antigen presentation, cytokine secretion, and antibody production. The common lymphoid progenitor progresses to form the pro-B cells which further progress to form pre-B cells. Pre-B cells express “μ” heavy chain and “κ” or “λ” light chains. Pre-B cells mature to form immature B cells and ultimately mature B cells, which express both IgM and IgD receptors. The B cell receptor is a transmembrane immunoglobulin complex that recognizes antigens.

7.2 T Lymphocytes T cells originate in the bone and mature in the thymus. There are two types of lymphocytes—helper T cells and cytotoxic T lymphocytes (CTL). The T cell receptor is incapable of recognizing antigens unless processed and presented by the antigen-presenting cells (APCs) via the major histocompati-

Innate and Adaptive Immunity

bility complex (MHC). Cytotoxic T cells recognize antigen patterns when presented by the MHC class I. Once activated, cytotoxic T cells induce apoptosis of the infected cells. Helper T cells do not have cytotoxic action. Helper T cells recognize antigen patterns presented by the MHC class II. This leads to the activation of the helper T cells and production of cytokines that synchronize the activity of other cells such as APCs, B cells, and CTLs. The function of T and B lymphocytes can be evaluated using mitogens to stimulate their growth and proliferation.

7.3 Antigen-Presenting Cells Antigen-presenting cells (APCs) play a mediator role in the adaptive immune system. APCs possess the MHC which enables antigen presentation to T lymphocytes. Although many cell lines have the capacity to present antigens, dendritic cells, B cells, and macrophages are designated as “professional” antigen-presenting cells. Professional APCs are efficient in internalizing, processing, and expressing antigens on the MHC. Dendritic cells are present at the interfaces between the body and the environment, such as the skin, mucosal lining of nose, stomach, and intestines. Question 4 An inhibitory molecule in the T cell receptor which helps to regulate activation of a T cell response is A. B. C. D.

CD28 CD80 CTLA-4 LAT

Answer and Explanation Answer: C CD28 is a costimulatory molecule within the T cell receptor. CD80, also known as B7–1, is on B cells and interacts with CD28. It also reacts with CTLA-4, but in an inhibitory manner. LAT, or Linker for Activation of T cells, is an adaptor protein within the TCR signaling pathway.

7.4 The Function of CD4+ and CD8+ T Cells When the body encountered a dangerous external antigen, the epithelial barrier offers the first line of protection to

9

prevent entry of the antigen into the body. If the antigen escapes this initial protection, they may be taken up by antigen-­presenting cells and presented to the T cell receptor on the surface of T lymphocytes by the MHC on the surface of the APC.  The interaction of the TCR and the MHC induces the release of cytokines which modulates the immune response. For invading organisms, this contributes to the “­amplification” property of the adaptive immune system, although an opposite effect may also be possible if the antigen is deemed not to be dangerous. T cells can differentiate into helper CD4+ T cells or cytotoxic CD8+ T cells. CD4+ T cells are activated after antigen presentation by the MHC Class II complex. These cells have no inherent phagocytic activity, but act as mediators and produce cytokines such as interferon-γ that assist in the elimination of the pathogen via macrophage activation. Activated CD4+ T cells help B cells to undergo clonal expansion which leads to the production of large amounts of antibodies. These are known as effector B cells or plasma cells. These antibodies can bind to dangerous antigens to induce their inactivation or removal from the body. Cytotoxic CD8+ T cells are activated after presentation of the antigen by MHC Class I. They also have no phagocytic activity but after interacting with MHC, clonal expansion of CD8+ Th cells ensues. As the name suggests, activated CD8+ T cells can initiate direct killing of cells that have been infected by a virus. As in the case of CD4+ cells, cytokines are also released which can lead to amplification of the response against viral pathogens that have been taken up by antigen-presenting cells. The transporter associated with antigen processing 1 (TAP1) mediates antigen transfer from the cytosol to the endoplasmic reticulum within the cell. Deletion of the genes encoding TAP1 impairs presentation of antigen by MHC Class I. Another component of the MHC Class I complex is β-2microglobulin, which is encoded by the B2M gene. β-2microglobulin is required for transport of MHC Class I to the plasma membrane. There are many other types of T cells. T regulatory cells help to temper the activity of activated CD4+ and CD8+ T cells. They are stimulated by the action of IL-2 and generally carry the surface marker FOXP3. These cells are important because they prevent the immune system from reacting to non-harmful antigens or even self-antigens. For that reason, they play a significant role in preventing autoimmune diseases (Fig. 2).

K. Nomula et al.

10 Fig. 2  CD4+ T cells and CD8+ T cells illustarted. (a) The function of CD4+ helper T cells (b) The function CD8+ cytotoxic T cells. (Source: https://opentextbc. ca/biology/ chapter/23-2-adaptiveimmune-­response/)

a CD4+ T cell

Binding of TCR on the surface of the T cell with the MHC Class II-antigen complex on the antigen presenting cell

APC Activated Helper T cell

TCR

MHC Class II

CD4+ T cell

Epitope APC

Clonal expansion

Cytokines

CD4+ T cell clone CD4+ T helper cells direct B cells to differentiate into antigen specific plasma cells

CD8+ cells activated by cytokines

CD8+ T cell

b CD8+ T cell MHC Class I

TCR Epitope

B cell

Perforins form pores in the plasma membrane and granzymes damage cellular proteins and cause apoptosis Infected cell Infected cell

Infected cell Interaction of MHC Class I-epitope complex with the TCR of a cytotoxic T cell leads to release of granzyme and perforin (represented by blue and brown dots respectively)

Question 5 Which is a T cell independent vaccine? A. DTaP B. Hepatitis B vaccine C. Pneumovax-23 D. SARS-CoV-2 mRNA vaccine Answer and Explanation Answer: C When our immune system detects protein antigens, these antigens are taken up by antigen-presenting cells and presented via MHC Class II complexes to the T cell receptor on

CD8+ T cell

This leads to lysis and death of the infected cell

T helper cells, which then trigger antigen-specific B cells to produce antibody. This process occurs for not only natural epitopes but vaccines. Thus, a protein conjugate vaccine such as DTaP or a recombinant protein vaccine such as Hepatitis B would require T cell cooperation with B cells to mount a humoral response. However, other nonprotein antigens, such as polysaccharides, as is the case in the Pneumovax or PPSV23 vaccine, directly stimulate production of antibody by B cells without the help of T helper cells. These are referred to as T cell independent vaccines. In the case of SARS-CoV-2 mRNA vaccines, the mRNA is packaged along with the translation apparatus necessary to generate spike protein epitopes, and theoretically this would then go through

Innate and Adaptive Immunity

the same mechanism as other protein antigens through T cell receptor recognition of MHC Class II presentation of these antigens to stimulate antibody production by B cells.

8 Summary and Conclusions With each year, more and more information regarding the immune system is being revealed. There are millions of molecules, cells, networks, pathways, and interactions that must function in concert to maintain good health. For the purpose of organizing these complex interactions, concepts such as innate and adaptive immunity have been introduced in recent years. These concepts help build a foundation for understanding the immune system. It would be ludicrous to try and cover all the knowledge that has been accumulated over the years on human immunity and it would be impossible for any textbook to achieve this feat, let alone a single chapter. Therefore, it was not the intent of this chapter to discuss everything about innate and adaptive immunity, but to introduce the foundations on which to build the student’s understanding and organization of this knowledge. It is important to remember that these classifications are artificial and not perfect, as innate immunity and adaptive immunity are not two completely separate entities but are interconnected by multiple cells and molecules that support a functioning and effective operation of our immune system to fend off dangerous elements and maintain human health.

11

Bibliography 1. Alberts B, Johnson A, Lewis J, et al. Molecular biology of the cell. 4th ed. New York: Garland Science; 2002. Innate Immunity. https:// www.ncbi.nlm.nih.gov/books/NBK26846/ 2. Alberts B, Johnson A, Lewis J, et  al. Molecular biology of the cell. 4th ed. New  York: Garland Science; 2002. Chapter 24, The Adaptive Immune System. https://www.ncbi.nlm.nih.gov/books/ NBK21070/ 3. Schleimer RP, Kato A, Kern R, Kuperman D, Avila PC. Epithelium: at the interface of innate and adaptive immune responses. J Allergy Clin Immunol. 2007;120(6):1279–84. https://doi.org/10.1016/j. jaci.2007.08.046. 4. Merle NS, Noe R, Halbwachs-Mecarelli L, Fremeaux-Bacchi V, Roumenina LT.  Complement system part II: role in immunity. Front Immunol. 2015;6:257. https://doi.org/10.3389/ fimmu.2015.00257. 5. Kaplan MJ, Radic M.  Neutrophil extracellular traps: doubleedged swords of innate immunity. J Immunol (Baltimore, Md: 1950). 2012;189(6):2689–95. https://doi.org/10.4049/ jimmunol.1201719. 6. Lim JJ, et  al. Diversity and versatility of phagocytosis: roles in innate immunity, tissue remodeling, and homeostasis. Front Cell Infect Microbiol. 2017;7:191. https://doi.org/10.3389/ fcimb.2017.00191. 7. Panda SK, Colonna M.  Innate lymphoid cells in mucosal immunity. Front Immunol. 2019;10:861. https://doi.org/10.3389/ fimmu.2019.00861. 8. Vivier E, Tomasello E, Baratin M, et al. Functions of natural killer cells. Nat Immunol. 2008;9:503–10. https://doi.org/10.1038/ ni1582. 9. Rudolph MG, Stanfield RL, Wilson IA.  How TCRs bind MHCs, peptides, and coreceptors. Annu Rev Immunol. 2006;24:419–66.

Immune Cells and Functions Kenneth Paris and Andrew M. Abreo

Case 1 A 7-month-old boy with failure to thrive and recurrent pneumonia was admitted to the hospital in respiratory distress. Past medical history was notable for two previous episodes of pneumonia and suppurative lymphadenitis requiring incision and drainage with wound cultures growing normal flora. There is no family history of primary immunodeficiency. His birth history was unremarkable. The patient’s symptoms included productive cough and tachypnea. Clinical symptoms did not improve despite antimicrobial therapy for community-acquired pneumonia. Computed tomography (CT) scan of the chest was obtained which revealed mediastinal lymphadenopathy and right lower lobe consolidation with scattered abscesses (Fig.  1). Bacterial and fungal blood cultures were negative throughout the admission. IR-guided drainage of a lung abscess revealed growth of Phaeoacremonium parasiticum (Fig. 2). The patient was treated with broad-spectrum antibiotic and antifungal therapy. The neutrophil oxidative burst was abnormal compared to normal control. The remainder of the immunologic evaluation was normal. Genetic testing identified a pathogenic variant in CYBB confirming the diagnosis of X-linked chronic granulomatous disease (CGD). Question 1 Which cytokine is critical for neutrophil recruitment? A. Interleukin 4 (IL-4) B. Interleukin 5 (IL-5) C. Interleukin 6 (IL-6) D. Interleukin 8 (IL-8) E. Interleukin 13 (IL-13) Answer: The correct answer is D K. Paris (*) · A. M. Abreo Division of Allergy and Immunology, Department of Pediatrics, Louisiana State University Health Sciences Center, Children’s Hospital New Orleans, New Orleans, LA, USA e-mail: [email protected]; [email protected]

Phagocytes, including neutrophils and macrophages, are a major component of the innate immune system. These cells are recruited to sites of infection to eliminate microbes and damaged cells. Neutrophils are the most abundant phagocyte in the bloodstream. They are also known as polymorphonuclear leukocytes (PMNs) because the nucleus is composed of multiple, connected lobules. Neutrophil production in the bone marrow is stimulated by granulocyte colony-­stimulating factor (G-CSF) and granulocyte-macrophage colony-­ stimulating factor (GM-CSF). G-CSF and GM-CSF are produced by cells at the site of infection to replenish short-lived and depleted neutrophils. G-CSF interacts with the G-CSF receptor (CD114) on myeloid progenitor cells to drive the differentiation of myeloblasts and neutrophils. Neutrophils are recruited from the bone marrow to sites of infection by various inflammatory cytokines, such as interleukin 8 (IL-8, CXCL8) and leukotriene B4 (LTB4). Chemotaxis of neutrophils from the blood into tissues depends on choreographed interactions between selectins, cytokines, integrins, and cell-­ cell junction proteins. IL-8 (CXCL8) interacts with CXCR1 and CXCR2 on neutrophils to increase the affinity of the LFA-1 (CD11aCD18) and Mac-1 (CD11bCD18) integrins to their ligands ICAM-1 and ICAM-2 on the endothelial surface. Stable integrin-mediated arrest of neutrophils to the endothelium is required prior to transmigration into the tissue site. Question 2 Which neutrophil granule contains human alpha-defensins? A. B. C. D. E.

Specific granules Azurophilic granules Secretory granules Gelatinase granules Cytotoxic granules

Answer: The correct answer is B Neutrophils contain membrane-bound granules filled with preformed proteins. The most abundant are azurophilic

© The Author(s), under exclusive license to Springer Nature Switzerland AG 2022 M. Mahmoudi (ed.), Absolute Allergy and Immunology Board Review, https://doi.org/10.1007/978-3-031-12867-7_2

13

14

K. Paris and A. M. Abreo

Fig. 1  CT scan of the lung showing right lung pneumonia with parenchymal abscesses

(primary), specific (secondary), and gelatinase granules. Neutrophilic granule contents stain neutral pink with hematoxylin and eosin staining, unlike eosinophils (bright red) and mast cells (dark blue). The granules contain a variety of preformed antimicrobial peptides and proteases that contribute to the degradation of the extracellular matrix and microbicidal activity. Specific granules contain a variety of proteins including lactoferrin, cathelicidin, and matrix metalloprotease 8. Azurophilic granules contain human alpha-defensins, myeloperoxidase, proteinase 3, elastase, and cathepsin G/C. Defensins are cationic peptides that insert into the microbial membrane leading to disruption and death. Defensins and other granule contents also interact with chromatin extruded from neutrophils to form neutrophil extracellular traps (NETs). NETs capture microbes in the extracellular scaffold leading to microbial death but also neutrophil cell death.

Activated neutrophils are also capable of synthesizing cytokines and other inflammatory mediators. Question 3 Which helper T cell produces cytokines that contribute to neutrophil production and recruitment? A. T helper 1 cell (Th1) B. T helper 2 cell (Th2) C. T helper 9 cell (Th9) D. T helper 17 cell (Th17) E. T follicular helper cell (Tfh) Answer: The correct answer is D CD4+ effector T cells are divided into four subsets: T helper 1 cell (Th1), T helper 2 cell (Th2), T helper 17 cell

Immune Cells and Functions

15

Fig. 2  Left hilar lymph node biopsy, silver stain highlights Phaeoacremonium parasiticum fungal hyphae (GMS, final magnification 600×). (Image courtesy of Randall Craver, MD. Children’s Hospital New Orleans, LA)

(Th17), and T follicular helper cell (Tfh). Each subset produces distinct cytokines that contribute to different functions in host defense. Th17 cells develop under the influence of interleukin 1 (IL-1), interleukin 6 (IL-6), interleukin 23 (IL-­23), and TGF-β. Th17 cells produce interleukin 17A (IL-17A) and interleukin 22 (IL-22) in response to activation by bacteria and fungi. IL-17A induces the epithelium to produce G-CSF, IL-8 (CXCL8), and TNF-α. G-CSF stimulates the bone marrow to produce neutrophils. IL-8 (CXCL8) is a chemokine that recruits neutrophils to tissue. Therefore, IL-17A produced by Th17 cells indirectly leads to the production and recruitment of neutrophils. IL-22 is involved in the maintenance of the epithelial barrier but does not play a role in neutrophil chemotaxis. Abnormalities in the IL-17 pathway result in chronic mucocutaneous candidiasis due to defective recruitment of neutrophils to the site of Candida infections. Th1 cells produce IFN-γ to activate macrophages. Th2 cells produce IL-4, IL-5, and IL-13 to activate ­eosinophils and mast cells in defense against

parasitic infections and allergic disease. Tfh cells produce IL-21 to enhance B cell antibody production in germinal centers. Question 4 Which immunoglobulin is the principal opsonin for neutrophil phagocytosis of microbes? A. Immunoglobulin G (IgG) B. Immunoglobulin A (IgA) C. Immunoglobulin M (IgM) D. Immunoglobulin E (IgE) E. Immunoglobulin D (IgD) Answer: The correct answer is A Phagocytes engulf microbes through a process called phagocytosis. Neutrophils recognize infected and damaged cells through pattern recognition receptors, Fc receptors, and complement receptors. Phagocytosis is enhanced by host

16

molecules, called opsonins, binding to microbes and marking them for ingestion. Immunoglobulin and complement are examples of opsonins. Fc receptors and complement receptors are examples of opsonic phagocytic receptors. Fc receptors are expressed on different leukocytes and bind to the constant region (Fc) of immunoglobulin molecules. Immunoglobulin G (IgG) is the principal opsonin for antibody-­ mediated phagocytosis. IgG1 and IgG3 bind to FcγRI (CD64) expressed on phagocytes with high affinity, and the interaction promotes receptor activation and efficient phagocytosis. Other Fc receptors (FcγRII, FcγRIII) facilitate phagocytosis but have a lower affinity for IgG. Complement molecules are generated by the activation of three major pathways in response to microbes: classical, alternative, and lectin. All complement pathways lead to the production of complement fragment C3b and the formation of the membrane attack complex (MAC). C3b bound to the microbial surface is recognized by complement receptors 1 and 3 (CR1, CR3) to promote phagocytosis. Receptor activation leads to ingestion of the microbe followed by the formation of a phagosome and phagolysosome maturation. Question 5 What is the biochemical function of NADPH oxidase? A. Convert superoxide to hydrogen peroxide B. Convert molecular oxygen to superoxide C. Convert hydrogen peroxide to water and molecular oxygen D. Convert hydrogen peroxide to hypochlorous acid E. Convert molecular oxygen to hydrogen peroxide Answer: The correct answer is B Phagocytosed microbes are killed in phagolysosomes by hypohalous acids generated by a process called the respiratory burst. NADPH oxidase is a complex of six proteins that converts molecular oxygen molecules to superoxide anions. Superoxide anions are converted to hydrogen peroxidase by superoxide dismutase. Myeloperoxidase combines hydrogen peroxide and chloride anions to produce hypochlorous acid. Hypochlorous acid is, directly and indirectly, toxic to a variety of organisms, including Aspergillus species, Staphylococcus aureus, Burkholderia cepacia complex, Serratia marcescens, and Nocardia species. Patients with chronic granulomatous disease (CGD) have a defect in the NADPH oxidase complex leading to an inability to generate the respiratory burst. The most common genetic defect is a mutation in cytochrome b-245 beta subunit (CYBB) which encodes gp91phox and causes X-linked CGD. Patients with CGD have recurrent and deep-seated bacterial and fungal infections due to an inability to generate hypohalous acids. CGD is diagnosed by measuring superoxide production via the dihydrorhodamine (DHR) test. Nonfluorescent dihydrorhodamine is oxidized to fluores-

K. Paris and A. M. Abreo

cent rhodamine in normal, activated neutrophils. Patients with CGD are incapable of oxidizing dihydrorhodamine due to defective NADPH oxidase and will lack fluorescence on flow cytometry after neutrophil activation. Case 2 After a 5-day history of cough, fever, and difficulty feeding, a 7-month-old boy was admitted to the hospital with respiratory distress in January. He was previously healthy and had been meeting all developmental milestones. He was found to be positive for respiratory syncytial virus. He received supportive care and remained hospitalized for 7 days. He had a persistent requirement for supplemental oxygenation and was eventually intubated. A bronchoalveolar lavage on hospital day #9 revealed infection with Pneumocystis jirovecii and he was treated with appropriate antimicrobial therapy. Figure 3 is a representative image of a lung biopsy in another patient with Pneumocystis jirovecii pneumonia. His newborn screening assay for T cell lymphopenia was normal. There was no known family history of immune deficiency; however, his maternal grandmother’s male sibling was ill during infancy and ultimately died prior to 1 year of age. Quantitative immunoglobulins were performed which showed marked hypogammaglobulinemia of IgG and IgA with elevated IgM. His T and B cell enumeration assays were normal and mitogen testing revealed normal proliferative responses to PHA, ConA, and PWM. Genetic testing showed a pathogenic mutation in the gene encoding CD40L (CD154) confirming the suspected diagnosis of X-linked hyper-IgM syndrome. Question 1 Which cytokine is a major growth factor for dendritic cells? A. B. C. D. E.

FLT3LG T-bet GATA3 RORγt PAX5

Answer: The correct answer is A Dendritic cells are found in the blood, epithelium, and tissue compartments. They are bone marrow-derived and develop under the influence of fms-related tyrosine kinase 3 ligand (FLT3LG). Dendritic cells are divided into two major subsets: classical (conventional) and plasmacytoid. Classical dendritic cells are the most efficient professional antigen-­ presenting cell (APC) due to their ability to constantly sample the environment and migrate to the lymph nodes to present to CD4+ T cells. Other professional APCs include macrophages and B cells due to their ability to capture and present antigen in the context of class 2 major histocompatibility complex (MHC). Dendritic cells recognize microbial

Immune Cells and Functions

17

Fig. 3  Medium power view of lung biopsy, silver stain highlights the alveoli filled with fungus Pneumocystis jirovecii (GMS, final magnification 100×). Image courtesy of Matthew Stark, MD. Children’s Hospital New Orleans, LA

antigens using a variety of cell surface and cytoplasmic pattern recognition receptors. Plasmacytoid dendritic cells are primarily found in the blood. They are a major source of type I interferons (IFN-α, IFN-β) in response to viral infections. The development of CD4+ effector T cell subsets is driven by other transcription factors: Th1 (T-bet), Th2 (GATA3), and Th17 (RORγt). PAX5 is a transcription factor involved in B cell differentiation. Question 2 Which interaction increases the expression of CD80 and CD86 on antigen-presenting cells? A. PD-L1 and PD-1 B. CD86 and CTLA-4 C. ICOS-L and ICOS D. CD40L and CD40 E. OX40L and OX40

Answer: The correct answer is D Dendritic cells, macrophages, and B cells are professional antigen-presenting cells. Professional APCs express class 2 MHC molecules and can present to CD4+ T cells. T cell activation by APCs requires not only recognition of the antigen-­MHC complex but also costimulation. The interaction between CD28 on T cells and CD80 and CD86 on activated APCs is a critical costimulatory pathway. Expression of CD80 and CD86 by APCs is dependent on the activation of the APC by other molecules. CD40 ligand (CD40L, CD154) is expressed on T cells after recognition of the antigen-MHC complex. CD40L engages CD40 on APCs to enhance the expression of CD80 and CD86. Increased expression of CD80 and CD86 augments costimulation, T cell activation, and T cell proliferation. PD-1 and CTLA-4 are inhibitory receptors on T cells. ICOS is a T cell receptor required for the development of T follicular helper cells (Tfh). OX40 is a T cell receptor that promotes T cell sur-

18

K. Paris and A. M. Abreo

vival and polarization of the immune response to a Th2 phenotype. Question 3 Which cytokine is the primary activator of macrophages? A. Interleukin 2 (IL-2) B. Interleukin 10 (IL-10) C. Interleukin 12 (IL-12) D. Interferon-β (IFN-β) E. Interferon-γ (IFN-γ) Answer: The correct answer is E. Macrophages are a type of phagocyte derived from blood monocytes. Monocytes are recruited to tissue compartments during infection or injury. The primary function of macrophages is to engulf and destroy microbes. Macrophage function is enhanced by interferon-γ (IFN-γ). IFN-γ is a type II interferon as opposed to type I interferons (IFN-α, IFN-β) involved in anti-viral defense. IFN-γ is primarily produced by CD4+ T helper 1 cells (Th1), type 1 innate lymphoid cells (ILC1), NK cells, and CD8+ T cells. IL-12 stimulates Th1 cells to secrete IFN-γ which signals through the STAT1 transcription factor in macrophages. Macrophage activation also requires stimulation from CD40 on the macrophage interacting with CD40L on activated Th1 cells. The combination of IFN-γ and CD40-CD40L signaling enhances the efficiency of microbe killing in the phagolysosome. CD40-CD40L signaling also increases the expression of costimulatory molecules on macrophages to enhance T cell activation and proliferation. Activated classical macrophages also synthesize and secrete cytokines involved in the acute inflammatory response to microbes. Question 4 What is the macrophages?

function

of

alternatively

activated

A. Secretion of IL-1 and TNF-α B. Phagocytosis and killing of microbes C. Tissue repair and fibrosis D. Antigen presentation to T cells E. Removal of apoptotic cells Answer: The correct answer is C Alternatively activated macrophages are a subset involved in tissue repair and suppression of inflammation. Alternative activation is driven by IL-4 and IL-13, whereas IFN-γ leads to classically activated macrophages. IL-4 and IL-13 are Th2

cytokines primarily produced by Th2 cells and ILC2. Alternative macrophages produce IL-10 and TGF-β. IL-10 inhibits the antigen-presenting function of dendritic cells and macrophages and prevents them from secreting IL-12, a potent stimulator of IFN-γ production by Th1 cells. The lack of IFN-γ inhibits the development of classically activated macrophages and impairs the function of macrophages in cell-mediated immunity. TGF-β inhibits the production of classical macrophages but also drives tissue repair and fibrosis by enhancing fibroblast proliferation and collagen synthesis. Classically activated macrophages secrete pro-inflammatory cytokines, present antigen to T cells in the context of MHC, phagocytose and kill microbes, and participate in efferocytosis to remove apoptotic cells. Question 5 Which of the following is not a result of CD40-CD40L signaling? A. Macrophage activation B. Inhibition of T cell activation C. Class switch recombination D. Germinal center reaction E. Increased expression of CD80/CD86 Answer: The correct answer is B T cell activation increases the expression of CD40L on the T cell surface. CD40 is expressed on macrophages, dendritic cells, and B cells. CD40-CD40L signaling enhances the APC function by increasing the expression of costimulatory molecules. The interaction promotes the killing function of macrophages in combination with IFNγ. CD40 on B cells interact with CD40L on T follicular helper cells (Tfh). CD40 signaling on B cells not only stimulates B cell proliferation but leads to the development of the extrafollicular and germinal center response. CD40mediated expression of activation-­induced cytidine deaminase enables class switch recombination from IgM to IgG, IgA, or IgE. Abnormalities in CD40-CD40L signaling are illustrated by X-linked hyper-­ IgM syndrome (HIGM). HIGM is caused by a mutation in CD40L (CD154). Patients have decreased levels of IgG and IgA due to absent class switching. Immune response to intracellular microbes is defective due to an abnormal interaction between T cells and professional APCs, and many patients suffer from severe Pneumocystis jirovecii infection. CD40-­ CD40L is not involved in T cell inhibition. The best described inhibitory T cell receptors are PD-1 and CTLA-4.

Immune Cells and Functions

Bibliography 1. Abbas AK, Lichtman AH, Pillai S. Cellular and molecular immunology. 10th ed. Philadelphia, PA: Elsevier; 2022. 2. de la Morena MT. Clinical phenotypes of hyper-IgM syndromes. J Allergy Clin Immunol Pract. 2016;4(6):1023–36.

19 3. Lehman HK, Segal BH. The role of neutrophils in host defense and disease. J Allergy Clin Immunol. 2020;145(6):1535–44. 4. Thomsen IP, Smith MA, Holland SM, Creech CB.  A comprehensive approach to the management of children and adults with chronic granulomatous disease. J Allergy Clin Immunol Pract. 2016;4(6):1082–8.

HistocompatIbility Antigens (HLA) and Transplantation Michell M. Lozano Chinga, David Buchbinder, and Jolan E. Walter

1 Introduction In this chapter, we will use a case-based approach to review topics pertinent to the use of hematopoietic stem cell transplantation (HSCT) in the setting of inborn errors of immunity (IEI). IEI are disorders that result from pathogenic variants of genes involved in the immune response which place the host at increased risk of infections, autoimmunity, hyperinflammation, and other manifestations of immune dysregulation. We will review the most common indications for HSCT in IEI as well as important considerations including donor selection and conditioning which affect the outcomes of patients with IEI undergoing HSCT. We will also discuss the complications associated with HSCT including infectious and noninfectious complications such as graft-­ versus-­host disease (GVHD), graft failure, and autoimmunity among others. Finally, we will discuss late effects and survivorship in this unique population. Despite the improvement in outcomes, complications could occur during transplant as well as in the short-term and long-term post-HSCT

M. M. Lozano Chinga (*) Division of Immunology, Rheumatology and Allergy, University of Iowa Hospitals and Clinics, Iowa City, IA, USA e-mail: [email protected] D. Buchbinder Division of Hematology, Children’s Hospital of Orange County, Orange, CA, USA Department of Pediatrics, University of California at Irvine, Orange, CA, USA e-mail: [email protected] J. E. Walter Division of Allergy and Immunology, Department of Pediatrics, Morsani College of Medicine, University of South Florida, Tampa, FL, USA Division of Allergy/Immunology, Department of Pediatrics, Johns Hopkins All Children’s Hospital, St. Petersburg, FL, USA Division of Allergy and Immunology, Massachusetts General Hospital for Children, Boston, MA, USA e-mail: [email protected]

period. Comprehensive evaluation includes screening for complications (toxicities), subsequent chronic health conditions, and immune reconstitution along with careful attention to psychosocial health and well-being. The evaluation for HSCT and surveillance posttransplant is best achieved by a collaborative relationship between immunologists and transplant specialists. Case 1 An 11-day-old Caucasian male was born at 39 weeks of gestation to a nulliparous 26-year-old mother with unremarkable prenatal history and ongoing breastfeeding. The family history was noncontributory and there was no reported consanguinity. The neonate was discharged home on the day of life two. His newborn screen was presumptive positive for SCID with undetectable T cell receptor excision circles (TRECs). The mother was instructed to hold breastfeeding and store breastmilk until maternal cytomegalovirus (CMV) status is clear. The neonate was brought to the Immunology outpatient clinic for further clinical and laboratory evaluation. Complete blood count with differential documented absolute lymphopenia: 1700 cells/mm3 (normal range 2000–11,000 cells/ mm3). Flow cytometry evidenced severe T cell lymphopenia (CD3+  =  5 cells/mm3, CD3+CD4+  =  2 cells/mm3, CD3+CD8+ = 5 cells/mm3) with poor thymic production and full absence of naive of T cells (CD3+ CD45RA+); however, normal number of B cells (995 cell/mm3) and natural killer (NK) cells (175 cell/mm3). The patient was admitted due to high suspicion for SCID and treatment with prophylactic antimicrobials including acyclovir, fluconazole, and atovaquone was initiated immediately. The following day, second tier testing confirmed normal IgG (912  mg/dL, presumably maternal), low IgM (18  mg/dL), and undetectable IgA (1 in million) and only selected cases are transplanted for severe disease.

1.1.1 Indications for Hematopoietic Stem Cell Transplant in Inborn Errors of Immunity The first successful HSCTs for IEI were reported in one patient with SCID and one with WAS.  These events took place in 1968, 1 year after the characterization of the human major histocompatibility complex (MHC). Since that time, the understanding of IEI and HSCT has greatly improved. HSCT has been proven to be curative for multiple genetic causes of IEI including adaptive immune defects (e.g., SCID, DOCK8 deficiency, DOCK2 deficiency, IPEX [immune dysregulation, polyendocrinopathy, enteropathy, X-linked], CD40 ligand deficiency, CD40 deficiency, X-linked lymphoproliferative disease, activated PI3K delta syndrome, familial hemophagocytic lymphohistiocytosis, GATA2 deficiency, MHC class II deficiency, autosomal dominant IgE syndrome, CTLA4 haploinsufficiency, LRBA deficiency), and innate immune defects (e.g., CGD, RAB27A deficiency, reticular dysgenesis, and LAD type I). Table 1 lists all IEI, based on the International Union of Immunological Societies (IUIS) criteria, where HSCT has been used. The heterogeneity of IEI is enormous and it is vital to understand that HSCT is not indicated for the treatment of all IEI, especially antibody deficiency syndromes where immunoglobulin replacement therapy is sufficient to maintain

HistocompatIbility Antigens (HLA) and Transplantation Table 1  Inborn errors of immunity reported with treatment approach of HSCT IUIS I. Immunodeficiencies affecting cellular and humoral immunity IL2RG deficiency, XLa ADA deficiency, ARa AK2 deficiency (reticular IL-7Ra deficiency, AR dysgenesis), AR IL-21 deficiency, AR B2M (MHC class I deficiency), IL-21R deficiency, AR AR ITK deficiency, AR BCL10 deficiency, AR JAK3 deficiency, AR CARD11 deficiency, AR LCK deficiency, AR CD27 deficiency, AR LRBA deficiency, AR CD3d, AR MAGT1 deficiency, XL CD3e, AR MALT1 deficiency, AR CD3g, AR MST1 deficiency, AR CD3z deficiency, AR NIK deficiency, AR CD8 deficiency, AR Omenn syndrome, AR CD40 deficiency, AR OX40 deficiency, AR CD40 ligand deficiency, XL RAG1 deficiency, ARa CD45 deficiency, AR RAG2 deficiency, AR Cernunnos/XLF deficiency, AR RFX5 (MHC class II CIITA (MHC class II deficiency), deficiency), AR AR RFXAP (MHC class II Coronin-1A deficiency, AR deficiency), AR CTPS1 deficiency, AR RFXANK (MHC class II DCLRE1C deficiency (Artemis), deficiency), AR ARa RhoH deficiency, AR TAP1 (MHC class I deficiency), DNA ligase IV deficiency, AR AR DNA PKcs deficiency, AR TAP2 (MHC class I deficiency), DOCK2 deficiency, AR AR DOCK8 deficiency, AR TAPBP (MHC class I ICOS deficiency, AR deficiency), AR IKBKB deficiency, AR TCRa deficiency, AR ZAP-70 deficiency, AR IUIS II. Combined immunodeficiencies with associated syndromic feature RMRP deficiency (cartilage hair ATM deficiency (ataxia-­ hypoplasia, AR) telangiectasia-­AT), AR RECQL3 deficiency (bloom Complete DiGeorge syndromeb syndrome, AR)c Dyskeratosis congenita (TERC, TINF2, AD)c STAT3 deficiency (AD-HIES) Dyskeratosis congenita (NOLA2, STAT5b deficiency, AR NOLA3, DCLRE1B, PARN, AR)c STIM-1 deficiency, AR TBX1 deficiency, HSb Dyskeratosis congenita (TERT, RTEL1, TPP1, AD/AR)c TTC7A deficiency (ID with multiple intestinal atresias), AR Dyskeratosis congenita (DKC1, XL)c WAS deficiency (Wiskott-­ NBS1 deficiency (Nijmegen Aldrich syndrome), XLa breakage syndrome), ARd NEMO/IKBKG deficiency (ectodermal dysplasia with ID/ EDA-ID, XL) ORAI-I deficiency, AR IUIS III. Predominantly antibody deficiencies NFKB2 deficiency (AD) BTK deficiency, XLe PI3KR1 deficiency (AD, LOF) CARD11 deficiency (AD, GOF) PI3KR1 deficiency (AR) Common variable PI3K-d activated (AD, GOF) immunodeficiency (CVID), AR

23 Table 1 (continued) IUIS IV. Diseases of immune dysregulation RNASEH2A deficiency ADA2 deficiency, AR (Aicardi-Goutieres syndrome, ADAR1 deficiency (Aicardi-­ Goutieres syndrome, AGS6), AD AGS4), AR RNASEH2B deficiency CTLA4 deficiency, AD (Aicardi-Goutieres syndrome, IL-10 deficiency, AR AGS2), AR FOXP3 deficiency (IPEX RNASEH2C deficiency syndrome, XL) STXBP2 deficiency (FHL5), AR/ (Aicardi-Goutieres syndrome, AGS3), AR AD PRKCd deficiency, AR SAMHD1 deficiency (Aicardi-­ Goutieres syndrome, AGS5), AR SH2D1A deficiency (XLP1) STAT3 deficiency (AD, GOF) XIAP deficiency (XLP2) IUIS V. congenital defects of phagocyte number, function, or both CGD, AR (p22phox deficiency) GATA-2 deficiency, AD CGD, AR (p40phox deficiency) ITGB2 deficiency (leukocyte CGD, AR (p47phox deficiency) adhesion deficiency type 1, CGD, AR (p67phox deficiency) LAD1), ARa CGD, XL (gp91 phox deficiency)a Rac 2 deficiency, AD HAX1 deficiency (Kostmann disease, SCN3), AR SBDS deficiency (Shwachman-­ diamond syndrome), AR SLC35C1 deficiency (leukocyte adhesion deficiency type 2, LAD2), AR WAS GOF (X-linked neutropenia/myelodysplasia) IUIS VI. Defects in intrinsic and innate immunity CXCR4 gain-of-function (WHIM IL12/IL23RB1 deficiency (MSMD, AR) syndrome, AD) IFNγR1 deficiency (MSMD, AD) IL-12p40 (MSMD, AR) IFNγR1 deficiency (MSMD, AR) STAT1 deficiency (MSMD, AD) IFNγR2 deficiency (MSMD, AR) STAT 1 deficiency (predisposition to severe viral infection, AR) STAT 1 GOF (chronic mucocutaneous candidiasis/ CMC, AD) In gray the most common defects are highlighted a  Gene therapy available b  Thymus transplant is preferred c  HSCT is mainly indicated with bone marrow failure d  HSCT is mainly indicated in case of cancer e  HSCT in the developing world

patient infection-free and with good quality of life. Here we describe the most common IEI for which patients receive HSCT. Severe Combined Immunodeficiency SCID encompasses a group of heterogeneous genetic disorders that have in common profound T cell impairment

24

s­ econdary to absent or diminished T cells or T cell function. Clinical manifestations are usually present early in life, after the loss of maternal antibodies. Features include failure to thrive, recurrent infections by common and opportunistic pathogens as well as infections after live-virus vaccines (e.g., Bacille Calmette-Guerin [BCG]) and/or signs of inflammatory or graft-versus-host disease due to expansion of oligoclonal lymphocytes (Omenn syndrome) or the engraftment of maternal T cells. If not treated there is almost 100% mortality during the first year of life, unless the patient spontaneously develop revertant T cell lines that partially correct the underlying severe immunodeficiency. The first successful HSCT was reported in 1968, over 50 years ago; however, the treatment was not available in a timely manner for most infants for decades. The importance of early HSCT was established in the early 2000s and summarized in a milestone paper by Dr. Rebecca Buckey in 2011. This study described the long-term outcomes of 166 infants with SCID that received a nonconditioned related donor HSCT in a single institution and compared it to HSCT done at other institutions where chemotherapy as conditioning prior HSCT was used. The study concluded that if a HSCT from a relative or matched unrelated control could be done in the first few months of life (90%) in comparison with older patients. The best transplant approach was examined by Sung-Yun Pai et al. based on a 25 multicenter study among 240 infants with SCID patients in 2014. This study concluded that the survival rate was high regardless of donor type among infants who received transplants at 3.5 months of age or younger and among older infants without prior infection or with an infection that had resolved. The only exception is adenosine deaminase (ADA) deficiency that may evolve slower. In patients with ADA deficiency SCID, enzyme replacement therapy can be used as a bridge during a prolonged period until the conditions/requirements for the best definitive therapy are available. The most important risk factor for poor outcome in patients with SCID undergoing HSCT is an active infection. The quality of CD3+ T cell recovery, but not survival, is affected by the type of conditioning and genetic subtype of SCID. In fact, the genetic, phenotypic, and functional diversity of SCID adds further complexity to the best approach. Wiskott-Aldrich Syndrome WAS is an X-linked disorder with an incidence of 1  in 100,000 live male births. This disorder is caused by hemizygous mutations in the WAS gene and characterized by microthrombocytopenia, eczema, and immunodeficiency affecting B and T cells. Patients with WAS are also at increased risk of autoimmunity and malignancy. Management of WAS

M. M. Lozano Chinga et al.

includes immunoglobulin supplementation, prevention and timely treatment of infections, and control of bleeding and autoimmune complications. These supportive care efforts have increased the survival of these patients; however, successful HSCT is the only definitive treatment and corrects immunodeficiency and thrombocytopenia. A scoring system was implemented to describe the severity of the phenotype of patients with WAS gene variations. This scoring system is based on clinical and laboratory findings including thrombocytopenia, small platelets, eczema, immunodeficiency, infections, autoimmunity, malignancy, congenital neutropenia, and myelodysplasia. A score of 0 is given to patients with X-linked neutropenia and 700,000 cord blood units available worldwide. Given the unique ability of cord blood to be used successfully despite greater HLA disparity, single or double HLA mismatches can be considered. Importantly, cord blood unit donors are therefore available for over 80% of the possible recipients of any ethnic group. Cord blood is collected at the time of birth, after which it is processed and cryopreserved in the cord blood bank. The time of cryopreservation has shown no effect on post-thaw cell viability and neutrophil engraftment probability. Benefits associated with the use of UCB as a potential donor source include immediate availability and lack of risks to the donor. On the other hand, important barriers to the use of umbilical cord blood must be acknowledged. The limited cell dose often precludes the use of cord blood in many older children and adults. The use of two cord blood units and ex  vivo expansion of cord blood stem cells can increase the number of available stem cells allowing adults to be suitable recipients. High costs compared to the acquisition of a haploidentical donor are notable. Cord blood units typically approximate 100  mL or less and will yield a CD34+ cell count of 0.2  ×  106 CD34+ cells per kg of recipient body weight and a CD3+ cell count of 2.5 × 106 CD3+ per kg of recipient body weight. Regarding GVHD risk, cord blood as a graft source is associated with the lowest risk of GVHD in comparison to other graft sources.

M. M. Lozano Chinga et al.

kg), total body irradiation (TBI) (300  cGy), and alemtuzumab and GVHD prophylaxis consisting of sirolimus. He underwent a 10/10 HLA-matched unrelated donor PBSC transplant. The patient had neutrophil engraftment (first of three consecutive days of an absolute neutrophil count >500 cells/mm3) on Day +19. At day +100 posttransplant, lineage-­ specific (myeloid, CD3, and NK) chimerism was 100%, 80%, and 95%, respectively. Posttransplant complications included Grade I GVHD involving the skin which was successfully treated with corticosteroids and Klebsiella pneumoniae bacteremia treated with meropenem. Six months posttransplant the patient developed cervical lymphadenopathy and hepatosplenomegaly. A biopsy confirmed Epstein-­ Barr virus (EBV)-associated monomorphic PTLD treated successfully with rituximab. Question 1 What type of conditioning regimen is associated with excellent outcomes and stable mixed myeloid chimerism in patients with CGD? A. Not conditioning B. Reduced intensity conditioning C. Myeloablative conditioning D. Antibody-based conditioning

Answer and Explanation Answer: B Consideration of the choice of conditioning is important to the success of HSCT in IEI. Success equates to the establishment of adequate donor-derived T cell immunity (naïve T cell production), and B cell immunity (no need for immunoglobulin supplementation, and specific antibody production) in case of adaptive immune defects, and myeloid lineage for innate IEI. The purpose of conditioning with myeloablative Case 2 agents such as alkylating agents (e.g., busulfan, treosulfan) is A 23-year-old male with diagnosis of X-linked chronic gran- to “make space” in the bone marrow niche to allow engraftulomatous disease (pathogenic truncating nonsense muta- ment of donor HSCs. Conditioning with immunosuppressive tion: CYBB c.469C  >  T; protein mutation: p.Arg157X) is agents (e.g., fludarabine, cyclophosphamide) serves to being evaluated post-HSCT. The patient was diagnosed with ensure ablation of the host immune system to prevent rejecCGD at 6 months of age after presenting with a liver abscess. tion of the donor HSCs. In some forms of IEI, there is an During childhood, the patient had recurrent painful oral inability to reject the donor graft, such as in SCID, making ulcers and later in life developed recurrent perianal abscesses conditioning less needed with subsequent lower risk of morand fistulas. Colonoscopy revealed granulomatous duodeni- bidity and mortality. However, pitfalls of unconditioned tis and pancolitis. The patient was receiving prophylactic HSCT procedures include incomplete immune reconstitutrimethoprim-sulfamethoxazole, posaconazole, and terbin- tion and waning immunity. Novel approaches to conditionafine for prior history of fungal pneumonia with chronic pul- ing are being developed to limit toxicity (e.g., JSP191, monary infiltrates. A decision was made to avoid the use of humanized monoclonal anti-CD117 c-kit in clinical trials, biologic therapies and steroids due to concerns of worsening which clears hematopoietic stem cells targeted) and the immunosuppression. The patient did not have an active increased risk for chronic health conditions or so-called late infection at the time of HSCT and received a non-­ effects while supporting robust T and B cell immunity among myeloablative conditioning regimen with busulfan (5  mg/ survivors of HSCT for IEI.

29

HistocompatIbility Antigens (HLA) and Transplantation

1.2.7 Conditioning Intensity and Donor Chimerism

associated with the destruction of myeloid cells and opening of niches for HSC engraftment. These regimens also possess the capacity to destroy lymphoid cells leading to immunoabDonor Chimerism lation as well. MAC regimens that are utilized in allogeneic The establishment of adequate donor chimerism is impacted HCT for IEI include cyclophosphamide in combination with by the intensity of conditioning and remains a critical ele- alkylating agents such as busulfan or treosulfan. MAC regiment to the success of HSCT in IEI. Donor chimerism refers mens allow for donor T cells to engraft and are typically to a state which exists after engraftment in which cell popu- associated with the establishment of complete donor chimelations (myeloid, T cells, B cells) may be derived from dif- rism. This is due to complete or near complete clearance of ferent individuals. The measurement of donor chimerism recipient T cells facilitating complete donor T cell chimerelies on differences in short tandem repeat (STR) of the rism. Reduced intensity conditioning (RIC) regimens are DNA of the donor versus the recipient (patient). This can be associated with diminished toxicity and have allowed for the assessed at lymphocyte subset level. extension of HSCT to older patients with IEI or patients with Complete donor chimerism is associated with the absence significant comorbid conditions in association with their of recipient cell populations whereas mixed donor chime- IEI. RIC regimens are also associated with a greater risk of rism is associated with the presence of both recipient and mixed chimerism and graft failure or graft dysfunction foldonor cell populations. Mixed myeloid chimerism and mixed lowing HSCT. RIC regimens that are frequently utilized in lymphoid chimerism have different implications. Cells allogeneic HSCT include fludarabine in combination with derived from the myeloid lineage (mainly neutrophils) have alkylating agents such as busulfan or melphalan. Low dose a short lifespan and require bone marrow production; there- TBI is also utilized in RIC regimens (e.g., 2  Gy TBI) for fore, they are reflective of engraftment of HSCs and associ- IEI. Conditioning regimens may also be non-myeloablative ated donor chimerism of the HSCs. Cells derived from the (NMA) and are associated with minimal toxicity. There is no lymphoid lineage (mainly T cells) have a longer lifespan and need for stem cell support for autologous recovery. Large are not reflective of the engraftment of HSCs, but stable numbers of donor T cells and HSCs as well as adequate donor T cell chimerism is necessary for adequate engraft- immunoablation are required for the success of these regiment of HSCs. mens to ensure donor engraftment. NMA conditioning regiIn SCID, there is a selective advantage for the donor-­ mens may include fludarabine in combination with low dose derived T cells to survive and proliferate in the periphery. If TBI or fludarabine in combination with cyclophosphamide full B cell chimerism is also achieved, the patient may not as examples. The use of non-myeloablative conditioning is need immunoglobulin replacement therapy. However, in also associated with the presence of mixed chimerism. most circumstances only mixed chimerism is achieved, and thus patients require long-term monitoring for changes in Examples of Disease-Specific Considerations their disease phenotype. On the contrary, in patients with SCID is an IEI in which unconditioned HSCT may be conCGD, the correction of myeloid compartment is of high sidered; however, outcomes are variable. OS is excellent importance. Even low neutrophil chimerism with residual (>80%) in the setting of unconditioned matched sibling NADPH activity as low as 10% may be sufficient to prevent donor HSCT for severe combined immunodeficiency; howclinical disease. Therefore, data on donor chimerism should ever, unconditioned HSCT with unrelated donors or cord be analyzed in light of the underlying IEI. blood may be less favorable. Although classically, many HSCT procedures for SCID were unconditioned, a greater Conditioning Intensity interest in the use of conditioning has occurred in recent Table 3 provides an overview of aspects of conditioning in decades as limited engraftment of donor HSC which fail to HCT for IEI. Myeloablative conditioning (MAC) regimens support adequate T and B cell immunity long-term. include a variety of agents or combinations of agents that are Conditioning prior to HSCT for SCID is associated with Table 3  Aspects of conditioning in inborn errors of immunity Intensity Non-myeloablative

Myeloablation NA

Reduced intensity

+

Myeloablative

++

Examples Fludarabine (150–160 mg/m2) + Total body irradiation (2 Gy) Fludarabine (150–160 mg/m2) + Cyclophosphamide (20–40 mg/kg) Fludarabine (150–160 mg/m2) + Melphalan (140 mg/m2) Fludarabine (150–160 mg/m2) + Busulfan (AUC 60–70 mg*h/L) Fludarabine (150–160 mg/m2) + Busulfan (AUC 85–95 mg*h/L) Busulfan (AUC 85–95 mg*h/L) + Cyclophosphamide (120–200 mg/kg)

Abbreviations: AUC area under the curve, Gy gray, NA not applicable

30

M. M. Lozano Chinga et al.

improved T cell and B cell immunity, and a diminished need for a second HSCT procedure. This is particularly salient for specific forms of SCID with a tendency for inflammation (e.g., RAG deficiency). As anticipated, conditioning leads to a greater risk for tissue injury and complications (e.g., acute GVHD and late effects). Specific forms of SCID with DNA repair defects require special consideration (e.g., ARTEMIS deficiency) due to sensitivity to alkylating agents and greater risk for devastating late effects (e.g., growth failure, dental issues) with conditioning. Similar to SCID, conditioning intensity may be tailored to each specific IEI or category of IEI and recipient, incorporating other factors such as donor and stem cell source. As an example, WAS is an IEI that requires greater consideration of MAC regimens. RIC regimen use in WAS is associated with mixed myeloid and lymphoid chimerism and risk for persistent thrombocytopenia and autoimmunity, respectively. CGD is an IEI that requires greater consideration of RIC regimens. RIC regimen use in CGD is associated with excellent outcomes (OS >90%) and stable mixed myeloid chimerism (>90%) which is curative for patients with CGD including those with intractable and often invasive infections and inflammatory complications. Concluding Thoughts and Future Directions Ongoing studies are attempting to tailor conditioning exposures to ensure adequate T cell and B cell engraftment while limiting the risk for late effects. Conditioning SCID infants diagnosed early (C-SIDE) represents the first randomized trial to assess if lower doses of busulfan can support robust establishment of T and B cell immunity while mitigating toxicity and late effects in newborns with SCID. Antibody-­ based conditioning is also under investigation. Recent clinic data demonstrates that anti-CD117 (c-Kit) antibody is able to clear the niches of HSCs facilitating donor HSC engraftment without toxicity in patients with SCID.  These approaches and others are currently being evaluated to optimize the health and well-being of patients with IEI. Question 2 What organs/system is not included in the evaluation of acute graft-versus-host disease? A. B. C. D.

Skin Liver Blood Gut

Answer and Explanation Answer: C GVHD occurs frequently following allogeneic HSCT for IEI and offers no benefit for patients with IEI as it may in patients with selected malignant disorders. An absence of

GVHD is associated with robust immune reconstitution. On the other hand, treatment of GVHD with immunosuppressive agents is associated with impaired immune reconstitution, infectious risk, and an increased risk of morbidity and mortality. A variety of factors shape the risk of GVHD such as PBSCs as a graft source (greater numbers of T cells), MAC conditioning, and greater HLA disparity. DLI used to boost antiviral immunity or engraftment following HSCT for IEI is associated with a greater risk for GVHD. Other risk factors for GVHD specific to IEI include the presence of transplacental maternal engraftment often detected in SCID patients or early mixed donor chimerism with a gradual increase in donor T cell chimerism which may be protective. Classically, acute GVHD was described by the presence of signs and symptoms of GVHD occurring prior to Day +100 post-HCT and chronic GVHD after Day +100 post-­ HCT.  Currently, it is recognized that acute GVHD and its manifestations may occur after Day +100. Moreover, acute GVHD can be classified as persistent, recurrent, or de-novo. There may also be a so-called overlap syndrome in which features of both acute and chronic GVHD coexist. Table 4 provides a brief description of pharmacologic and non-pharmacologic approaches frequently utilized for GVHD prophylaxis as well as their mechanisms and toxicities. Prevention of GVHD is paramount and requires the use of pharmacologic agents which interfere with T cell activation, T cell function, as well as T cell proliferation. Decreasing the number of T cells using T cell depletion techniques including both in vivo and ex vivo approaches are also frequently utilized. Pathophysiology of Acute and Chronic GVHD

Tissue damage which may occur in relation to the underlying IEI, and its treatment including conditioning regimens (even when RIC or NMA regimens are utilized) supports the development of GVHD. Tissue damage directly results in the elaboration of proinflammatory cytokines such as tumor necrosis factor (TNF)-alpha and IL-1. The elaboration of proinflammatory cytokines also occurs indirectly. For example, damage to the gut results in the translocation of microbial products such as lipopolysaccharide (LPS) which activates host antigen-presenting cells including monocytes and macrophages leading to the elaboration of proinflammatory cytokines. LPS may also lead to the augmentation of the inflammatory response through activation of the innate immune system (e.g., Toll-Like Receptors [TLRs], Nod-Like Receptors [NLRs]). Donor T cells are then activated by this inflammatory milieu. Target cell death ensues via a variety of mechanisms (e.g., FAS-mediated apoptosis, etc.) in association with the destruction of epithelial cells in specific target organs (e.g., skin, liver, gut). Chronic GVHD also involves a breakdown in central and peripheral tolerance to self-antigens with resultant autoimmunity. This occurs in an analogous fashion to the break-

HistocompatIbility Antigens (HLA) and Transplantation

31

Table 4  Aspects of graft-versus-host disease prophylaxis Category Calcineurin inhibitors

Examples Cyclosporine tacrolimus

Antimetabolites

Methotrexate

Mycophenolate mofetil

mTOR inhibitors Sirolimus In vivo T cell depletion

Antithymocyte globulin Alemtuzumab

Posttransplant cyclophosphamide

Ex vivo T cell depletion

TCR alpha/beta + CD19 depletion

Mechanism of action Bind immunophilins which interact with calcineurin, a calmodulin-dependent phosphatase involved in the activation of nuclear factor of activated T cells (NFAT) proteins. The NFAT proteins translocate to the nucleus and are associated with the transcription of various cytokine genes that are vital for T cell proliferation, activation, and function such as interleukin (IL)-2 Methotrexate is an anti-metabolite which inhibits dihydrofolate reductase (DHFR). DHFR reduces folic acid to tetrahydrofolate (THF) which is necessary for thymidylate and purine synthesis A prodrug of mycophenolic acid, an inhibitor of inosine 5′ monophosphate dehydrogenase which results in depletion of guanosine nucleotides. This is associated with the inhibition of T cell and B cell proliferation Bind FKBP12 and complexes with mTOR resulting in the inhibition of IL-2 mediated T cell proliferation T cell depletion in blood and peripheral lymphoid tissue via complement-dependent lysis and T cell activation associated apoptosis Anti-CD52 which results in T cell and B cell depletion in the peripheral blood and peripheral lymphoid tissues via complement-dependent lysis, antibody-dependent cytotoxicity, and direct apoptosis Depletion of host T cells which proliferate in response to donor antigens and the depletion of donor T cells which proliferate in response to host antigens. Specific subsets of T cells are spared which are important for viral immunity and immune reconstitution Depletion of alloreactive T and B cells which are implicated in the pathogenesis of GVHD and Epstein-Barr virus (EBV)related disease while ensuring that T cells that support engraftment and surveillance against malignant cells and viral pathogens remain

Toxicities Increased risk of infection, hypertension, nephrotoxicity, encephalopathy, and microangiopathy

Nausea, vomiting, myelosuppression, hepatotoxicity, nephrotoxicity, and infection risk Gastrointestinal toxicity including nausea, vomiting, and myelosuppression, and infection risk Risk of infection, hyperlipidemia, and microangiopathy Risk of infection, infusion reactions, flu-like illness, serum sickness Risk of infection, infusion reactions, flu-like illness

Risk of infection, nausea, vomiting, and hemorrhagic cystitis

Increased risk of graft rejection, delayed immune reconstitution, and risk of infectious complications

Table 5  Aspects of acute graft-versus-host disease staging and grading Organ Skin (rash)a Liver (bilirubin)b Gut (diarrhea)c

Stage

Grade Skin Liver Gut

1 500 mL/day I 1 or 2 – –

2 25–50% BSA 4–6 mg/dL > 1000 ml/day II 3 or 1 or 1

3 >50% BSA 7–15 mg/dL > 1500 ml/day III – 2–3 or 2–4

4 Desquamation and/or bullae > 15 mg/dL Severe abdominal pain and/or ileus IV 4 or 4 –

 Exclude other causes: drug reaction, viral exanthem, engraftment syndrome, chemotherapy-related, radiation-related  Exclude other causes: sinusoidal obstruction syndrome, viral infection, drug toxicity, sepsis c  Exclude other causes: chemotherapy-related, drug-related, infection a

b

down of central and peripheral tolerance which occurs in selected IEI due to defects in thymic stroma or HSC leading to disordered thymopoiesis and diminished thymic output. Tissue injury from GVHD or conditioning exposures extends to thymic epithelial cells which is associated with a breakdown in central tolerance as the cortical thymic epithelial cells and medullary thymic epithelial cells are essential to the process of positive and negative selection. During positive and negative selection, thymocytes whose T cell receptors do not recognize self-antigen or thymocytes that recognize self-antigen intensively in the context of peptides

presented by MHC are deleted. Regulatory T cells which are vital to the maintenance of peripheral tolerance may also become impaired in chronic GVHD.  B cell dysregulation may also contribute to chronic GVHD as developing B cells also engage with antigens in the bone marrow with a process similar to the positive and negative selection of T cells by which the deletion of autoreactive B cells occurs. Staging and Grading of Acute and Chronic GVHD

Table 5 provides a brief overview of the organ staging and grading of acute GVHD as well as potential diagnoses which

M. M. Lozano Chinga et al.

32

have substantial clinical overlap with the features of acute GVHD. Organ staging of acute GVHD is based on criteria by Glucksberg. This staging requires the assignment of a stage for each organ system involved including skin, liver, and gut. Stages are assigned for each organ using a score of 0–4. The Seattle grading system is then used which allows for an overall grade to be assigned from 0 to 4 using the organ-specific staging results. The diagnosis of acute GVHD is challenging due to clinical diagnoses which may present with features that overlap with acute GVHD.  Serum biomarkers are being utilized to a greater extent in the diagnosis of acute GVHD and its prognosis. As an example, markers of intestinal crypt damage including regenerating islet derived 3 alpha (REG3alpha) as well as Suppressor of Tumorigenesis 2 (ST2) have been shown to be predictive of mortality in association with acute GVHD. Features of chronic GVHD including organ-specific manifestations as well as a brief overview of the staging and grading of chronic GVHD are depicted in Table 6. The most frequently involved organs including the skin, mouth, eye, gut, and liver are those mentioned above; however, joints, muscles, genitalia, esophagus, nails, and lungs may also be involved in the setting of chronic GVHD. The diagnosis of chronic GVHD requires the confirmation of diagnostic (e.g., lichen planus or lichen sclerosis) and distinctive manifestations (e.g., papulosquamous lesions). Signs and symptoms are utilized to define organ-specific scores from 0 to 3 (no involvement or symptoms to severe functional compromise). A global severity score is assigned (mild, moderate, severe) based on the number of organs involved and their organ-­ specific scores. Serum biomarkers are also under intense investigation as tools in the diagnosis of chronic GVHD and its prognosis.

Table 6  Aspects of chronic graft-versus-host disease assessmenta Evaluation History

Eyes

Mouth

Sinus Gastrointestinal

Musculoskeletal

Genitals Lungs Exams Physical exam

Organ system Skin

Mouth

Musculoskeletal

Laboratory

Adjunctive Overall assessment

Treatment of Acute and Chronic GVHD

Acute and chronic GVHD requires prompt therapy. The first-­ line agent which is used for the treatment of acute and chronic GVHD is corticosteroids. For acute GVHD, standard therapy includes methylprednisolone 2  mg/kg/day divided into two or three equal doses followed by a short taper. Steroid refractory acute GVHD is diagnosed if there is a lack of clinical improvement or worsening in the signs or symptoms of acute GVHD after 3–5  days of treatment. Second-line therapy is considered if a diagnosis of steroid-­ dependent or refractory GVHD is present. Examples of second-­line agents utilized in acute GVHD are depicted in Table 7. For chronic GVHD, standard therapy includes methylprednisolone 1  mg/kg/day followed by a prolonged taper. Steroid refractory chronic GVHD is often considered if there is a worsening in the signs or symptoms of chronic GVHD after 1–2 weeks of treatment or a lack of improvement after

Organ System Skin

Gastrointestinal Hematologic Lungs Global severity Mild Moderate

Severe

Symptoms Tight, raw, sores, itchy, rash, shiny scars, scaly, flaky, dark or light spots, hair thin or loss, nail changes Dry, use of artificial tears, gritty, excess tearing, discomfort, trouble opening, impacts daily activity Sores, ulcers, sensitivity to temperature/toothpaste/spicy foods/ drinks, dry mouth, trouble opening mouth, issues impact daily intake Drainage, congestion, pain, sinus infection, surgery Nausea, vomiting, diarrhea, appetite changes, weight changes, and swallowing difficulties impacts daily intake Tightness, tightness muscle cramps, muscle weakness, swollen joints, limits daily activity Penile/vaginal dryness/discomfort Short of breath, need for supplemental oxygen, cough or wheezing Assessment Extent of cGVHD within skin surface surveying for diagnostic or distinctive signs of cGVHD and quantify extent of erythema, dyspigmentation, and sclerosis Extent of cGVHD within oral cavity surface surveying for erythema, lichenoid lesions, ulcers, mucoceles Extent of cGVHD-associated sclerosis or fasciitis by surveying for diminished joint range of motion AST, ALT, alkaline phosphatase, bilirubin Platelet count Forced expiratory volume (1 s) Definition 1–2 organs involved, severity = mild (excluding lung) 3 or 3+ organs involved, severity = mild to moderate (lung only mild) 3 or 3+ organs involved, severity = severe (lung moderate to severe)

 Adapted from Carpenter PA. How I conduct a comprehensive chronic graft-versus-host disease assessment. Blood 2011 a

1–2 months of treatment. Second-line therapy is considered if a diagnosis of steroid-dependent or refractory chronic GVHD is present. A variety of approaches now exist for the treatment of chronic GVHD. Various examples of agents are depicted in Table 8. Importantly, patients with acute or chronic GVHD also require aggressive methods of infection prophylaxis.

HistocompatIbility Antigens (HLA) and Transplantation

33

Table 7  Examples of second-line acute graft-versus-host disease treatment Anti-cytokine antibodies

Examples Basiliximab Infliximab

Antimetabolites

Methotrexate

Mycophenolate mofetil

JAK inhibitor

Ruxolitinib

T cell depletion

Antithymocyte globulin Alemtuzumab

Other

Extracorporeal photopheresis

Mechanism of action Monoclonal antibody directed at interleukin (IL)-2 receptor preventing formation of the IL-2 binding site abrogating activation of T cells Monoclonal antibody directed at tumor necrosis factor (TNF) alpha which play an important role in the development of GVHD An anti-metabolite which inhibits dihydrofolate reductase (DHFR). DHFR reduces folic acid to tetrahydrofolate (THF) which is necessary for thymidylate and purine synthesis

Toxicities Risk of infection and infusion reactions Risk of infection and infusion reactions Nausea, vomiting, myelosuppression, hepatoxicity, nephrotoxicity, and infection risk Nausea, vomiting, A prodrug of mycophenolic acid, a inhibitor of inosine 5′ monophosphate dehydrogenase which results in depletion of guanosine myelosuppression, and infection risk nucleotides. This is associated with the inhibition of T cell and B cell proliferation A selective Janus kinase (JAK) 1/2 inhibitor with a impact on interferon Risk of infection, cytopenias (IFN) gamma and IFN gamma receptor signaling and subsequent reductions in T cell infiltration T cell depletion in blood and peripheral lymphoid tissue via Risk of infection, infusion complement dependent lysis and T cell activation associated apoptosis. reactions, flu-like illness, serum sickness Anti-CD52 which results in T cell and B cell depletion in the peripheral Risk of infection, infusion blood and peripheral lymphoid tissues via complement-dependent lysis, reactions, flu-like illness antibody-dependent cytotoxicity, and direct apoptosis Extracorporeal exposure of peripheral blood mononuclear cells Requires vascular access, high (PBMCs) to photoactivated 8-methoxypsoralen and reinfusion of the cost, and specialized center treated cells. When exposed to ultraviolet (UV)-A light it is activated and cross-links DNA resulting in apoptosis with changes in lymphocytes, monocytes, and dendritic cells

Table 8  Examples of second line chronic graft-versus-host disease treatment Category B cell directed

Examples Rituximab Ibrutinib

Antimetabolites

Methotrexate

Mycophenolate mofetil

mTOR inhibitors Sirolimus JAK inhibitor

Ruxolitinib

Tyrosine kinase inhibitor

Imatinib

Other

Extracorporeal photopheresis

Mechanism of action Monoclonal antibody directed at CD20 on B cells leading to selective depletion of CD20+ B cells Inhibits Bruton tyrosine kinase (BTK) in B cells and Interleukin-­ 2-­inducible T cell kinase (ITK) in T cells leading to diminished B cell survival and inhibition of selected subsets of T cells; respectively An anti-metabolite which inhibits dihydrofolate reductase (DHFR). DHFR reduces folic acid to tetrahydrofolate (THF) which is necessary for thymidylate and purine synthesis A prodrug of mycophenolic acid, an inhibitor of inosine 5′ monophosphate dehydrogenase which results in depletion of guanosine nucleotides. This is associated with the inhibition of T cell and B cell proliferation Binds FK binding protein-12 and complexes with mTOR resulting in the inhibition of IL-2 mediated T cell proliferation A selective Janus kinase (JAK) 1/2 inhibitor with an impact on interferon (IFN) gamma and IFN gamma receptor signaling and subsequent reductions in T cell infiltration Inhibits both platelet-derived growth factor (PDGF) and transforming growth factor (TGF)-beta signaling intracellularly which reduces the development of fibrosis Extracorporeal exposure of peripheral blood mononuclear cells (PBMCs) to photoactivated 8-methoxypsoralen and reinfusion of the treated cells. When exposed to ultraviolet (UV)-A light it is activated and cross-links DNA resulting in apoptosis with changes in lymphocytes, monocytes, and dendritic cells

Toxicities Hypogammaglobulinemia, risk of infection, and infusion reactions Risk of infection, fatigue, nausea, vomiting, diarrhea, muscle spasms, and bruising Nausea, vomiting, myelosuppression, hepatoxicity, nephrotoxicity, and infection risk Nausea, vomiting, myelosuppression, and infection risk Risk of infection, hyperlipidemia, and microangiopathy Risk of infection, cytopenias

Risk of infection, cytopenias, fluid retention Requires vascular access, high cost, and specialized center

34

M. M. Lozano Chinga et al.

Question 3 During the post-engraftment period, what predisposes bacterial infections? A. B. C. D.

Neutropenia Hypogammaglobulinemia Mucositis Intravascular devices

Answer and Explanation Answer: B Infections which occur prior to HSCT and those that occur following HSCT are an important cause of early and late morbidity and mortality when considering HSCT for IEI.  There are multiple factors which shape the impact of infection in the setting of a specific diagnosis of an IEI including pathogens, prior treatment of IEI, time from diagnosis of the IEI to HSCT, as well as the degree of HLA disparity, donor and graft source, intensity of conditioning, and complications including GVHD. Severe Combined Immunodeficiency

SCID is one particular example of an IEI in which infections are a vitally important aspect of pre-HSCT planning and post-HSCT outcomes. Pre-HSCT infections are associated with adverse HSCT outcomes. The 5-year survival for patients with SCID post-HSCT is 80–95% when transplanted prior to the onset of infections. With the successful application of newborn screening, the number of newborns with SCID whose diagnosis was triggered by infection has continued to diminish. Despite newborn screening efforts, many newborns with SCID still experience infections prior to HSCT which can potentially cause organ damage. Judicious use of supportive care therapy including early prophylaxis with Bactrim and azoles has diminished the burden of pre-­ HSCT infections with Pneumocystis Jirovecci pneumonia (PJP) and Candida. Suppression of double-stranded DNA viruses such as cytomegalovirus (CMV) remain paramount prior to HSCT. Importantly, the use of conditioning in SCID patients with active infections is also associated with inferior OS which requires thoughtful consideration. Post-HSCT immune reconstitution is also an important consideration in the setting of SCID. As an example, for those patients that do not achieve robust B cell reconstitution, hypogammaglobulinemia and specific antibody production remain poor and the risk of severe bacterial infections remain elevated in the absence of immunoglobulin supplementation.

important cause of morbidity in patients with phagocytic defects. Herpesviruses as CMV and bacterial infections can be problematic in patients with T cell deficiencies. To reduce the risk of complications, treatments targeted at colonizing bacteria can be helpful prior to and during HSCT, for example, the use of antibiotics for patients with eczema and Staphylococcus aureus or those with significant pulmonary disease and colonizing organisms such as non-tuberculous mycobacteria. Those at high risk of CMV may benefit from prophylaxis with novel agents such as letermovir. For patients with phagocytic defects, such as CGD, patients are able to undergo HSCT with active infections including fungal infections although specific considerations may include the use of granulocyte transfusions during the period of neutropenia. Post-HSCT infections also remain of particular importance following HSCT for IEI. As an example, hypogammaglobulinemia and risk of severe bacterial infections may be abrogated with immunoglobulin supplementation. Viral reactivation with double-stranded DNA viruses may be abrogated with antiviral agents such as ganciclovir and foscarnet. For other viral infections such as Epstein-Barr virus (EBV) and associated lymphoproliferative disease post-HSCT, adoptive cellular therapies may be required such as EBV cytotoxic T lymphocytes. Bacterial Infections

Bacterial infections can appear anytime following HSCT; however, they are more common in the pre-engraftment period secondary to severe neutropenia, presence of intravascular devices, and mucositis. Infections can be caused by gram-negative and gram-positive bacteria resulting in bacteremia, pneumonia, colitis as well as central nervous system, urinary, skin, and soft tissue infections. Patients are not routinely placed on prophylactic antibiotics; however, if there is a concern for a bacterial infection the recommendation is to start broad-spectrum antibiotics following guidelines for neutropenic fever. During the post-engraftment period, bacterial infections are secondary to hypogammaglobulinemia and hyposplenism. The most common bacterial infections during this period are secondary to encapsulated bacteria as Haemophilus influenzae, Streptococcus pneumonia, and Neisseria meningitidis. Chronic GVHD can be associated with further immunosuppression in these patients with a subsequent increased risk of infections. Figure  1 shows the infection risk in relationship with the specific period after HSCT.

Other Inborn Errors of Immunity

Fungal Infections

Pre-HSCT infections are also important when considering other IEIs as many patients present to HSCT with a significant burden of infections which requires careful identification and treatment. Fungal infections, for example, are an

Fungal infections are an important cause of morbidity and mortality in patients following HSCT. The diagnosis of fungal infections can be challenging since the fungal pathogen is not always identified with a culture and diagnosis may be

HistocompatIbility Antigens (HLA) and Transplantation

35

Fig. 1  Infection risk during HSCT

Late Phase Days = or > +100 Pre-engraftment Days +30 – +100

Preengraftment Days 0 – +30

Enteric viruses/Respiratory viruses HHV-6

Viruses HSV

VZV CMV

EBV - PTLD

Aspergillus species, Toxoplasma, Candia species

Fungi

Pneumocystis jiroveci Bacteria

Gram negative and gram positve 0

suggested by symptoms, imaging, and indirect evidence of fungal infection such as a positive galactomannan or beta D-glucan assay. The risk of invasive candidiasis and mold infections is higher during the pre-engraftment period due to the associated severe neutropenia, mucositis, and central venous access devices. After engraftment, the presence of a central venous access device in conjunction with severe gastrointestinal GVHD can also be associated with a greater risk of invasive candidiasis. The risk of opportunistic mold infections such as Aspergillosis in the post-engraftment period is also related to prolonged and severe cell-mediated immunodeficiency. Aspergillus is the most common cause of fungal pneumonia in patients following HSCT. Prophylaxis against fungal infections is routinely used during the pre-­engraftment period. Azoles affect cytochrome P450 metabolism; therefore, attention must be paid to possible interactions with other medications often utilized during HSCT.  Alternative medications include echinocandins such as micafungin. The risk of PJP is approximately 5–15% among patients not receiving prophylaxis; however, effective prophylaxis decreases the risk of PJP significantly. The preferred regimen is treatment with trimethoprim-sulfamethoxazole ­(TMP-­SMX). Other medications used for PJP prophylaxis include dapsone, pentamidine, and atovaquone. Viral Infections

Viral infections are easily transmissible and thus are quite common. Following HSCT patients can acquire a variety of viral infections including respiratory and gastrointestinal

30

60

Encapsulated bacteria 90

120

120

180

viruses such as metapneumovirus, respiratory syncytial virus, parainfluenza, influenza, and adenovirus. Viral pathogens that establish latency such as CMV, EBV, varicella (VZV), BK, and huma herpesvirus 6 (HHV6) can also reactivate and can be associated with significant morbidity and mortality. Herpes virus prophylaxis with acyclovir is usually given to seropositive patients following HSCT. Periodic laboratory surveillance of certain viruses such as CMV and EBV can help detect the viruses before patients become symptomatic allowing for preemptive therapy. If a viral pathogen is identified, then antivirals and other supportive care agents are given as clinically indicated. Cytomegalovirus is one of the most common viruses causing infections in patients following HSCT.  CMV is a herpesvirus typically acquired early in life. Patients undergoing HSCT and their possible donors have their CMV IgG positivity assessed to identify CMV serostatus. If the recipient is seronegative, the preferred donor is seronegative as well. Similarly, if the recipient is positive, the preferred donor is seropositive. A high risk of CMV reactivation is seen with a negative donor and a positive recipient. Leukoreduction and CMV testing of blood products has reduced the likelihood of patients acquiring CMV through blood transfusions. CMV is monitored periodically with PCR-based techniques in patients following HSCT in order to treat the infection as soon as its diagnosed. CMV can affect multiple organs with the most common infections being pneumonitis and colitis. Ganciclovir is the first-line treatment for CMV.  Foscarnet is similarly effective; how-

36

M. M. Lozano Chinga et al.

ever, it is associated with a greater burden of side effects including nephrotoxicity. CMV-specific cytotoxic T cells have also been demonstrated to be effective in treating patients with CMV and an inadequate response to traditional therapies. Letermovir has been studied as a new prophylactic medication which has been demonstrated to be safe, well-­ tolerated, and effective. EBV is another herpes virus associated with clinically significant viral infections in patients following HSCT. HSCT recipients and possible donors undergo testing to detect IgG against EBV prior to transplantation. Active surveillance with polymerase chain reaction (PCR)-based techniques is done periodically following HSCT. Most of the EBV infections following HSCT are secondary to endogenous virus reactivation. Typically, EBV is asymptomatic or associated with self-limited illness; however, in patients following HCST it can cause posttransplant lymphoproliferative disease (PTLD). High-risk patients include patients who have received a T cell-depleted HSCT or UCB transplant. EBV-­ related PTLD results from the inability of cellular immunity to eliminate B cells that have been infected with EBV allowing for abnormal lymphoproliferation. Management of PTLD includes decreasing doses of immunosuppressants and Rituximab, a chimeric murine antibody against CD20. Newer therapies include EBV-specific cytotoxic T lympho-

cytes. Prophylactic therapy against EBV is not recommended because of lack of efficacy. Question 4 What are potential treatments for graft failure? A. Additional graft from same donor B. Donor leukocyte infusion C. Immunosuppressant medication D. A and B Answer and Explanation Answers: A and B

1.3 Noninfectious Complications HCT remains a recognized curative treatment for many patients with nonmalignant disorders including IEI. Despite this, noninfectious complications involving a myriad of organ systems remain a formidable barrier to successful HSCT. Below we will outline a few examples that are particularly pertinent to patients with IEI.  Table  9 provides a brief summary of these and other noninfectious complications which arise following HSCT for IEI.

Table 9  Examples of noninfectious complications following hematopoietic cell transplantation Category Pulmonary

Examples Idiopathic pneumonia syndrome

Diffuse alveolar hemorrhage

Bronchiolitis obliterans syndrome

Bronchiolitis obliterans organizing pneumonia/ cryptogenic organizing pneumonia

Description IPS occurs in the first 100 days (within the first 30 days) following HCT is idiopathic pneumonia syndrome (IPS). IPS results from diffuse alveolar injury manifest radiographically as multilobar infiltrates on a chest radiograph or chest CT. Clinical manifestations of IPS following HCT include cough, difficulty breathing, and rales on clinical exam DAH occurs early following HCT and within the first 30 days following HCT. DAH is manifest radiographically as diffuse patchy consolidations on chest radiograph or chest CT. Clinical manifestations of DAT following HCT include difficulty breathing, hypoxia, and hypoxemia. A cough may also be present in addition to fever; however, hemoptysis remains infrequent. Bronchoalveolar lavage fluid may be used to document blood and hemosiderin-laden macrophages which are supportive of a diagnosis of DAH Destruction of small airways of the lung and manifest radiographically by often normal chest radiographs; however, on chest CT there is air trapping, bronchiectasis, and other changes. Clinically there is dyspnea, dry cough, and wheezing. Pulmonary function testing will demonstrate an obstructive pattern. Diagnostic criteria for BOS include a decrease in the forced expiratory volume (FEV1) BOOP/COP affects the alveoli and is manifest radiographically by what appears to be pneumonia. Chest radiographs and chest CT demonstrate infiltrates, ground glass opacities, and nodular opacities. Clinical features include fever, dyspnea, and cough. Pulmonary function testing demonstrates a restrictive pattern with a diminished diffusing capacity (DLCO) and diminished total lung capacity (TLC)

Treatment Treatment of IPS includes the use of anti-tumor necrosis factor (TNF)alpha agents such as etanercept

Treatment of DAH has focused on the use of corticosteroids

Treatment of BOS includes the use of corticosteroids; however, the use of inhaled corticosteroids, macrolide antibiotics, and leukotriene antagonists is considered

Treatment of BOOP/COP has focused on the use of corticosteroids

HistocompatIbility Antigens (HLA) and Transplantation

37

Table 9 (continued) Category Hepatic

Examples Sinusoidal obstruction syndrome

Genitourinary Thrombotic microangiopathy

Hemorrhagic cystitis

Other

Graft failure

Autoimmunity

Description SOS occurs during the first 30 days following HCT although late SOS is possible. Two or more of the following features must also be present: Elevated bilirubin, hepatomegaly, ascites, and weight gain. Liver biopsy is not required to make the diagnosis of SOS. Ultrasonography can be used to document hepatomegaly and/or ascites. Alterations in portal venous flow and hepatic venous outflow may also be documented but are often considered to be a late finding of SOS

Treatment Defibrotide is used for the treatment of severe SOS. Although the exact mechanism by which defibrotide leads to diminished organ-specific compromise related to SOS is unknown, it is thought that defibrotide possesses anti-­ inflammatory as well as anticoagulant properties Transplant-associated thrombotic microangiopathy (TA-TMA) For patients who develop signs of is an endothelial damage syndrome manifest as hypertension, TA-TMA while receiving CNIs they are discontinued. Treatment with thrombocytopenia, and elevated lactate dehydrogenase. terminal complement blockade is TA-TMA results from the activation of endothelial cells to produce a pro-coagulant state, activation of antigen-presenting considered. Eculizumab is a monoclonal antibody against C5 cells and lymphocytes, and activation of the complement that prevents the formation of the cascade with microthrombi formation MAC Prophylactic approaches including Conditioning leads to damage of the urothelium and immune the use of hyper-hydration and suppression is permissive for the reactivation of viruses (e.g., MESNA are employed in high-risk BK polyomavirus) in the urothelium. This results in patients. Treatment involves the use inflammation and hemorrhage. The diagnosis of HC requires of cidofovir with or without signs and symptoms of cystitis (dysuria and frequency). probenecid. Other agents have been Hematuria is also required (microscopic or gross). Ultrasonography of the bladder is a tool to evaluate for bladder utilized as well such as Levaquin or leflunomide. Adjunctive therapies wall thickening and debris. Quantification of virus (e.g., BK have also been associated with polyomavirus) in the blood and urine is also diagnostic improvements such as intravesicular therapies (e.g., cidofovir) Graft failure can be defined as primary or secondary based on Options for treatment may include a the lack of HSC engraftment or HSC engraftment followed by so-called boost which requires an a subsequent loss of HSC engraftment. Graft failure may also additional graft from the same be defined by a lack of T cell engraftment manifest as a lack of donor, often CD34+ selected and CD3+ T cells or absence of donor T cell chimerism and/or the administered without conditioning. Donor lymphocyte infusion (DLI) lack of HSC engraftment manifest as an absolute neutrophil count (ANC)  500/μL without evidence can also be utilized to stabilize mixed chimerism that is falling or to of donor myeloid chimerism. Graft failure may also be convert a state of mixed chimerism considered following HCT for IEI when persistent immune to full donor chimerism. Graft dysfunction occurs typically in association with recurrent or failure can also be treated with severe infections or autoimmunity following HCT. Graft failure may also manifest as inadequate hematopoiesis despite retransplantation clinical and laboratory evidence of HSC engraftment Organ-specific manifestations of autoimmunity following HCT Treatment typically include corticosteroids, high dose are diverse. One of the most common and frequent immunoglobulin, rituximab, and manifestation is an autoimmune cytopenias. Other organspecific manifestations are infrequent, but include the thyroid combinations of agents such as (e.g., hypothyroidism), central and peripheral nervous systems sirolimus, bortezomib, (e.g., myasthenia gravis), integument (e.g., vitiligo), liver (e.g., mycophenolate mofetil, splenectomy, and in refractory case hepatitis), and kidney as examples retransplantation

1.3.1 Graft Failure Graft failure is a rare complication following HSCT for IEI.  Graft failure can be defined as primary or secondary based on the lack of HSC engraftment or HSC engraftment followed by a subsequent loss of HSC engraftment. Graft failure may also be defined by a lack of T cell engraftment manifest as a lack of CD3+ T cells or absence of donor T cell chimerism and/or the lack of HSC engraftment manifest as an absolute neutrophil count (ANC)   500/μL without evidence of donor myeloid chime-

rism. The interval of time elapsed following HCT required to define graft failure is a subject of debate. It is important to consider various clinical factors and HCT-related factors (e.g., specific IEI diagnosis, degree of HLA disparity, conditioning exposures, T cell depletion, stem cell source) when approaching the diagnosis of graft failure as well as the potential etiologic agents such as infections (e.g., CMV, HHV6) and medications (e.g., Bactrim, Ganciclovir). Graft failure may also be considered following HCT for IEI when persistent immune dysfunction occurs typically in associa-

38

tion with recurrent or severe infections or autoimmunity following HCT. Graft failure may also be manifest as inadequate hematopoiesis despite clinical and laboratory evidence of HSC engraftment. Options for treatment may include a so-called boost which requires an additional graft from the same donor, often CD34+ selected and administered without conditioning. DLI can also be utilized to stabilize mixed chimerism that is falling or to convert a state of mixed chimerism to full donor chimerism; however, the risk of GVHD in the setting of IEI must be considered. Graft failure can also be treated with retransplantation. Retransplantation may occur using the same donor; however, the procurement of the second source of HSCs can be considered. More importantly, the health of the recipient must also be considered adequate as conditioning is typically utilized. The choice of conditioning, donor, and graft source in the setting of retransplantation for graft failure is controversial. Successful HSC engraftment has been documented in association with long-term OS following retransplantation for patients with IEI.

1.3.2 Pulmonary Complications Patients with IEI are particularly susceptible to pulmonary complications prior to HCT.  Recurrent upper and lower respiratory traction infections are frequently observed leading to issues such as bronchiectasis. Other pulmonary complications are also frequently observed such interstitial lung disease. In IEI, pulmonary disease serves as a risk factor for morbidity (e.g., a need for respiratory support) following HCT as well as mortality following HCT. Importantly, pulmonary risk factors should be carefully assessed prior to HCT using pulmonary function tests, high-resolution computed tomography of the chest, bronchoalveolar lavage, and biopsy if indicated. Defining the presence of interstitial lung disease or pulmonary infection (e.g., aspergillus, atypical mycobacteria, etc.) is important to ensure optimal control prior to HCT and can also assist with the tailoring of the HCT procedure. Noninfectious pulmonary complications which occur most commonly in the early period (the first 100 days following HCT) are idiopathic pneumonia syndrome (IPS) and diffuse alveolar hemorrhage (DAH). IPS typically occurs early following HCT and typically within the first 30 days following HCT.  IPS results from diffuse alveolar injury which is manifest radiographically as multilobar infiltrates on a chest radiograph or chest CT. Clinical manifestations of IPS following HCT may include cough, difficulty breathing, and rales on clinical exam. Treatment of IPS has focused on the use of anti-tumor necrosis factor (TNF)-alpha agents such as etanercept. DAH also occurs early following HCT and typically within the first 30 days following HCT. DAH is manifest radiographically as diffuse patchy consolidations

M. M. Lozano Chinga et al.

on chest radiograph or chest CT. Clinical manifestations of DAT following HCT may include difficulty breathing, hypoxia, and hypoxemia. A cough may also be present in addition to fever; however, hemoptysis remains infrequent. Blood and hemosiderin-laden macrophages on a ­bronchoalveolar lavage are typically documented. Treatment of DAH has focused on the use of corticosteroids. Pulmonary complications which occur most commonly in the late period (following the first 100 days following HCT) are bronchiolitis obliterans syndrome (BOS), and bronchiolitis obliterans organizing pneumonia/cryptogenic organizing pneumonia (BOOP/COP). BOS is associated with the destruction of small airways of the lung. BOS is manifest radiographically by often normal appearing chest radiographs; however, on chest CT there is often evidence of air trapping as well as bronchiectasis, and other findings. BOS is manifest clinically by dyspnea, dry cough, and wheezing as well as an absence of fever. Pulmonary function testing will typically demonstrate an obstructive pattern. Diagnostic criteria for BOS include a decrease in the forced expiratory volume (FEV1) by 20% with a FEV1 to forced vital capacity (FVC) ratio of 90%. This evolution has supported to identify if the patient has need of ongoing immunoglobuan increasing number of HSCT survivors with a prior diag- lin replacement as well as to provide prophylactic medicanosis of IEI worldwide. Survivors of HSCT for IEI can be tions. Immune reconstitution should be evaluated impacted by late-occurring chronic health conditions or so-­ periodically, especially if there are concerns about recurrent called late effects which can occur following HSCT. infections. Routinely, patients are followed closely after In pediatric patients, continuous monitoring of height and HSCT. Depending on the preferences of the center, follow- weight allows to identify patients who may require nutri­up, as well as surveillance for potential late effects and man- tional and endocrinological evaluation. Developmental agement of late effects, may be carried out by the HSCT screening should also be done when patients are able to do so

M. M. Lozano Chinga et al.

40 Table 10  Specific organ/system evaluations post-HSCT Organ/system Endocrine

Associated late effects Growth retardation/obesity

Hypothyroidism

Gonadal insufficiency

Diabetes/dyslipidemia

Osteoporosis/avascular necrosis

Ophthalmologic

Impaired vision, cataracts, xerophthalmia

Audition

Hearing loss, tinnitus, vertigo

Dental

Teeth development

Neurocognitive

Developmental delay

Cardiovascular

Coronary artery disease, heart failure, arrhythmias, peripheral arterial disease, cerebrovascular disease

Pulmonary

Chronic cough, interstitial lung disease, chronic GVHD

Renal

Hypertension, chronic kidney disease, tubular injury

Gastrointestinal

Liver dysfunction, strictures, liver GVHD

Skin

Skin GVHD (poikiloderma, ichthyosis, eczema, keratosis pilaris) Increase risk of infections (depending on stage and use of immunosuppressant for GVHD)

Immunologic

Psychosocial

Decrease quality of life, social withdrawal, underemployment, chronic fatigue, depression, anxiety

Evaluation/management Height, weight, BMI, Tanner staging every 6 months Bone age Referral to endocrinology if clinically indicated Physical examination of thyroid, TSH, and free T4 Referral to endocrinology if clinically indicated Annual assessment of pubertal and reproductive function Males at risk: Morning testosterone, semen analysis Females at risk: LH, FSH, and estradiol. Gynecologic evaluation if at risk for vaginal fibrosis Referral to endocrinology if clinically indicated Fasting blood glucose of hemoglobin A1C and lipid profile every 2 years Referral to endocrinology if clinically indicated DEXA scan 1 year after transplant and as clinically indicated MRI if suspicious of avascular necrosis Recommendations about regular physical activity and adequate intake of calcium and vitamin D Referral to endocrinology if clinically indicated Visual acuity and fundoscopic examinations every 1–3 years by ophthalmology Audiology evaluation 1 year after transplant and as clinically indicated Dental hygiene education Oral assessments every 6–12 months Neuropsychological evaluation including processing speed, attention, visual motor integration, memory, comprehension, verbal fluency, executive function and planning Annual history regarding cardiac symptoms (dyspnea, chest pain, palpitations, syncope) Echocardiogram done based on personal risk Referral to cardiology if clinically indicated Pulmonary function tests yearly Imaging considered depending on clinical assessment Counseling about smoking and second-hand smoking Blood pressure with every medical visit Annual urinalyses, blood urea nitrogen, creatinine, and electrolytes 1 year after transplant and as clinically indicated Referral to nephrology if clinically indicated ALT, AST, bilirubin, serum ferritin 1 year posttransplant and as clinically indicated Counseling in limiting alcohol use Annual skin check Sun protection Immunizations CMV monitoring in patients at risk of CMV reactivation Psychosocial assessment including access to healthcare and insurance

HistocompatIbility Antigens (HLA) and Transplantation

to facilitate therapies if needed. Neurocognitive deficits can be related to the IEI itself as mentioned above or to conditioning, for example, the use of TBI, which is avoided in younger children. Any patient who received alkylators as part of their conditioning may be at increased risk of hormonal imbalances such as hypothyroidism and hypogonadism with subsequent infertility. In eligible patients, egg and sperm preservation can be offered prior to HSCT. Pulmonary dysfunction can be common, especially in patients with prior recurrent respiratory infections. Additionally, GVHD and inadequate immunoglobulin replacement can cause further lung damage. All patients should be assessed for smoking history including second-­ hand smoke. Pulmonary function tests are recommended yearly for patients after HSCT.  Imaging such as chest CT imaging is indicated depending on the need for additional clinical assessment. Finally, because children and adults may have significant psychosocial impairments resulting from their own disease, chronic illness, prolonged hospitalizations, and social isolation, their physical, emotional, and social health as well as their overall quality of life can be assessed through standardized surveys and measurement tools. This will allow for the identification of survivors who may benefit from specific supportive care therapies. Evaluations can be repeated as needed across the trajectory of survivorship care.

Bibliography 1. Castagnoli R, Delmonte OM, Calzoni E, Notarangelo LD. Hematopoietic stem cell transplantation in primary immunodeficiency diseases: current status and future perspectives. Front Pediatr. 2019;7:295. https://doi.org/10.3389/fped.2019.00295. 2. Buckley RH, Schiff SE, Schiff RI, et al. Hematopoietic stem-cell transplantation for the treatment of severe combined immunodeficiency. New Engl J Med. 1999;340(7):508–16. https://doi. org/10.1056/nejm199902183400703. 3. Burroughs LM, Petrovic A, Brazauskas R, et al. Excellent outcomes following hematopoietic cell transplantation for Wiskott-Aldrich syndrome: a PIDTC report. Blood. 2020;135(23):2094–105. https://doi.org/10.1182/blood.2019002939. 4. Chiesa R, Wang J, Blok HJ, et  al. Hematopoietic cell transplantation in chronic granulomatous disease: a study of 712 children and adults. Blood. 2020;136(10):1201–11. https://doi.org/10.1182/ blood.2020005590. 5. Copelan EA.  Hematopoietic stem-cell transplantation. New Engl J Med. 2006;354(17):1813–26. https://doi.org/10.1056/ nejmra052638. 6. Wachsmuth LP, Patterson MT, Eckhaus MA, Venzon DJ, Gress RE, Kanakry CG. Post-transplantation cyclophosphamide prevents graft-versus-host disease by inducing alloreactive T cell dysfunction and suppression. J Clin Invest. 2019;129(6):2357–73. https:// doi.org/10.1172/jci124218. 7. Abdelhakim H, Abdel-Azim H, Saad A. Role of αβ T cell depletion in prevention of graft versus host disease. Biomed. 2017;5(3):35. https://doi.org/10.3390/biomedicines5030035.

41 8. Champlin RE, Schmitz N, Horowitz MM, et al. Blood stem cells compared with bone marrow as a source of hematopoietic cells for allogeneic transplantation. IBMTR histocompatibility and stem cell sources working committee and the European Group for Blood and Marrow Transplantation (EBMT). Blood. 2000;95(12):3702–9. 9. Rocha V, Cornish J, Sievers EL, et  al. Comparison of outcomes of unrelated bone marrow and umbilical cord blood transplants in children with acute leukemia. Blood. 2001;97(10):2962–71. https:// doi.org/10.1182/blood.v97.10.2962. 10. Shaw P, Shizuru J, Hoenig M, Veys P, IEWP-EBMT. Conditioning perspectives for primary immunodeficiency stem cell transplants. Front Pediatr. 2019;7:434. https://doi.org/10.3389/ fped.2019.00434. 11. Heimall J, Puck J, Buckley R, et  al. Current knowledge and priorities for future research in late effects after hematopoietic stem cell transplantation (HCT) for severe combined immunodeficiency patients: a consensus statement from the second pediatric blood and marrow transplant consortium international conference on late effects after pediatric HCT.  Biol Blood Marrow Transplant. 2017;23(3):379–87. https://doi.org/10.1016/j.bbmt.2016.12.619. 12. Dvorak CC, Hassan A, Slatter MA, et al. Comparison of outcomes of hematopoietic stem cell transplantation without chemotherapy conditioning by using matched sibling and unrelated donors for treatment of severe combined immunodeficiency. J Allergy Clin Immun. 2014;134(4):935–943.e15. https://doi.org/10.1016/j. jaci.2014.06.021. 13. Thakar M, Logan B, Buckley R, et  al. Transplantation outcomes for children with severe combined immune deficiency (SCID) have improved over time: a 36-year summary report by the primary immune deficiency treatment consortium (PIDTC). Biol Blood Marrow Transplant. 2020;26(3):S18–9. https://doi.org/10.1016/j. bbmt.2019.12.083. 14. Schuetz C, Neven B, Dvorak CC, et al. SCID patients with ARTEMIS vs RAG deficiencies following HCT: increased risk of late toxicity in ARTEMIS-deficient SCID.  Blood. 2014;123(2):281–9. https:// doi.org/10.1182/blood-­2013-­01-­476432. 15. Gooptu M, Antin JH.  GVHD prophylaxis 2020. Front Immunol. 2021;12:605726. https://doi.org/10.3389/fimmu.2021.605726. 16. Zeiser R, Blazar BR.  Pathophysiology of chronic graft-­ versus-­ host disease and therapeutic targets. New Engl J Med. 2017;377(26):2565–79. https://doi.org/10.1056/nejmra1703472. 17. Flinn AM, Gennery AR. Treatment of pediatric acute graft-versus-­ host disease—lessons from primary immunodeficiency? Front Immunol. 2017;8:328. https://doi.org/10.3389/fimmu.2017.00328. 18. Servais S, Lengline E, Porcher R, et  al. Long-term immune reconstitution and infection burden after mismatched hematopoietic stem cell transplantation. Biol Blood Marrow Transplant. 2014;20(4):507–17. https://doi.org/10.1016/j.bbmt.2014.01.001. 19. Satwani P, Baldinger L, Freedman J, et al. Incidence of viral and fungal infections following busulfan-based reduced-intensity versus myeloablative conditioning in pediatric allogeneic stem cell transplantation recipients. Biol Blood Marrow Transplant. 2009;15(12):1587–95. https://doi.org/10.1016/j.bbmt.2009.08.006. 20. Girmenia C, Bertaina A, Piciocchi A, et al. Incidence, risk factors and outcome of pre-engraftment gram-negative bacteremia after allogeneic and autologous hematopoietic stem cell transplantation: an Italian prospective multicenter survey. Clin Infect Dis. 2017;65(11):1884–96. https://doi.org/10.1093/cid/cix690. 21. Tomblyn M, Chiller T, Einsele H, et  al. Guidelines for preventing infectious complications among hematopoietic cell transplantation recipients: a global perspective. Biol Blood Marrow Transplant. 2009;15(10):1143–238. https://doi.org/10.1016/j. bbmt.2009.06.019. 22. Ozdemir ZN, Bozdag SC. Graft failure after allogeneic hematopoietic stem cell transplantation. Transfus Apher Sci. 2018;57(2):163– 7. https://doi.org/10.1016/j.transci.2018.04.014.

42 23. Smith CC, Lakin P, Lehmann LE, Pai SY, Whangbo J. Pulmonary complications in patients with primary immunodeficiency undergoing hematopoietic stem cell transplantation. Biol Blood Marrow Transplant. 2020;26(3):S16–7. https://doi.org/10.1016/j. bbmt.2019.12.081. 24. Buxbaum NP, Pavletic SZ.  Autoimmunity following allogeneic hematopoietic stem cell transplantation. Front Immunol. 2020;11:2017. https://doi.org/10.3389/fimmu.2020.02017.

M. M. Lozano Chinga et al. 25. Chow EJ, Anderson L, Baker KS, et  al. Late effects surveillance recommendations among survivors of childhood hematopoietic cell transplantation: a children’s oncology group report. Biol Blood Marrow Transplant. 2016;22(5):782–95. https://doi.org/10.1016/j. bbmt.2016.01.023. 26. Pai SY, Logan BR, Griffith LM, et  al. Transplantation outcomes for severe combined immunodeficiency, 2000–2009. New Engl J Medicine. 2014;371(5):434–46. https://doi.org/10.1056/ nejmoa1401177.

Allergic Diseases of the Eye Matthew R. Norris and Neeti Bhardwaj

Abbreviations AKC GPC IgE MCT MCTC OA PAC SAC VKC

Atopic keratoconjunctivitis Giant papillary conjunctivitis Immunoglobulin E Mast cell (Tryptase variant) Mast cell (Tryptase and chymase variant) Ocular allergy Perennial allergic conjunctivitis Seasonal allergic conjunctivitis Vernal keratoconjunctivitis

1 Introduction The term ocular allergy is used to describe a heterogeneous collection of immunological inflammatory processes affecting the anterior surface of the eye that affects between 20 and 40% of the United States population. It costs Americans an estimated 2–4 billion dollars annually. Despite having a significant economic and clinical burden, less than 10% of patients with allergic conjunctivitis seek medical attention. In this chapter, we use case-based discussion to review the spectrum of ocular hypersensitivity disorders, which includes ocular nonallergic hypersensitivity disorders (such as giant papillary conjunctivitis) and ocular allergic

M. R. Norris Department of Medicine, Division of Pulmonary, Allergy and Critical Care Medicine, Pennsylvania State College of Medicine, Penn State Health-Milton S Hershey Medical Center, Hershey, PA, USA e-mail: [email protected] N. Bhardwaj (*) Department of Pediatrics, Division of Pediatric Allergy-­ Immunology, Pennsylvania State College of Medicine, Penn State Health-Milton S Hershey Medical Center, Hershey, PA, USA e-mail: [email protected]

d­ isorders (such as seasonal and perennial allergic conjunctivitis, vernal keratoconjunctivitis, and atopic keratoconjunctivitis). We will also review diagnostic criteria, key differential diagnoses, principles of evaluation, and management for each of them. We will conclude with a discussion of emerging therapies. Case 1 A 35-year-old male presents to the office in August with complaint of red, itchy eyes. His symptoms began 6 years prior to the visit. They are intermittent, beginning around May and persisting until late October, and tend to flare while mowing the lawn. Although typically responsive to oral diphenhydramine as needed in the past, his symptoms have been worsening despite use several times per day and now include the sensation of excessive tearing and a “gritty” sensation when he closes his eyes. On examination, he has conjunctival injection with peri-limbal sparing. No conjunctival papillae are noted. Aeroallergen skin testing identifies sensitization to grass mix and a tear osmolarity of 320 mOsm/L. Question 1 What is the most likely diagnosis? A. Seasonal allergic conjunctivitis B. Seasonal allergic conjunctivitis with concomitant dry eye disease C. Atopic keratoconjunctivitis D. Dry eye disease E. Perineal allergic conjunctivitis Answer and Explanation Answer: B The patient’s history and concordant skin testing support the diagnosis of seasonal allergic conjunctivitis. However, overuse of diphenhydramine has resulted in tear film instability and development of concomitant dry eye disease. This is described in greater detail in the section below on

© The Author(s), under exclusive license to Springer Nature Switzerland AG 2022 M. Mahmoudi (ed.), Absolute Allergy and Immunology Board Review, https://doi.org/10.1007/978-3-031-12867-7_4

43

44

d­ ifferential diagnoses. Furthermore, his lack of blurring of vision, photophobia, or periocular involvement suggests atopic keratoconjunctivitis.

2 Allergic Conjunctivitis: Seasonal or Perennial Both seasonal and perennial allergic conjunctivitis are inflammatory processes of the ocular surface that occur in individuals sensitized to aeroallergens. They are defined by duration of illness, with seasonal symptoms being intermittent and perennial symptoms being year-round. The seasonal form is more prevalent and accounts for the majority of cases of allergic conjunctivitis. It is triggered by pollen (grasses, trees, weeds) that surface during specific seasons. The exact species and onset of pollen season vary depending on geographical location in the United States. Tracking of regional pollen counts is often performed by allergists, who are increasingly making their data available to the public through the American Academy of Allergy, Asthma, and Immunology’s National Allergy Bureau. However, actual pollen counts are not readily available to most stakeholders as the process of counting is both laborious and not typically reimbursed. For this reason, we often rely on a pollen index (a prediction based on historical data) to see which species may be correlating with the patient’s symptoms. Perennial allergic conjunctivitis, as the name implies, is associated with perennial allergens such as animal dander, dust mite, and cockroach. These allergens are present in the environment year-round. Patients may have a mixed phenotype of allergic conjunctivitis, characterized by year-long symptoms from a perennial allergen with seasonal flares during a pollen season.

M. R. Norris and N. Bhardwaj

MCT is more commonly found in the conjunctival epithelium than MCTC and responds best to treatment with antihistamines. Basophils, eosinophils, and Th2 cells play a role in the late-phase reaction, which does not respond well to antihistamine therapy.

3 Pathogenesis

Human mast cells are categorized based on granule-­ associated neutral proteases (tryptase and chymase) which are unique to mast cells. On this basis, mast cells have been divided into MCT (tryptase) and MCTC (tryptase/chymase) phenotypes. The normal human conjunctival epithelium rarely has mast cells but when they exist, they are mainly the MCT phenotype. Individuals with seasonal and perennial allergic conjunctivitis have increased numbers of MCT mast cells and eosinophils. In addition, they may have IgE antibodies, histamine, tryptase, eotaxin, and eosinophil cationic protein in the tear film. The pathogenesis of allergic conjunctivitis has long been recognized as biphasic, with an early phase reaction and a late phase reaction. The early phase reaction is the product of IgE antibody-dependent interaction, where disruptions in epithelial cell tight junctions result in penetration by a pre-­ sensitized allergen. This exposure results in cross-linking of specific IgE molecules to the high-affinity receptors (FceRI) on mast cells (MCT predominant) and subsequent mast cell release of preformed (histamine and tryptase) and newly synthesized mediators (prostaglandin D2 and leukotrienes C4, D4, and E4) in the substantia propria. Symptoms attributable to the release of histamine only last 20–30 min; however, the cascade of the other inflammatory mediators set up the late-phase reaction through the recruitment of inflammatory cells from the bloodstream into tissue via promotion of Question 2 adhesion molecules (e.g., VCAM-1, ICAM-1/CD54, and You stop the patient’s diphenhydramine and initiate a combi- E-selectin). T cells are also recruited simultaneously to difnation of cetirizine 10 mg daily and artificial tears as needed ferentiate and secrete an inflammatory cytokine milieu up to four times daily. He returns 2 weeks later stating the (Fig. 1). The late phase reaction occurs between 4 and 24 h intensity of his symptoms has improved dramatically; how- after symptom provocation in 33–100% of patients. The ever, they are still bothersome. On examination, you observe overlapping biological effects of these mediators (both in the improved but persistent conjunctival injection with peri-­ early and late phases) contribute to the characteristic symplimbal sparing. Decreased activity of which cells is most toms of ocular allergy—namely itching, redness, and watery likely responsible for the patient’s symptomatic discharge. improvement? Question 3 A. Mast Cells (Tryptase variant) Which of the following physical exam findings is most char B. Mast Cells (Tryptase/Chymase variant) acteristic of allergic conjunctivitis? C. Basophils D. Eosinophils A. Allergic shiners E. Th2 Cells B. Peri-limbal sparing C. Horner-Trantas dots Answer and Explanation D. Hyperosmotic tear film Answer: A E. Herbert follicles

Allergic Diseases of the Eye

45

Fig. 1  Allergic Inflammation of the ocular surface. (Reproduced with permission from Valentine L, Norris MR, Bielory L.  Comparison of structural components and functional mechanisms within the skin vs.

the conjunctival surface. Curr Opin Allergy Clin Immunol. 2021;21(5): 472–479)

Answer and Explanation Answer: B In allergic conjunctivitis, the intensity of vascular engorgement decreases toward the limbus of the eye. Peri-­ limbal redness (aka: ciliary flush) occurs secondary to dilation of the radial vessels and is better characterized as iritis. Horner-Trantas dots (gelatinous papillae on the limbal conjunctiva) and Herbert follicles (superior limbal follicles) are physical exam findings more suggestive of vernal conjunctivitis. Allergic shiners, or periorbital venous congestion, are associated with allergic rhinitis rather than allergic conjunctivitis. Although hyperosmolar tear film can be seen in cases of allergic conjunctivitis, it often signifies a concomitant dry eye syndrome.

4 Clinical Features of Allergic Conjunctivitis In taking the history of suspected ocular allergy, it is most important to screen for ocular pain or blurred vision with photophobia. These are alarming if present and may merit further evaluation by an ophthalmologist for uveitis, keratitis, or glaucoma depending on the patient’s history and rest of the physical exam. Ocular discharge is not always present however when present is either watery or “stringy/ropey.” Mucoid, or purulent discharge, with morning crusting and difficulty opening the eyelids is typically seen in bacterial infection rather than SAC or PAC. Ocular pruritus and erythema are important to identify, for if neither are present

46

you should consider an alternative condition such as dry eye disease. Ocular itching is the dominant symptom reported in allergic conjunctivitis. On review of pharmaceutical studies looking at minimum allergen dose for symptom provocation, ocular pruritus occurs at significantly lower doses of allergen extract independently of the development of erythema, which occurs at much higher doses of allergen extract. Thus, you may have a patient who complains of seasonal itchy eyes without erythema depending on the nature of the allergen exposure and degree of patient’s sensitization. The complaint of “eye grittiness” is a good screening question for concomitant dry eye disease however nonspecific. Symptoms should be bilateral if allergic in etiology and will often be associated with allergic rhinitis. Thorough medication history should be obtained as many medications can exacerbate ocular allergic symptomatology, such as diphenhydramine. Onset and persistence of symptoms are important as animal exposures or seasonal variations can offer important cues as to whether the symptoms are more likely going to be seasonal or perennial in nature. For patients with perennial allergies, dust mite, animal dander, and feather are the most commonly implicated allergens in North America. Onset is further important in differentiation as infections caused by viruses and bacteria generally present in one eye first, with the second eye getting involved a few days later. Identifying whether patients use periocular cosmetics or contact lenses is also important as these can be sources of conjunctival irritation. The patients will often have concurrent complaints of rhinitis. Periorbital darkening commonly known as “Allergic Shiners” may also be seen with accumulation of blood or other fluids in the infraorbital groove. Symptoms can be objectively measured using the Total Ocular Symptom Score (TOSS), a validated metric that asks patients whether they have experienced any of the following four ocular symptoms in the past week: itchiness, redness, tearing (or eyes watering), or swelling. For each symptom they are asked whether they experienced symptoms all of the time (4 points), most of the time (3 points), half of the time (2 points), some of the time (1 point), or none of the time (0 points). The score from all four symptoms is added together, then divided by 16 and multiplied by 100 to get the average score. Calculating a TOSS score can be of assistance to physicians caring for SAC or PAC, with utility ranging from an objective evaluation of a new therapeutic’s efficacy to trending patient perception of symptom severity objectively over time. On physical examination, symptoms may or may not be present depending on the severity of the illness or time from patient’s last exposure to a suspected allergen. If conjunctival injection is present, there should be minimal erythema in the peri-limbal region of the eye. Vision, shape of the pupil, ocular movement, light reactivity, and retinal reflexes should

M. R. Norris and N. Bhardwaj

not be affected if the etiology of the patient’s complaints is secondary to allergic conjunctivitis. Question 4 Which of the following tests, if positive, offers the least support for the diagnosis of allergic conjunctivitis? A. Measurement of total IgE in tears B. Measurement of tear osmolarity C. Conjunctival provocation testing D. Conjunctival cytodiagnosis E. Skin prick testing Answer and Explanation Answer: B Hyperosmolarity only identifies the presence of secondary dry eye disease rather than supporting the diagnosis of allergic conjunctivitis. Presence of IgE in tears, conjunctival provocation testing, skin prick testing, and conjunctival cytodiagnosis would all support the presence of allergic conjunctivitis if positive.

5 Diagnosing Allergic Conjunctivitis Objective testing should be performed to confirm the suspected aeroallergen as identification allows for providing guidance for allergen avoidance. Allergy offices are equipped with the ability to perform skin prick testing, which is a rapid and simple procedure that provides evidence of specific external environmental allergen sensitivity. Although sensitive for allergic rhinitis and allergic asthma, skin testing may not always correlate with ocular surface allergen sensitization. In these cases, serum-specific IgE measurements should be considered when SPT results are discordant with the medical history. Other tests that can be performed but are not usually available to allergists in the clinic include measurement of total IgE in tears (which should be negative), tear osmolarity (if hyperosmotic suggestive of dry eye disease), Schirmer testing (abnormal if ≤5 mm wetting after 5 min without anesthesia, ≤3 mm with anesthesia), and conjunctival provocation testing (mainly used in drug studies). Additionally, conjunctival cytodiagnosis can be performed by an ocular specialist or ophthalmologist to better elucidate harder to diagnose ocular pathology as even the presence of one eosinophil is highly suggestive of atopic pathology. Additional discussion regarding diagnostics for ocular allergy is mentioned later in this chapter. Question 5 As you discuss his primary diagnosis of seasonal allergic conjunctivitis, he recollects that for the previous 3 years he

Allergic Diseases of the Eye

has observed symptoms are milder and take longer to ­provoke in early June than in late August. What is the most likely explanation for his observation? A. Sensitization B. Tachyphylaxis to diphenhydramine C. Secondary allergen sensitization D. Priming effect E. Variations in pollen allergenicity Answer and Explanation Answer: D The priming effect is a phenomenon by which repeated exposure to allergen results in reversible increased reactivity to the allergen. Sensitization is the process by which one becomes allergic to a particular substance. Tachyphylaxis is defined as rapidly diminishing response to successive doses of a drug. There is a widespread belief in the medical community that taking long-term antihistamines makes them less effective; to date, there is no literature to support this. Skin testing only identified monosensitization to grass pollen mix, reducing the likelihood of secondary allergen-provoking symptoms. Pollen allergenicity variation may explain differences in symptom provocation on a day-by-day basis but would not explain a general trend throughout a pollen season that resets each year.

6 Additional Factors Affecting Allergic Conjunctivitis One key factor to understand when managing allergic conditions is the priming effect. The priming effect describes the phenomenon where chronic allergen exposure results in reversible increased reactivity to a given allergen. Although the mechanism has not been fully elucidated, it is believed that chronic stimulation of the inflammatory cascade results in increased immune cell migration to the conjunctival surface. The increased number of immune cells provides additional sites for IgE-allergen interaction, a greater quantity of preformed mediators, and greater poststimulation capacity for producing cytokines. This is important as symptom severity scores linearly rise until a threshold pollen level is reached, at which point symptom scores increase nonlinearly, and eventually level off. The inflection point where the linear increase becomes nonlinear has been hypothesized to be the point where the receptors responsible for the IgE-­ mediated reaction saturate. A similar relationship between symptom scores and medication scores occurs throughout a pollen season. These findings support the theory that the predominance of symptoms secondary to the late phase reaction is responsible for increased symptom scores and decreased medication efficacy as pollen counts rise.

47

Although sensitization to an aeroallergen such as pollen requires a combination of environmental exposure and genetic predisposition to develop allergies, the rate of pollen sensitization is often confounded by variations in regional geography, variations in CO2 and ozone levels, seasonal and annual differences in temperature, and pollen immunogenicity (which itself can vary from plant to plant as well as varies from a single plant over the course of its pollen season). With the boom of urbanization in the past half century, allergenic plants such as ragweed are thriving near humans in areas not traditionally thought to be problematic for the given plant. A key concern to the allergist treating allergic conjunctivitis is global warming, as warmer temperatures and higher carbon dioxide levels cause plants to grow more vigorously, create more pollen, and create more allergen per pollen grain than otherwise expected. Additionally, pollen seasons are getting longer as temperatures remain warmer. With longer pollen seasons characterized by higher pollen counts of increasing allergenicity, we can predict SAC will continue to have increasing clinical and pharmacoeconomic impact. Case 2 An 11-year-old male, who recently migrated from Algeria is referred to you by his pediatrician. His father reports that the child has been experiencing eye symptoms, especially itching and burning for the past 2 years with progressive worsening. He notes that the symptoms are more pronounced during spring. The itchiness is persistent. He experiences runny nose and watery eyes during spring. Upon exam, you notice moderate injection of conjunctiva bilaterally, with white ropy discharge in the lower fornices. Skin test is negative for seasonal and perennial allergens. Question 1 What is the most likely diagnosis? A. Seasonal allergic conjunctivitis B. Atopic keratoconjunctivitis C. Vernal keratoconjucntivitis D. Blepharoconjunctivitis E. Viral conjunctivitis Answer and Explanation Answer: C The correct answer is C (vernal keratoconjunctivitis). This condition is seen primarily in prepubescent males, in dry climates, which fits the description of this patient. Moreover, mucoid ropey discharge is also a classic finding in this condition. Choice A is not correct as this patient experiences pruritus year-round. The nature of ocular discharge in seasonal allergic conjunctivitis is typically clear and watery, and these patients usually report nasal symptoms of allergy as well. Choice B is incorrect as this patient is not e­ xperiencing symp-

M. R. Norris and N. Bhardwaj

48

toms involving the eyelids or periocular skin and does not have concomitant eczema. Choice D is incorrect as blepharoconjunctivitis is characterized by inflammation of the eyelid margins, with the accumulation of crust worse in the morning. These symptoms have not been described for this patient. Choice E is incorrect because viral conjunctivitis presents with clear discharge, and is an acute condition, contrary to the chronic symptoms reported by this patient.

7 Vernal Keratoconjunctivitis Vernal keratoconjunctivitis (VKC) is a chronic inflammatory disorder of the conjunctiva. The onset is generally before age 10 years. The duration is typically 2–10 years, with resolution in late puberty. Although VKC is considered rare in most developed countries (with an estimated mean annual incidence of 1.24 per 10,000 persons in the United States), the prevalence is significantly higher in warm/dry subtropical countries of Africa, the Middle East, Latin America, and Asia. While it affects males predominantly at younger ages, gender distribution equalizes in older age patients. Other atopic manifestations can be seen in 40–75% of patients with VKC, with 40–60% of patients having a history of atopy. Characteristic is a marked seasonal flaring of symptoms, frequently being in the spring (hence the term vernal). Question 2 Which of the following therapies would have the least likely chance of offering symptomatic relief to the patient? A. Allergen immunotherapy B. Use of refrigerated artificial tears C. Use of topical corticosteroids D. Use of oral steroids E. Use of topical antihistamines Answer and Explanation Answer: A Not all patients with VKC have positive skin tests, nor do their flares follow a particular environmental aeroallergen stimulus. Refrigerated artificial tears would serve to lubricate the ocular surface and offer temporary analgesic relief. Topical/oral corticosteroids and topical antihistamines can potentially be therapeutic in the treatment of VKC.

8 Pathogenesis The pathogenesis of VKC is controversial and has yet to be fully elucidated. It has historically been considered an allergic condition as there is evidence of mast cell (MCTC),

Fig. 2  Horner-Trantas dots. Reproduced with permission from Bielory L. Allergic and immunologic disorders of the eye. Part II: ocular allergy. J Allergy Clin Immunol. 2000 Dec;106(6):1019–32

eosinophil, Th2, and IgE involvement. The tarsal conjunctival epithelium and the substantia propria have been shown to have large numbers of mast cells (mainly MCTC) than individuals without VKC. T cells enhancing IgE and IL-4 production have been found in both layers, and major basic protein has been found throughout the conjunctival epithelium. Allergen-specific antibodies, as well as mast cell mediators, have been demonstrated in the tears of VKC patients. However, not all patients have positive skin tests or sensitization to a particular allergen. Genetic influence has been established by observations of increased prevalence of atopy in affected families as well as in studies of monozygotic versus dizygotic twins. In addition to the increasing recognition of genetic factors is the identification of possible neuroendocrine-­immune interaction that may further be contributing to a non-IgE-mediated presentation of VKC. Question 3 The focal white dots in Fig.  2 are primarily aggregates of which of the following? A. B. C. D. E.

T lymphocytes B lymphocytes Monocytes Neutrophils Eosinophils

Answer and Explanation Answer: E Horner-Trantas dots are aggregates of degenerated epithelial cells and eosinophils accumulating on a gelatinous, hyperplastic limbus, and are commonly seen in VKC and AKC.

Allergic Diseases of the Eye

9 Clinical Features and Diagnosis There are three clinical forms: (1) the palpebral form with giant papillae on the tarsal conjunctiva; (2) The limbal form with Horner-Tranta dots (clumps of necrotic eosinophils and epithelial cells); (3) a mixed form of both the palpebral and limbal forms. The limbal and mixed forms are more common in central and southern Africans whereas palpebral VKC is most frequent in Europe and America. VKC is characterized by severe ocular pruritus and photophobia. These patients may also experience foreign body sensation, mucoid discharge, and bletharospasm. Most patients experience a thin, copious milk-white fibrinous secretion (composed of eosinophils, epithelial cells, and Charcot-Leyden crystals). Compared to AKC, the skin of the lids and lid margins is spared and symptoms are confined to the conjunctiva and cornea. Papillary response of the conjunctiva, primarily of the upper tarsus or limbus, is seen, with flattened papillae greater than 1 mm in diameter. These classic “cobblestone” tarsal papillae are often associated with thick, ropy mucus. With disease progression, fibrous tissue proliferation can lead to “giant papillae” reaching 7–8 mm in diameter, predominantly on the upper tarsal plate. This is described as “cobblestoning,” with the cobblestones persisting during the quiescent phase, and becoming extremely swollen during the active phase, which is usually spring (Fig. 3). Horner-Trantas dots are aggregates of degenerated epithelial cells and eosinophils accumulating on the gelatinous, hyperplastic limbus. Corneal changes, especially shield ulcers and subsequent scarring can be sight-threatening. Other ocular complications reported include steroid-induced cataract, steroid-­ induced glaucoma, central corneal scars, keratoconus, irregular astigmatism, and dry eye syndrome. Diagnosis is primarily clinical, based on history and physical findings.

Fig. 3  Vernal keratoconjunctivitis: Note the excessive cobblestoning of conjunctival surface and mucus forming pseudomembrane over the epithelial surface. (Reproduced with permission from Bielory L. Allergic and immunologic disorders of the eye. Part II: ocular allergy. J Allergy Clin Immunol. 2000 Dec;106(6):1019–32)

49

The presentation is typically in young males in warm climates, who may experience intense photophobia, and ptosis and show the classic cobblestone papillae. Earlier age of presentation and lack of dermatitis of periocular skin help distinguish VKC from AKC. Question 4 Which of the following findings is more commonly associated with atopic keratoconjunctivitis rather than vernal keratoconjunctivitis? A. Dennie Morgan lines B. Lack of periorbital involvement C. Younger age of onset D. Corneal involvement E. Seasonal flaring of symptoms Answer and Explanation Answer: D Corneal involvement is more common in AKC than in VKC. Dennie Morgan lines are nonspecific infraorbital creases that may be seen in many forms of ocular allergy. Atopic keratoconjunctivitis frequently presents with periorbital eczema. Patients with VKC are often younger when they initially present than in AKC. Seasonal flaring will be seen in SAC and VKC rather than in AKC where remission and flaring are without a seasonal correlation.

10 Atopic Keratoconjunctivitis (AKC) AKC is characterized by bilateral, chronic inflammation involving the conjunctiva and eyelids that can be potentially sight-threatening. Despite relative rarity, AKC is thought to occur anywhere between 20 and 40% of individuals with atopic dermatitis. It is more prevalent in men than women, with peak incidence between the second and fifth decade of life. Although AKC has not been associated with a racial or geographic predilection, a family history of atopy is common. Similarly, ~95% of patients with AKC will have concomitant eczema and ~87% of patients will have concomitant asthma. An understanding of the pathogenesis of AKC comes from histologic and immunohistochemical analysis of conjunctival biopsy specimens and from tear film analysis for mediators and cells. These studies demonstrate the role of type 1 and 4 hypersensitivity mechanisms. The conjunctival epithelium of patients with AKC has mast cells (MCT type) and eosinophils, unlike normal individuals. Moreover, there is an increase in helper T cells, amplifying the immune response. The substantia propria in AKC has an increased number of mast cells (compared to normal) as well as eosinophils (not normally present in substantia propria). Moreover,

50

M. R. Norris and N. Bhardwaj

involvement may help distinguish AKC from VKC. Near to complete resolution of symptoms when out of their season and lack of chronic conjunctival inflammation in SAC also helps to distinguish VKC from AKC. Question 5 Which of the following is best for differentiation of vernal keratoconjunctivitis from giant papillary conjunctivitis?

Fig. 4  Atopic keratoconjunctivitis. Note the injection and chemosis of bulbar conjunctiva. Periorbital skin is affected by the formation of infraorbital creases (Dennie Morgan Lines). (Reproduced with permission from Bielory L. Allergic and immunologic disorders of the eye. Part II: ocular allergy. J Allergy Clin Immunol. 2000 Dec;106(6):1019–32)

the substantia propria in AKC shows increased T cells, B cells, and Langerhans cells leading to a complex immune cell profile. The greater the inflammatory cell density in the conjunctiva, the poorer is the tear stability. While Th2 cytokines predominate most allergic processes, lymphocytes with Th1 cytokine profiles have been demonstrated in substantia propria of AKC patients. The main symptom of AKC is itching, followed by discharge, redness, blurring of vision, photophobia, and pain. The periocular skin may show scaling and flaking on a reddened base. The skin of the lids may become leathery, leading to the development of cicatricial ectropion and lagophthalmos (incomplete closure of the eyelids). The infraorbital skin of the eyelid may show single or double infraorbital creases called Dennie Morgan lines caused by edema or thickening (Fig. 4). Extensive chronic eye rubbing may lead to loss of lateral eyebrows in older patients (de Hertoghe’s sign). Other manifestations include lateral canthal ulceration, cracking, and loss of eyelashes (madarosis). Secondary staphylococcal blepharitis may occur due to maceration and induration of eyelids. The tarsal conjunctiva may show papillary reaction, more prominently in the inferior conjunctival fornix. Peri-limbal, gelatinous hyperplasia may occur on the bulbar conjunctiva. A reported finding is Horner-­ Trantas dots (aggregates of degenerated epithelial cells and eosinophils accumulating on the gelatinous, hyperplastic limbus). The most common corneal finding is punctate epithelial keratopathy. Persistent epithelial defects, scarring, microbial ulceration, and neovascularization all lead to vision loss. Some patients may experience noninflammatory progressive thinning of the cornea (keratoconus). Corneal scarring and neovascularization may result in blindness. Frequent use of steroids in treatment of this condition can lead to the development of cataracts, although the lens opacity typically associated with AKC is an anterior or subcapsular cataract. Anterior uveitis or iris abnormalities are not reported. Older age at presentation and significant lid

A. Contact lens use preceding onset of symptoms B. Intermittent nature of symptoms C. Seasonal flaring D. Photophobia E. Periocular eczema Answer and Explanation Answer: A History of contact lens use is the main differentiating factor from vernal keratoconjunctivitis which presents similarly.

11 Giant Papillary Conjunctivitis Giant papillary conjunctivitis (GPC) is characterized by a chronic inflammatory process leading to formation on giant papillae of the upper tarsal conjunctiva. It has been reported in individuals wearing soft, hard, and rigid gas-permeable contact lenses. It is moreover seen in association with ocular prostheses and exposed sutures coming in contact with the conjunctiva. As many as 20% of soft contact lens wearers may be affected by GPC. The incidence is ten times higher in people wearing regular soft contact lenses than rigid (gas-­ permeable) contact lenses. The incidence among daily-wear disposable contact lens users and rigid-contact lens users is the same. Wearing disposable contact lenses during sleep increases the risk of developing GPC. Asthma and allergic rhinitis are also risk factors. The inflammatory response seen in GPC is likely multifactorial, with mechanical trauma and chronic irritation of the conjunctiva secondary to contact lens use believed to cause the onset of GPC. The buildup of protein on the surface of the contact lens may cause an allergic reaction. Poor tear clearance in these patients may further facilitate protein buildup on the lens. Biopsy studies show increased mast cells (MCT) in the conjunctival epithelium although no significant increase is seen in the substantia propria. Release of mediators from mast cells (as evidenced by tryptase found in the tears of these patients) causes increased capillary permeability and tissue infiltration with inflammatory cells, similar to that seen in seasonal and perennial allergic conjunctivitis. Patients complain of ocular itching, redness, burning, increased mucus discharge in the morning, and photophobia.

Allergic Diseases of the Eye

51

Table 1  Clinical and histopathological features of allergic diseases of the eye Feature Clinical attributes

SAC Intermittent, usually spring Seasonal allergens Coexistent allergic rhinitis, asthma Bilateral involvement Signs/symptoms Ocular itching, tearing, redness, chemosis, not sight-threatening

PAC Persistent, year-round Perennial allergens Coexistent allergic rhinitis, asthma Bilateral involvement Ocular itching, tearing, redness, chemosis, not sight-threatening

VKC Peak incidence 3–20 years Male predominance 3:1 Bilateral involvement Prevalent in warm, dry climate, worse in spring

AKC Peak incidence 20–50 years Both males and females Persistent with exacerbations Bilateral involvement Coexistent atopic dermatitis, asthma, allergic rhinitis, chronic symptoms

GPC Both males and females, bilateral involvement, exposure to contact lenses and prosthesis, chronic symptoms

Severe ocular itching, photophobia, stringy mucoid discharge, cobblestoning and giant papillae on upper tarsal conjunctiva, corneal involvement with shield ulcers, Trantas’ dots on limbus, sight-threatening

Mild ocular itching and mucoid discharge, giant papillae, contact lens intolerance, protein buildup on contact lenses, not sight-threatening

Pathophysiology Mast cell/ IgE-mediated, atopy Mast cell/ eosinophil infiltration of conjunctiva and substantia propria

Mast cell/ IgE-mediated, atopy Mast cell/ eosinophil infiltration of conjunctiva and substantia propria

IgE- or non-IgE-mediated. Increased mast cells and eosinophils in conjunctival epithelium and substantia propria. Eosinophil major basic protein deposition in conjunctiva, increased collagen

Severe itching, erythema and flaking of periocular skin, photophobia, cobblestoning and giant papillae in inferior fornix, corneal erosions, conjunctival scarring, Trantas’ dots, keratoconus (rare) anterior subcapsular cataract, sight-threatening IgE- or non-IgE-mediated. Increased mast cells and eosinophils in conjunctival epithelium and substantia propria Epithelial cell hypertrophy, increased collagen

The presentation may be months to years after starting contact lens use. Small papillae are seen in mild cases, due to contact lenses riding high on the eyeball and irritating the conjunctival lining of the eyelid. Tear deficiency contributes to the progression of these changes, with redness of the inside of the upper eyelid being the first finding, followed by opacification and development of characteristic papillae. History of contact lens use is the main differentiating factor from vernal keratoconjunctivitis which presents similarly. A summary of the high yield features of these ocular allergic disorders can be found in Table 1.

12 Differential Diagnosis of Ocular Allergy Clinical allergists and immunologists should be able to differentiate allergic conditions from other immune-mediated ocular disorders as various inflammatory conditions of the eye may present with red eyes. A thorough clinical examination is the first step in this process. It begins with the external component that surrounds the eye, and then the eye itself. Clinical signs to note when examining the eyelids and eyelashes are erythema of the lid margin, scaling, thickening or swelling (blepharitis and dermatitis), periorbital discoloration (allergic shiners, heliotrope), blepharospasm, or ptosis. The conjunctiva should be examined for chemosis ­(swelling),

Giant papillae, conjunctival thickening, increased mast cells and eosinophils in conjunctival epithelium

hyperemia (injection), palpebral and bulbar papillae, and cicatrization (scarring). Assessment of the ocular discharge helps differentiate different forms of conjunctivitis (clear in allergic or viral thick and purulent in bacterial). When deeper tissues such as sclera, vitreous, or choroid are involved, injection and pain are the notable findings. Autoimmune disorders, such as systemic lupus erythematosus and rheumatoid arthritis, cause scleritis associated with pain. An important sign of intraocular inflammation (iritis, uveitis) is the formation of a ring of erythema at the limbal junction of the cornea (ciliary flush), whereas there is characteristic sparing of the peri-limbal area in conjunctivitis. Chronic inflammatory response in the conjunctiva and substantia propria leads to the formation of papillae seen in AKC, VKC, and GPC. A comprehensive list of differentials to consider based on their anatomic location can be seen in Fig. 5. The eyelid skin is soft, pliable, and thin. Inflammation of the eyelid margins is called blepharitis and is often misdiagnosed as an ocular allergy. Like atopic dermatitis patients, colonization with Staphylococcus aureus is seen. Patients complain of symptoms such as burning, itching, tearing, and a feeling of dryness primarily in the mornings. The eye may be glued shut when the patient awakens in the morning due to crusted exudate. Scales and collarettes of exudative material around the eyelash bases are seen. Blepharitis can be controlled by improved hygiene with detergents such as nonstinging baby shampoos. Allergic contact blepharoconjuncti-

52

M. R. Norris and N. Bhardwaj Location

Eyelid

Acne Rosacea Staphylococcal

Conjunctival

Infectious

Allergic Chronic

Vermal Conjunctivitis Atopic Conjunctivitis Giant Papillary Conjunctivitis

Acute Seasonal Perennial

Lacrimal Gland

Immune Viral

Adenovirus

Bacterial Chlamydia Gonococcal

Parasitic Fungal

Dry Eye

Autoimmune

Lacrimal Duct

Intraocular Uveitis Sarcoid Autoimmune Infectious

Vascuitis Episcleritis/Scleritis Pseudotumor Pemphigus/Pemphigoid Kawasaki’s Disease

Immunodeficiency Ataxia-Telangiectasia AIDS

Contact

Fig. 5  Differential diagnosis of the red eye. (Reproduced with permission from Bielory L, Delgado L, Katelaris CH, Leonardi A, Rosario N, Vichyanoud P. ICON: Diagnosis and management of allergic conjunctivitis. Ann Allergy Asthma Immunol. 2020 Feb;124(2):118–134)

vitis may be caused by cosmetics applied to the hair (hair dye), face, fingernails (nail paint), or periorbital area. Ninety percent of patients with this condition are female. It is predominantly lymphocyte-mediated hypersensitivity reaction that affects the periocular skin, eyelids, or ocular surface. These reactions linked to cosmetics are often the result of sensitization to preservatives, fragrances, and additives in the products. Contact lens solutions have also been implicated in the development of allergic contact blepharoconjunctivitis. Infectious conjunctivitis can be divided into bacterial or viral. Acute bacterial conjunctivitis is characterized by symptoms localized primarily to one eye initially, subsequently involving the other side often several days later. Typical symptoms include ocular irritation, conjunctival redness, and mucopurulent discharge. Inflammation is mediated largely by polymorphonuclear cells (neutrophils). A preauricular node may sometimes be palpable. Mucopurulent discharge of chlamydial infection lasts for more than 2 weeks. Intracytoplasmic inclusion bodies may be seen on conjunctival scrapings. Viral conjunctivitis presents with watery discharge, conjunctival injection, and chemosis. If the cornea is involved, there may be associated ocular pain, which should prompt referral to an ophthalmologist. Tear film has a trilaminar structure composed of an outer lipid layer, an aqueous middle layer, and an inner mucin layer. True dry eye disease develops from decreased tear production, increased tear evaporation, or an abnormality of the tear film. These changes cause the tear film to become hyperosmolar, which in turn results in irritation and inflammation of the conjunctiva. Dry eye syndromes are commonly reported in computer users, long-term contact lens wearers, perimenopausal and postmenopausal women, and patients with acquired immunodeficiency syndrome. Patients may present with mild conjunctival injection and potentially with

excessive tearing, leading to misidentification as seasonal or perineal allergic conjunctivitis. Patients often experience a gritty, sandy feeling in the eye along with itching and burning. These symptoms worsen throughout the day as the aqueous tear film evaporates. Winter months are associated with exacerbation of the symptoms due to decrease in relative humidity with indoor heating in the households. Of key clinical importance, long-term use of medications with anticholinergic properties can cause dry eye syndrome. These include first-generation antihistamines, tricyclic antidepressants, β-blockers, and chemotherapeutic agents. Keratitis can present with red eye, foreign body sensation, and photophobia. Bacterial keratitis is most often seen with contact lens use. Herpes infection can cause viral keratitis. Scleritis typically presents with severe pain and is usually associated with an underlying systemic illness. The pain is often worse at night and early morning and radiates into the face. There may be accompanying redness, watery discharge, and photophobia. Episcleritis also presents with redness, clear discharge, and engorged blood vessels on a clear scleral background. It is seen in middle-aged females. Unlike scleritis, episcleritis is painless and does not cause visual impairment. It may sometimes be associated with autoimmune disease, although most cases are isolated events. Iritis typically presents with isolated limbal redness and constricted pupil. It is usually due to underlying infectious or autoimmune conditions. Acute closure glaucoma is a rare cause of red eye and is usually associated with headache, nausea, and vomiting. This is an ocular emergency characterized by a distinctive ciliary flush. Drug-induced conjunctivitis is caused mostly by preservatives such as benzalkonium chloride, thimerasol, parabens, ethylenediaminetetraacetic acid (EDTA), and chlorobutanol, in eye drops, with prolonged use exceeding 3 months. The

Allergic Diseases of the Eye

classic presentation is worsening of ocular symptoms after initial improvement with the medication. Discontinuation of the medication leads to the resolution of symptoms. Reactions that occur in response to prolonged use of topical ­preparations are often seen in the lower eyelid and inferior conjunctiva due to the pooling of the liquid therapeutics in this area. The typical signs are red-colored inflamed conjunctiva, papillae development, keratitis, and chemosis. Vasoconstricting eye drops may also lead to conjunctivitis medicamentosa characterized by increased conjunctival injection and rebound hyperemia. Occupational conjunctivitis occurs in response to airborne substances in the workplace, inciting inflammation via allergic or nonallergic pathways. Examples of such substances include laboratory animal antigens grain, organic chemicals and irritants, wool, plants, and detergent protease. Occupational rhinitis and asthma may often coexist. Nonallergic Perennial (Vasomotor) Conjunctivitis is a perennial, chronic condition in which the symptoms are neither immunologic nor infectious in origin and are not associated with ocular eosinophilia. While it is more common in the elderly population, it is seen also in athletes, especially those exposed to chlorinated swimming pools. Patients with vasomotor conjunctivitis complain of excessive tearing with exposure to tobacco smoke, fumes, and perfumes, leading to conjunctival injection. Among the elderly, the cause is physiologic age-related mechanical and anatomic changes.

53

the risk of irritant effects of the preservatives. Refrigeration of ocular medications has also been shown to improve the tolerability of their application as well as the overall patient perception of symptom improvement. Ocular lubricants are available at different viscosity levels. The low viscosity preparations are effective for mild conditions, in terms of symptom relief and tolerance. For more severe cases, high-viscosity tears are used but may cause blurring of vision Antihistamines are the first-line drugs often used by patients on their own prior to seeking medical attention. However, overuse of this class of medications, especially the first-generation antihistamines can worsen dry eye syndrome, and may even worsen allergic conjunctivitis by decreasing the barrier effect of natural tears film. Topical antihistamines are preferred due to their rapid onset of action, and lack of systemic side effects. By acting as an inverse agonist on the histamine receptor, topical antihistamines block the acute phase of allergic response. They have a high safety margin however repeat dowing is necessary as the duration of action is often short. Some second-generation antihistamines, such as emadastine, have a longer duration of action and are better tolerated. Cetirizine is available as an ophthalmic solution and has shown good efficacy and tolerability. Topical antihistamines should not be used in patients with acute angle closure glaucoma. Agents mitigate the late phase of an allergic response by inhibiting the release of mast cell mediators. Single-action mast cell stabilizers usually require a preloading period of 3–5  days but up to 2  weeks and frequent instillation. This 13 Management of Ocular Allergy often leads to poor compliance. Some agents in this category are cromolyn, lodaxamide, nedocromil, and pemirolast. With The first step in managing ocular allergies is to identify the the availability of dual agents and multi-action agents, the causative stimulus and avoid it if possible. Identification can use of single-action mast cell stabilizers is limited. They are be done via skin prick testing or serum IgE testing, followed sometimes used as add-on agents. Newer topical ophthalmic by simple allergen avoidance measures such as wearing pro- agents have both antihistamine and mast cell stabilizing tective eyewear, keeping windows closed in both the home actions. Their duration of action is longer requiring less freand while driving, and showering before going to bed. quent instillation, improving compliance. Moreover, they Frequent washing of clothing and bedding is also helpful. have fewer side effects than pure antihistamine agents, Cold compresses and refrigerated artificial tears also provide improving tolerability. The dual agents target both the early symptomatic relief. Individuals who use contact lenses may and late phases of allergic response. Epinastine has been need to switch to eyeglasses during their peak seasons or use shown to increase comfortable wearing time among contact daily disposable lenses. During administration of topical lens wearers, with decreased need for rewetting drops. medications, contact lenses should be removed to avoid Other agents include ocular decongestants and topical interaction between contact lenses and preservatives in oph- nonsteroidal anti-inflammatories. The main effect of ocular thalmic preparations. The lenses may be replaced after decongestants is to improve the appearance of ocular ery10 min of topical medication administration. Daily dispos- thema. They do not improve the sensation of ocular itch. This able lenses are ideal during the pollen season as allergens can class of medication may be associated with worsening of accumulate on the lens; allergens can also become trapped narrow-angle glaucoma, rebound hyperemia, conjunctivitis between the conjunctival surface and the contact lens. pigmentosa, and tachyphylaxis. Therefore, long-term use of Artificial tears can help by washing away allergens and ocular decongestants is discouraged and should be avoided diluting inflammatory mediators on the ocular surface. It can in patients with angle closure glaucoma. They should also be also serve to mitigate tear deficiencies if present. Preservative-­ used with caution in those with cardiovascular disease, diafree preparations should be used when possible, minimizing betes, and hyperthyroidism. The only FDA-approved

54

o­ phthalmic nonsteroidal anti-inflammatory agent for treatment of allergic conjunctivitis is ketorolac. It blocks prostaglandin synthesis. Its use is limited to intermittent use or as an ­add-­on medication due to the availability of other preparations with higher efficacy and tolerability. Topical corticosteroids decrease the recruitment of inflammatory cells and subsequent cytokine production. However, their use is associated with cataract formation and increase in intraocular pressure (IOP). Their use is therefore limited to severe or refractory cases. Older formulations such as prednisolone, dexamethasone, and fluorometholone are known to induce cataract. Newer preparations such as loteprednol do not increase the risk of cataract but are associated with an increase in IOP at higher concentrations. Therefore, the use of topical corticosteroids should be limited to 2 weeks. Patients who need to be on these medications for longer than that should be monitored for adverse effects, under the care of an ophthalmologist. Topical corticosteroids are sometimes used for lid eczema however mild preparation such as hydrocortisone 1% are recommended for this indication. Intranasal corticosteroids (INS) have also been shown to have a beneficial effect on ocular allergy, possibly via inhibition of ocular-nasal reflex. Prolonged use of INS can potentially increase the risk of glaucoma and an increase in IOP compared with placebo has been demonstrated; however, glaucomatous optic disc changes have not been noted. Commonly prescribed INS such as fluticasone and mometasone have not been found to increase the risk of cataract or glaucoma and there is minimal to no systemic absorption. Patients using INS, especially high doses, on a prolonged base should have an annual eye examination and IOP measurement. Patients with VKC or AKC may need short bursts of systemic steroids for exacerbations. Calcineurin inhibitors, such as topical cyclosporine or tacrolimus, can be used for severe or refractory allergic conjunctivitis, AKC and VKC.  Moreover, topical tacrolimus 0.03% or 0.1% ointment and pimecrolimus 1% ointment may be used for eyelid eczema, VKC, and AKC. Their use presents the risk of local infections such as molluscum contagiosum, papilloma virus, and herpes. These agents are often associated with a burning sensation, affecting compliance. Systemic cyclosporine may be used for sight-­ threatening AKC.  A topical cyclosporine preparation (0.05%) is FDA-approved for dry eye syndrome. To date, allergen immunotherapy is the only therapy available that has been shown to modify the disease process. For treatment of SAC or PAC, AIT has been studied in both its subcutaneous form (SCIT) and sublingual form (SLIT). Both have been shown to offer improvement in conjunctival symptoms and medication utilization scores; however, SCIT is more efficacious than SLIT. SLIT has been found to have more frequent, milder adverse effects—especially at the

M. R. Norris and N. Bhardwaj

start of therapy; however, it has a lower risk of anaphylactic reaction. As SLIT has been shown to have higher rates of discontinuation than SCIT, the few pharmacoeconomic analyses to date have favored SCIT in favor of SLIT. Both forms will help treat ocular allergy as well as other forms of allergy such as nasal symptomatology. As both options have their own risks and benefits, the decision remains patientdirected with adherence being the greatest hurdle for either method.

14 Emerging Therapies With longer pollen seasons characterized by rising pollen counts and increasingly allergenic pollen, we are seeing seasonal allergies becoming more problematic to manage. With the late phase reaction thought responsible for worsening conjunctival symptoms and decreased medication efficacy throughout a pollen season, there is increased interest in the use of biologics for the treatment of allergic conjunctivitis. Biologics have also been of research interest for other ocular allergic disorders. Omalizumab, a recombinant humanized monoclonal antibody directed toward circulating IgE, has been shown to improve symptoms of allergic conjunctivitis in at least two randomized control studies evaluating the treatment of allergic rhinitis. Although no such studies exist for AKC or VKC, case reports exist suggesting the possible benefit of omalizumab after its initiation for a different indication. Despite having evidence of Th2-mediated inflammation, Dupilumab (a fully human monoclonal antibody that blocks IL-4 and IL-13) is associated with new-onset conjunctivitis in 9–28% of patients in clinical trials and 25–50% of patients in observational studies. The mechanism for this adverse effect has yet to be elucidated; however, one theory is that conjunctival inflammation is the result of IL-13 inhibition and subsequent goblet cell apoptosis and decreased mucin production. Severe atopic dermatitis, preexisting conjunctivitis, and low dupilumab levels are associated with an increased risk of conjunctivitis. The anti-IL-5 biologic agents mepolizumab and reslizumab, as well as the anti-IL-5 receptor agent benralizumab, have not been studied in the context of ocular allergy. Janus Kinase inhibitors are of increasinginterest in the treatment of atopic disease, with tofacitinib showing efficacy in a recent 2020 murine model of allergic conjunctivitis. Although biologics are likely to hold a major role in the future treatment of ocular allergic disease, neither have been extensively studied to date nor have been approved for use for the indication of ocular allergy. Contact lens-based drug delivery systems are also an area of interest. A contact lens-based drug delivery system for ketotifen has been shown to be comparable to direct topical drug delivery. Epinastine-releasing soft contact lenses have been shown to have prolonged drug delivery and higher effi-

Allergic Diseases of the Eye

cacy compared to epinastine hydrochloride eye drops. Such innovations would be promising options for providing vision correction as well as providing treatment of ocular allergy for contact lens wearers.

Bibliography 1. Ackerman S, Smith LM, Gomes PJ.  Ocular itch associated with allergic conjunctivitis: latest evidence and clinical management. Ther Adv Chronic Dis. 2016;7(1):52–67. https://doi. org/10.1177/2040622315612745. 2. Bielory L.  Allergic and immunologic disorders of the eye. Part II: ocular allergy. J Allergy Clin Immunol. 2000;106(6):1019–32. https://doi.org/10.1067/mai.2000.111238. 3. Bielory L.  Differential diagnoses of conjunctivitis for clinical allergist-immunologists. Ann Allergy Asthma Immunol. 2007;98(2):105–14; quiz 114-107, 152. https://doi.org/10.1016/ S1081-­1206(10)60681-­3. 4. Bielory L, Delgado L, Katelaris CH, Leonardi A, Rosario N, Vichyanoud P.  ICON: diagnosis and management of allergic conjunctivitis. Ann Allergy Asthma Immunol. 2019; https://doi. org/10.1016/j.anai.2019.11.014. 5. Bielory L, Schoenberg D.  Emerging therapeutics for ocular surface disease. Curr Allergy Asthma Rep. 2019;19(3):16. https://doi. org/10.1007/s11882-­019-­0844-­8. 6. Caillaud D, Martin S, Segala C, Besancenot JP, Clot B, Thibaudon M, French Aerobiology N.  Effects of airborne birch pollen levels on clinical symptoms of seasonal allergic rhinoconjunctivitis. Int Arch Allergy Immunol. 2014;163(1):43–50. https://doi. org/10.1159/000355630. 7. Caillaud DM, Martin S, Segala C, Besancenot JP, Clot B, Thibaudon M, French Aerobiology N. Nonlinear short-term effects of airborne Poaceae levels on hay fever symptoms. J Allergy Clin Immunol. 2012;130(3):812–814 e811. https://doi.org/10.1016/j. jaci.2012.04.034. 8. Caillaud DM, Martin S, Segala C, Vidal P, Lecadet J, Pellier S, et  al. Airborne pollen levels and drug consumption for seasonal allergic rhinoconjunctivitis: a 10-year study in France. Allergy. 2015;70(1):99–106. https://doi.org/10.1111/all.12522. 9. Durham SR, Nelson HS, Nolte H, Bernstein DI, Creticos PS, Li Z, Andersen JS. Magnitude of efficacy measurements in grass allergy

55 immunotherapy trials is highly dependent on pollen exposure. Allergy. 2014;69(5):617–23. https://doi.org/10.1111/all.12373. 10. Komi DEA, Wöhrl S, Bielory L.  Mast cell biology at molecular level: a comprehensive review. Clin Rev Allergy Immunol. 2020;58(3):342–65. https://doi.org/10.1007/ s12016-­019-­08769-­2. 11. Feo Brito F, Mur Gimeno P, Carnes J, et al. Grass pollen, aeroallergens, and clinical symptoms in Ciudad Real. Spain J Investig Allergol Clin Immunol. 2010;20(4):295–302. 12. Janeczko P, Norris MR, Bielory L. Assessment of receptor affinities of ophthalmic and systemic agents in dry eye disease. Curr Opin Allergy Clin Immunol. 2021;21(5):480–5. https://doi.org/10.1097/ ACI.0000000000000773. 13. Kiotseridis H, Cilio CM, Bjermer L, Tunsater A, Jacobsson H, Dahl A.  Grass pollen allergy in children and adolescentssymptoms, health related quality of life and the value of pollen prognosis. Clin Transl Allergy. 2013;3:19. https://doi. org/10.1186/2045-­7022-­3-­19. 14. Mendelson S, Quintanilla J, Norris MR, Bielory L.  Psychosocial impact of ocular surface allergic inflammatory disorders. Curr Opin Allergy Clin Immunol. 2021;21(5):465–71. https://doi. org/10.1097/ACI.0000000000000776. 15. Norris MR, Bielory L.  Cosmetics and ocular allergy. Curr Opin Allergy Clin Immunol. 2018;18(5):404–10. https://doi.org/10.1097/ ACI.0000000000000474. 16. Barney NP.  Chapter 38. Allergic and immunologic diseases of the eye. In: Adkinson NF, Bochner BS, Busse WW, Holgate ST, Lemanske RF, Simons FER, editors. Middleton’s allergy. 7th ed. Philadelphia, PA: W.B. Saunders; 2010. p. 605–23. 17. Prince A, Norris MR, Bielory L. Seasonal ocular allergy and pollen counts. Curr Opin Allergy Clin Immunol. 2018;18(5):387–92. https://doi.org/10.1097/ACI.0000000000000475. 18. Rodrigues J, Kuruvilla ME, Vanijcharoenkarn K, Patel N, Hom MM, Wallace DV.  The spectrum of allergic ocular diseases. Ann Allergy Asthma Immunol. 2021;126(3):240–54. https://doi. org/10.1016/j.anai.2020.11.016. 19. Valentine L, Norris MR, Bielory L. Comparison of structural components and functional mechanisms within the skin vs. the conjunctival surface. Curr Opin Allergy Clin Immunol. 2021;21(5):472–9. https://doi.org/10.1097/ACI.0000000000000775. 20. Waisel Y, Mienis Z, Kosman E, Geller-Bernstein C.  The partial contribution of special airborne pollen to pollen induced allergy. Aerobiologia. 2004;20(4):197–208.

Allergic Rhinitis Meera R. Gupta and Jessica Palmieri

Case 1 A 20-year-old male presents to your clinic with complaints of congestion, clear rhinorrhea, sneezing, itchy watery eyes, and throat clearing. He states that his symptoms are present year-round but he does note worsening of symptoms in the spring and summer. He does not have a long history of seasonal allergies. He moved to your town 2 years ago for college from out of state and he started noticing these symptoms a few months after starting school. He has a remote history of eczema as a child which is now resolved. He does not have a history of asthma and he denies allergies to foods, drugs, vaccines, and Hymenoptera. He has tried taking over-the-­ counter loratadine 10 mg daily and cetirizine 10 mg daily for 1–2  months at a time respectively, but he did not see any improvement in his symptoms with either medication. The patient states that he is having daily symptoms and is having difficulty falling and staying asleep 4–5 nights per week. He is subsequently having trouble focusing in class and is concerned about his grades. He would like to start a new medication regimen as he has no relief with daily oral antihistamines. In preparation for today’s appointment, he has withheld all oral antihistamines for 1 week and he does not take any additional medications or supplements. On your physical exam, you note significant bilaterally boggy, pale, and hypertrophied nasal turbinates. Cobblestoning of the oropharynx is present as are palatal petechiae. Dark circles are present under his eyes and his sclera are faintly red. Skin is normal texture and hydrated. Lungs are clear to auscultation without wheezes. The remainder of the physical exam is unremarkable.

Skin prick testing is done in the office. The results are as follows: Allergen Histamine (+ control) Saline (− control) Dust mite DF Dust mite DP Cockroach Mix Alternaria Russian Thistle Timothy Grass Bermuda Grass Johnson Grass Bahia Grass

Wheal/flare in mm 7/25 0/0 11/40 20/35 10/30 5/5 10/10 25/45 8/30 10/32 10/20

Question 1 Based on his reported symptoms, how would you best classify this patient’s allergic rhinitis? A. Moderate/severe persistent nonallergic rhinitis B. Moderate/severe intermittent allergic rhinitis C. Mild persistent allergic rhinitis D. Moderate/severe persistent allergic rhinitis

Question 2 What treatment plan would be appropriate to start this patient on after your first visit? (monotherapy?) A. Leukotriene receptor antagonist monotherapy B. Second generation antihistamine + inhaled nasal corticosteroid C. Inhaled nasal corticosteroid + inhaled nasal antihistamine spray D. A 7–10 day course of inhaled nasal decongestant monotherapy followed by a second-generation antihistamine

M. R. Gupta (*) · J. Palmieri Department of Pediatrics, Division of Immunology, Allergy, and Retrovirology, Baylor College of Medicine, Houston, TX, USA e-mail: [email protected] © The Author(s), under exclusive license to Springer Nature Switzerland AG 2022 M. Mahmoudi (ed.), Absolute Allergy and Immunology Board Review, https://doi.org/10.1007/978-3-031-12867-7_5

57

58

M. R. Gupta and J. Palmieri

Question 3 Your patient returns to your office in 6 weeks for follow-up. He has been properly adhering to the medication regimen you prescribed him however he has only noted mild improvement in his symptoms. Physical exam continues to show significant mucosal edema. What add-on therapy would be most appropriate? A. Addition of a second-generation antihistamine to your leukotriene receptor antagonist B. Addition of a 3-day course of an inhaled nasal decongestant to your inhaled nasal corticosteroid + inhaled nasal antihistamine spray regimen C. Addition of a second-generation antihistamine to your inhaled nasal antihistamine D. Stopping his current regime and starting a 3–5 day course of pseudoephedrine to help with turbinate hypertrophy

Question 4 Cytokines driving the inflammatory response in allergic rhinitis include A. Il-1 and IL-6 B. Il-10 C. Il-4, Il-5, Il-13 D. Soluble Il-2 and Il-18

Question 5 False positive can be seen in epicutaneous skin testing (SPT) with which of the following? A. Dermatographic patients B. Low potency of extracts C. Recent antihistamine use D. Insufficient (low) pressure from operator during SPT placement Answers: D, C, B, A, C

1 Allergic Rhinitis Allergic Rhinitis (AR) is an IgE-mediated inflammatory disease that is characterized by one or more of the following symptoms: nasal congestion, anterior or posterior rhinorrhea, sneezing, and itching and is frequently accompanied by symptoms involving the eye, ear, or throat. AR affects up to 60 million people in the United States annually, with surveys requiring a physician-confirmed diagnosis reporting a prevalence of 14% in adults and 13% in children. Uncontrolled AR has a significant effect on quality of life including impairments in physical and social functioning, disturbed

sleep, daytime somnolence, fatigue, irritability, memory defects, depression, and decreased attention and learning, as well as lost work and school days. Appropriate therapy can substantially reduce societal costs and significantly improve quality of life (QOL).

1.1 Classification AR can be classified by the frequency and duration of symptoms, the temporal pattern and context of exposure, or by symptom severity. Appropriate classification can assist in the selection of treatment strategies for an individual patient. Symptom frequency has been divided into intermittent, occurring  =4 wheezing days+ risk factors

Expert Report, using 12 years of age or older as a reference. The differences at younger ages are described. Assume a similar classification as in a 12 y/o if not listed (Table 1). In this case, the patient required SABA use every day, which indicates moderate; nighttime awakenings 4/week are also moderately persistent. Her FEV1 is 80% predicted or normal for age, which fits into intermittent or mild persistent. She visited the emergency department and was hospitalized at least twice in the last year, so she is not intermittent but persistent. Based on the overall impairment and risk categories, her asthma is moderate persistent. Question 2 How would you treat her? A. Formoterol 2 puffs bid B. Fluticasone propionate 44 μg 2 puffs bid C. Budesonide 180 μg 1 puff bid

Correct answer: E Patients with moderate persistent asthma should receive low-dose ICS/LABA (Budesonide/Formoterol 80/4.5 1 puff bid, Mometasone/Formoterol 100/5  μg 1 puff bid, Fluticasone/Salmeterol 100/50 1 puff bid) or medium-dose ICS (Beclomethasone dipropionate 80  μg 3–5 puffs/day, Fluticasone propionate 110 μg 1–2 puffs bid, Mometasone furoate 200  μg 2 puffs daily, Budesonide DPI 200  μg 2–4 puffs/day) as the preferred choice. The revised guidelines introduced the term single maintenance and reliever therapy (SMART) as the preferred choice for step 3 (moderate) and 4 (severe) subjects older than 4 years of age. SMART consists of a low-dose ICS/Formoterol combination, used as maintenance and rescue therapy 1–2 puffs qd-bid and as needed, up to 12 puffs per day for 12 y/o and older and up to 8 puffs/day for those 5–11 years of age. Choice A, LABA alone, should not be used in the treatment of asthma due to an increased risk of death, particularly among subjects homozygous for arginine at codon 16 of the β2 agonist receptor. Choice B, fluticasone 44  μg 2 puffs bid, or 176  μg/day, is a low dose recommended for mild persistent patients. Similarly, 360 μg/day budesonide, choice C, is also a low-dose steroid, requiring 2 puffs bid to achieve medium potency. Choice D would correspond to high-dose ICS/LABA, a medium dose would be 250/50 1 puff bid and 100/50 would be low-dose ICS/ LABA. According to new guideline revisions, Salmeterolbased ICS/LABA would not be recommended for use as rescue and maintenance due to the long onset of action of Salmeterol. Table 2 shows the medication dosage recommended according to severity criteria incorporating the revised guideline recommendations. Adequate inhaler technique education is an important part of asthma care.

77

Asthma

Table 2  Asthma treatment recommendations according to severity and age Asthma treatment: > = 12 years old Intermittent Mild Preferred SABA prn Low-dose ICS qd and SABA prn or ICS + SABA prn Consider Immunotherapy

Beclomethasone HFA 80 μg Fluticasone 110 μg Mometasone DPI 200 μg Budesonide DPI 180 μg Budesonide/ formoterol 160/4.5 Mometasone/ formoterol 200/5 Alternative N/A

Moderate

Severe

Severe+

++

Low-dose ICS/ formoterol qd and prn* Immunotherapy

Med. dose ICS/ formoterol qd and prn* NA

Med-high ICS/ LABA + LAMA and SABA prn Omalizumab, anti-IL-5, IL-5R, Il4 and Il-13

High-dose ICS/ LABA + OCS + SABA prn Omalizumab, anti-IL-5, IL-5R, Il4 and Il-13

2–3 puffs bid

4–6 puffs bid

+7puffs bid

1 puff bid 1 puff bid

2 puffs/bid 2 puffs/bid

3+puffs bid 3+puffs bid

1 puff bid

2 puffs/bid

3+puffs bid

N/A

1 puff bid

2 puffs bid

+2 puffs bid

N/A

1 puff bid

2 puffs bid

+2 puffs bid

Medium ICS+ SABA LTRA, chromones, zileuton, theophylline, prn or Low ICS/LABA, low and SABA prn ICS/LAMA, low ICS + LTRA, low ICS + Theo, low ICS + zileuton qd, and SABA prn

Med-high ICS/LABA Medium ICS/LABA or med ICS + LAMA or high ICS + LTRA and SABA prn and SABA prn or Med ICS + LTRA, med ICS + Theo, med ICS + zileuton qd, and SABA prn

Up to 12 puffs/day

Asthma treatment: 5–11 years old Intermittent Mild Preferred SABA prn Low-dose ICS qd and SABA prn Consider Beclomethasone HFA Fluticasone HFA Mometasone DPI Budesonide DPI Budesonide/ formoterol Mometasone/ formoterol Alternative N/A

Up to 8 puffs/day

Moderate

Severe

Severe+

++

Low-dose ICS/formoterol qd and prn*

Med. Dose ICS/ formoterol qd and prn*

High-dose ICS/ LABA and SABA prn Omalizumab >2 puff bid

High-dose ICS/ LABA + OCS + SABA prn Omalizumab

Immunotherapy 40 μg bid

Immunotherapy

44 μg bid 110 μg qd 90 μg bid

80/4.5 bid 50/5 bid

80 μg bid

LTRA qd, chromones, Medium ICS + SABA prn or theophylline, and Low ICS/LABA, low SABA prn ICS + LTRA, low ICS+ Theo qd, and SABA prn or

110 μg 1–2 puffs bid 220 μg 1–2 puffs qd 180 μg 1–2 puffs bid

220 μg 2 puff bid >2 puffs bid >2 puffs bid 2 puff bid 100/5 bid

Medium ICS/LABA and SABA prn or Med ICS + LTRA, or Theo qd and SABA prn

High ICS + LTRA or High ICS + Theo and SABA prn

High ICS + LTRA + OCS qd or High ICS + Theo + OCS and SABA prn

78

S. Nazario

Asthma treatment: 0–4 years old Intermittent Mild Preferred SABA prn Low-dose ICS qd and SABA prn

Fluticasone Budesonide Respules Budesonide/ formoterol* Mometasone/ formoterol* Alternative N/A

44 μg 2 puffs bid 0.25–0.5 mg/day

LTRA qd, chromones, and SABA prn

Moderate

Severe

Severe+

++

Low-dose ICS/LABA and SABA prn or medium-dose ICS and SABA prn or Low-dose ICS/formoterol qd and prn*

Medium-dose ICS/ LABA and SABA prn or Medium-dose ICS/ formoterol qd and prn* 110 μg 1 puff bid 0.5–1 mg/day

High-dose ICS/ LABA and SABA prn

High-dose ICS/ LABA + OCS + SABA prn

Med ICS + LTRA and SABA prn

High-dose ICS + LTRA and SABA prn

80/4.5 bid 50/5 bid

110 μg 2 puff bid >1 mg/day 2 puff bid 100/5 bid

High-dose ICS + LTRA + OCS qd and SABA prn

Only for 4 years old *5y/o and older

Question 3 She persisted uncontrolled with an asthma control test score of 12, reporting dyspnea on exertion and nocturnal asthma symptoms. After reviewing the inhaler technique, which comorbidities should be addressed. A. Obesity B. Atopy C. Chronic rhinitis D. Depression E. All the above Correct answer: E Asthma comorbidities are directly linked to uncontrolled asthma. Atopy is particularly prevalent in childhood-onset asthma and among non-Hispanic Blacks and Puerto Ricans. It is crucial to address the atopic component, particularly allergic rhinitis, and to recognize the “atopic march” or infants with atopic dermatitis and/or food allergies that later develop asthma and/or allergic rhinitis. Physiologically, the permeable fragile epithelia in the skin and airway have increased susceptibility to sensitization, and, thereafter, allergy development. Integrated allergy-specific environmental control measures and allergy desensitization, if indicated, are part of treatment. Obesity is another important asthma comorbidity. Increased body weight is associated with asthma exacerbations, increased asthma morbidity, and decreased response to corticosteroids and β-agonists. Chronic rhinitis is also associated with poorly controlled asthma. The nasal and pulmonary epithelia share common cellular components and mediators. Rhinitis, allergic or

chronic, and nasal polyposis are associated with asthma. Intranasal steroids and/or antihistamines are the drugs of choice. Asthma exacerbated respiratory disease (AERD) is a subgroup of patients with high asthma severity. Multidisciplinary evaluations including intranasal steroids, evaluation for surgical polypectomy, aspirin desensitization, and/or treatment with IL-4 and Il-13 antagonists or IgE antagonists are recommended. Depression and stress are also associated with increased asthma morbidity and exacerbations through the increased release of inflammatory cytokines such as IL-1, IL-6, and IL-8; stimulation of the adreno-pituitary axis leading to increased cortisol; and altered β-agonists and steroid receptor response. Education about how to identify triggers and exacerbation, having an asthma action plan, and knowing how and when to use rescue and maintenance medications is important. Influenza, pneumococcal, zoster, and coronavirus-19 immunization are also important parts of asthma treatment. Question 4 On follow-up, she still reported nocturnal awakenings every night and required a short-term bronchodilator 2–3 times/ day. Assuming adequate medication adherence and technique and attention to comorbidities, which is the best next step of therapy? A. Add leukotriene inhibitor B. Add systemic steroids C. Add long-term antimuscarinic agent D. Add theophylline E. Double ICS/LABA dose

Asthma

Correct answer: C The patient is still uncontrolled despite her treatment with an ICS/LABA.  NAEPP guidelines provide several alternative therapies, including systemic steroids, theophylline, and leukotriene inhibitors. The most recent Expert Panel recommends the addition of long-term antimuscarinic agents (LAMAs) as they are synergistic with ICS/LABA in asthma treatment instead of doubling the ICS/LABA dose. Treatment improves asthma control but does not improve quality of life or decrease exacerbations. Therapy should not be recommended in subjects with glaucoma or urinary retention. Education on the inhaler technique is essential and is intended to be used as maintenance, not as rescue therapy. LAMA can also be added to subjects uncontrolled on ICS, but LABA is still a preferred alternative, particularly among African Americans. Information provided was based on the use of tiotropium, as sufficient information on umeclidinium was not available at the time of the publication. No recommendation was made for subjects younger than 12 years of age. Question 5 Four weeks after the initial evaluation, the patient is still using albuterol 2–3 times per day and reported nocturnal awakenings almost every night. She was sensitized to mites and roaches, her IgE was 323, and her eosinophil count was 125 μg/ml. She weighs 80 kg. Assuming the patient is compliant with treatment, uses adequate technique, and that comorbidities are controlled, which would be the next step in therapy? A. Double dose of ICS B. Start Omalizumab C. Start Benralizumab D. Start Dupilumab E. Start Mepolizumab Correct answer: B Omalizumab is a humanized monoclonal antibody directed against the third constant region domain of immunoglobulin E (IgE). It prevents binding to the high-affinity IgE receptor (FcERI), downregulating the receptor on mast cells and basophils. It is dosed according to weight and IgE levels and is the preferred biologic for atopic subjects who have a weight and IgE level within the therapeutic range for medication. It improves the quality of life and reduces exacerbations, ED visits, and hospitalizations. Some subjects have a mild improvement in lung function. Omalizumab side effects include headaches, injection site reactions, and, in rare cases, anaphylaxis. It is recommended that subjects carry epinephrine autoinjectors and that they wait 2 h for the first 3 occasions and 30 min thereafter. The risk of anaphylaxis decreases with time. It is approved for asthma in children 6 years of age and older.

79

Doubling the dose of ICS does not improve symptom control in subjects already on ICS/LABA. Mepolizumab is a monoclonal humanized antibody directed against IL-5. It requires a minimum of 150 eosinophils per milliliter to start therapy. Among the approved monoclonals for eosinophilic asthma, mepolizumab requires the least number of eosinophils and does not require subjects to be allergic. It decreases exacerbation frequency, hospitalizations, and ED visits by half. Side effects include headaches, injection site reactions, and zoster exacerbations. It can be self-administered at home and is dosed at a fixed dose of 100  mg SC q 4  weeks. Recently, mepolizumab was approved by the Federal Drug Administration (FDA) for hypereosinophilic syndrome and eosinophilic granulomatous polyangiitis at 300 mg SC q 4 weeks. It is also approved for children 6 years of age and older. Benralizumab is a humanized monoclonal antibody directed against the IL-5 receptor. It induces the death of eosinophils by inducing antibody-dependent cytotoxicity of eosinophils by natural killer cells. It requires a minimum count of 300 eosinophils/dl. Unlike mepolizumab, it leads to a prompt depletion of eosinophil counts, allowing dosing every 2  months after the first 3 monthly injections. Side effects include local injection site reactions. Dupilumab is a human monoclonal antibody against the common alpha chain of the IL-4 and IL-13 receptors. It is approved for steroid-dependent asthma, nasal polyposis, severe persistent asthma, and atopic dermatitis. It is dosed every 2 weeks at 200 or 300 mg SC depending on the indication and the subject’s age. It is approved for patients aged 12 and up with asthma. Side effects include conjunctivitis, mostly described in the setting of atopic dermatitis and eosinophilia. Biologic therapies require prior evaluation to exclude parasitic diseases, particularly in places where parasitic infections are prevalent. Question 6 Which is a long-term pulmonary change seen in asthma? A. Epithelial fibrosis B. Decreased airway vasculature C. Decreased smooth muscle mass D. Mucus hypersecretion E. Decreased goblet cells Correct answer: D Asthma induces structural changes in the airway called airway remodeling. These changes include: • Airway epithelial thickening affecting small and large airways. The epithelia suffer damage and become hyperplastic with sloughing and the formation of mucus plugs.

80

• Thickening of the reticular basement membrane due to subepithelial fibrosis. • Increased airway smooth muscle due to hypertrophy or hyperplasia. • Mucus hypersecretion due to mucous gland hyperplasia and increased goblet cells. • Increased neovascularization. These anatomic changes lead to functional changes reflected by loss of airway elasticity and increased stiffness, leading to breaking of alveolar septa and further bronchoconstriction. Childhood-onset asthma may lead to impaired lung function in adults, particularly among those with low baseline FEV1; severe, persistent, early-onset symptoms; those with lower bronchodilator response; and increased airway hyperreactivity. Adult-onset asthma poor lung function also correlated with long duration, severity, frequent exacerbations, bronchial hyperreactivity, smoking, continued exposure to allergens, and elevated IgE and eosinophil count. Case 2 John, a 3 y/o boy, has frequent wheezing episodes and attends daycare. Question 7 Which are risk factors for persistent wheezing in children? A. Female sex B. Parents with asthma C. Lack of atopy D. Parasitic infection E. Brother with asthma Correct answer: B Several studies have evaluated predictive risk factors for asthma among children who wheeze. The best known, the Tucson Longitudinal study, evaluated newborns initially through 3 years of age and later expanded to 6- and 18-years old. Almost half of the children never wheezed, 20% wheezed as an infant and resolved by 6 years (transient), 14% had persistent wheezing since infancy (persistent), and 15% started wheezing between 3 and 6 years of age (late wheezers). Transient wheezers had no association with atopy but rather with maternal smoking, daycare attendance, or older siblings at school. Atopic persistent wheezers were more likely to continue wheezing through adolescence and had higher total IgE, atopic dermatitis, parents with asthma, and food allergies. Their lung function, which was originally normal, worsened over time. Several longitudinal or cross-sectional studies confirmed the Tucson study and identified risk factors for asthma. These include:

S. Nazario

• Atopy: multiallergen, Alternaria. • Boys have a greater risk than girls. • Decreased lung function was linked to prolonged or early wheezing. • Upper respiratory tract infections by virus: Rhinovirus (particularly Rhinovirus C), Respiratory Syncytial Virus, Influenza, Parainfluenza, Metapneumovirus; or bacteria: Streptococcus pneumonia, Hemophilus, Moraxella. The Asthma Predictive Index (API), derived from the Tucson Longitudinal Asthma Study, intends to predict the risk of asthma in the future among 3 y/o children who had a history of recurrent wheezing episodes. Parental asthma or physician-diagnosed atopic dermatitis are required for a positive score, or 2 minor criteria, including rhinitis, wheezing besides cold, or eosinophil count of 4%. The modified API increased the number of wheezing episodes to 4 and added aeroallergen sensitization as a major criterion. The minor criteria of allergic rhinitis were substituted for food sensitization. The API score has a large negative predictive value; thus, negative results predict that wheezing will resolve over time. Question 8 Which of the following is an inciting factor for asthma? A. Early-onset cat exposure B. Coronavirus infection C. Increased interferon-gamma production at birth D. Exposure to Rhinovirus infection E. High birth weight Correct answer: D Immune dysregulation at birth with Type 2 (T2) predominance over Type 1 response increases the risk of lower respiratory tract infections, wheezing, allergen sensitization, and asthma onset. It is unclear whether this immune imbalance is primary or secondary to other epigenetic or confounding factors, including pollution, stress, obesity, low birth weight, allergen exposure, or dietary factors, which could impinge on the immune response. Exposure to traffic pollution, particularly nitric oxide and dioxide, and particulate matter increase the risk of asthma inception and could be an important explanation for the increased asthma prevalence in urban dwellers. Allergic sensitization, particularly to cockroaches and rodents, is associated with asthma morbidity, particularly among inner-city children. A dose-response of dust mite exposure to sensitization has been shown, although the association with asthma morbidity is ambivalent. The associations between cat and dog exposure, sensitization, and allergies are complex. Studies suggest that early exposure to pets among children protects against asthma development.

Asthma

Viral infections in infancy can also affect the risk of asthma inception, particularly with Rhinovirus, by inducing a T2 immune shift (Answer D). Stress and depression in caregivers have been associated with asthma inception and morbidity. Obesity is also associated with increased asthma prevalence in children and adults but not in newborns. Preterm birth is associated with asthma. Question 9 John developed increased nasal rhinorrhea and a low-grade fever. The next day he started wheezing. Which would be the best treatment recommendation? F. Amoxicillin G. Albuterol inhalations q 4 h H. Prelone for 4 days I. Double ICS dose J. Inhaled budesonide 1 mg bid for 7 days Correct answer: J The expert panel revision addresses the treatment of infants and toddlers 4 years of age or younger who have had three or more wheezing episodes in their life or two episodes of wheezing in the last year, associated with respiratory tract infections, who were asymptomatic the rest of the time. They recommend the use of budesonide inhalation suspension bid for 7  days at the onset of respiratory infection symptoms, along with as-needed SABAs as rescue therapy. This treatment is started at home by the caregivers, following the recommendations in a written action plan described by the provider. Inhaled steroids at the onset of respiratory tract infection decrease the need for systemic steroids and have better outcomes than SABAs alone. In children 4–11 years of age, increasing the inhaled corticosteroid dose, even five times, did not reduce exacerbations or improve quality of life scores. Most exacerbations in infants are viral-induced, and antibiotics are not indicated. Oral corticosteroids can have a negative effect on growth, and inhaled corticosteroids are preferably used. Question 10 What is the role of fractional exhaled nitric oxide (FeNO) in the evaluation of asthma? A. Can be used in isolation to assess asthma control B. Predicts future exacerbations C. Assesses asthma exacerbation severity D. It is an adjunct to history, physical exam, and spirometry in asthma evaluation E. Levels less than 25 parts per billion are consistent with T2 inflammation and asthma diagnosis

81

Correct answer: D FeNO is a noninvasive biomarker that measures T2 inflammation. It is a safe procedure recommended as an adjunct to asthma monitoring in subjects 5 years of age and older. The value of FeNO measures in younger children has not yet been established. The FeNO value increases in allergic conditions such as allergic rhinitis and should be interpreted with caution in this setting. Levels less than 25  ppb if older than 12 y/o or   50  ppb or  >  35  ppb in children 5–12 years of age are consistent with active T2 inflammation and response to steroids and support a diagnosis of asthma. Levels in the intermediate range of 25–50 or 20–35 in children are nondiagnostic. FeNO measurement is an adjunct to asthma diagnosis and by itself does not establish the diagnosis. It predicts response to corticosteroids and can be used to monitor asthma in patients on ICS, ICS/LABA, montelukast, or omalizumab. FeNO monitoring decreases asthma exacerbations but does neither predict future exacerbations unless it is used every 2–3 months, nor does it predict asthma severity. It is not intended to measure adherence.

2 Conclusion The updated asthma management builds on the NAEPP guidelines, incorporating recommendations to use single maintenance and reliever therapy on mild and moderate asthmatics older than 4 years of age. It updates the use of long-­ acting muscarinic agents and other biologics that have been approved for T2 inflammation. This publication makes recommendations on the role of FeNO as an adjunct for the diagnosis of asthma and supports the role of subcutaneous immunotherapy in atopic controlled asthmatics and comprehensive environmental control measures geared to specific sensitization of subjects with asthma. Incorporating the asthma guideline recommendations is intended to improve asthma care and quality of life and reduce asthma morbidity and mortality.

Bibliography 1. Patel SJ, Teach SJ. Asthma. Pediatr Rev. 2019;40(11):549–65. 2. Nhlbi. National Heart, Lung, and Blood Institute National Asthma Education and Prevention Program Expert Panel Report 3: Guidelines for the Diagnosis and Management of Asthma Full Report 2007. 2007.

82

S. Nazario

3. The Global Initiative for Asthma is supported by unrestricted edu6. Barnthouse M, Jones BL. The impact of environmental chronic and cational grants from: Visit the GINA website at www.ginaasthma. toxic stress on asthma. Clin Rev Allergy Immunol. 2019;57:427–38. org. Global strategy for asthma management and prevention Visit 7. Middleton’s Allergy: Principles and Practice- ClinicalKey the GINA website at www.ginasthma.org © 2018 Global Initiative [Internet]. https://datubazes.lanet.lv:5250/#!/browse/book/3-­s2.0-­ f [Internet] 2014. www.ginasthma.org C20161002419 4. Cloutier MM, Baptist AP, Blake KV, Brooks EG, Bryant-­ 8. O’Byrne P, Fabbri LM, Pavord ID, Papi A, Petruzzelli S, Lange Stephens T, DiMango E, et  al. 2020 Focused Updates to the P. Asthma progression and mortality: the role of inhaled corticosteAsthma Management Guidelines: A Report from the National roids. Eur Respir J. 2019;54:1900491. Asthma Education and Prevention Program Coordinating 9. Khreis H, Kelly C, Tate J, Parslow R, Lucas K, Nieuwenhuijsen Committee Expert Panel Working Group. J Allergy Clin Immunol. M.  Exposure to traffic-related air pollution and risk of develop2020;146(6):1217–70. ment of childhood asthma: a systematic review and meta-analysis. 5. McGarry ME, Castellanos E, Thakur N, Oh SS, Eng C, Environ Int. 2017;100:1–31. Davis A, et  al. Obesity and bronchodilator response in black 10. Poowuttikul P, Saini S, Seth D. Inner-city asthma in children. Clin and hispanic children and adolescents with asthma. Chest. Rev Allergy Immunol. 2019;56:248–68. 2015;147(6):1591–8.

Cough and Allergic Diseases Satoshi Yoshida

Abbreviations AC BHR CF CVA GERD ICS LABA NAEB PC20

PNDS SABA UACS

Atopic cough Bronchial hyperresponsiveness Cystic fibrosis Cough variant asthma Gastroesophageal reflux disease Inhaled corticosteroids Long-acting inhaled β2 agonists Non-asthmatic eosinophilic bronchitis Load concentration (mg/mL) that reduces the amount of forced exhalation for 1  s before and after the test by 20% Postnasal drip syndrome Short-acting inhaled β2 agonists Upper airway cough syndrome

1 Introduction Cough is a very common complaint for which patients seek medical attention. A multicenter study by general practitioners has also reported that cough was the most frequent reason (11.7% of all) that made patients visit clinics. Clinical findings frequently indicate a specific cause. Cough is both an important physiological reflex protecting the airways, and a frequent complaint associated with virtually all pulmonary and several extrapulmonary diseases. Cough is also a contributing factor in the spreading of infectious disease. Strictly speaking, even if you cough in one word, there are various forms of infection transmission depending on the pathogen. Cough commonly forms “droplets” that contain a variety of pathogens that cause inhaled infections. And they

S. Yoshida (*) Department of Continuing Medical Education, Harvard Medical School, Boston, MA, USA

form smaller “microaerosols” that cause infections similar to airborne infections. For this reason, cough can contribute to the spread of various infectious diseases. Diseases that cause droplet infections include, for example, mycoplasma and pertussis, which are frequently transmitted to young people and children. COVID-19 is a disease that is spread by not only droplet but also microaerosols. In addition, tubercle bacilli are among those that are transmitted by the more infectious “airborne infection.” Acute cough due to the common cold is one of the most frequent causes of primary care consultations. Cough is a reflex that helps clear the airways of secretions, protects the airway from foreign body aspiration, and can be the manifesting symptom of a disease. Reflexes are characterized by complexity and plasticity and are caused by physical and chemical stimuli. Stimulatory and C-fiber receptors are activated in the respiratory tract, pleura, pericardium, and esophagus. The impulse is then transmitted to the brainstem cough generator circuit via the vagus nerves. There is also a connection to the cortex, allowing voluntary control of both eliciting and, to a limited degree, inhibiting cough. Efferent innervations reach the effector muscles (diaphragm, abdominal, intercostals, back, and muscles of the larynx and upper airway). Mucociliary clearance is the primary means of clearing the bronchial system. To a certain degree, cough can compensate for impaired mucociliary clearance (e.g., that caused by the effects of smoking). If mucociliary clearance is overwhelmed by aspiration, an intact cough reflex protects the lungs effectively. However, an impaired cough reflex, e.g., after stroke, results in life-threatening aspiration pneumonia. The clearing competence of the cough reflex depends on several conditions: obstruction of the airways, bronchial collapsibility, lung volumes, respiratory muscle and laryngeal function, and the amount and viscosity of the mucus. Cough is productive (wet) if the amount of the daily expectoration is ≥30  mL (two tablespoons’ worth). The phlegm can be mucous, serous, purulent, or bloody. Bronchial casts can also be produced. The cough reflex arc consists of five parts: (1) cough receptors; (2) afferent nerves; (3) brainstem

© The Author(s), under exclusive license to Springer Nature Switzerland AG 2022 M. Mahmoudi (ed.), Absolute Allergy and Immunology Board Review, https://doi.org/10.1007/978-3-031-12867-7_8

83

S. Yoshida

84

cough generator circuit; (4) efferent nerves; and (5) effector organs including muscles. Hypersensitivity of the cough nociceptors elicits pathological cough. Numerous respiratory and other diseases cause cough nerve hypersensitivity and thus produce cough. In a considerable number of clinical cases however only cough nerve sensitivity is affected without another “specific” cause. Such patients are suffering from idiopathic cough. Chronic cough is defined as symptoms lasting longer than 8  weeks, whereas acute cough lasts less than 3  weeks and subacute cough from 3 to 8  weeks. When persistent and excessive, cough can seriously impair quality of life and lead to vomiting, muscle pain, rib fractures, urinary incontinence, tiredness, syncope, and depression. It also has psychosocial effects, such as embarrassment and negative impact on social interactions. Etiologies of Chronic Cough in Adults and Children are shown in (Table 1). Table 1  Etiologies of chronic cough in adults and children Adults Most common Protracted bacterial bronchitis (i.e., postinfection cough)  Environmental exposures  Smoking status  Airway hyperresponsiveness  Upper airway cough syndrome  Postnasal drip syndrome Synobronchial syndrome  Upper airway intoxication  Chronic pharyngitis  Chronic laryngitis  Bronchial asthma  Cough variant asthma  Atopic cough Non-asthmatic eosinophilic bronchitis  Chronic obstructive pulmonary disease  Pneumothorax  Gastroesophageal disease  Laryngopharyngeal reflux disease  Angiotensin-converting enzyme inhibitor use Less common  Chronic aspiration  Bronchiectasis  Obstructive sleep apnea  Pertussis  Pulmonary embolism  Postinfectious bronchospasm  Sleep apnea Least common  Reactive airways dysfunction syndrome  Irritant-induced asthma  Arteriovenous malformation  Bronchiolitis  Bronchogenic carcinoma  Chronic interstitial lung disease  Irritation of external auditory canal

Table 1 (continued)  Persistent pneumonia  Tuberculosis  Sarcoidosis  Vocal cord dysfunction  Somatic cough syndrome Children Most common  Postinfectious cough  Aspiration  Airway hyperresponsiveness  Upper airway cough syndrome (in children older than 6 years)  Postnasal drip syndrome Synobronchial syndrome  Bronchial asthma (cough variant asthma)  Cystic fibrosis Less common  Protracted bacterial bronchitis (i.e., postinfection cough)  Environmental triggers  Foreign body (in younger children)  Gastroesophageal reflux disease  Pertussis  Postinfectious bronchospasm Non-cystic fibrosis bronchiectasis Tracheobronchomalacia Least common  Chronic aspiration  Congenital abnormality  Immunodeficiency  Primary ciliary dyskinesia Somatic cough syndrome  Tourette syndrome/tic  Isolated orphan airways disease

Case Presentation 1 36-year-old male. Four weeks ago, he visited a primary care outpatient clinic because he had symptoms such as fever of 100.0F (37.8  °C), headache, sore throat, runny nose, cough, and sputum. He was prescribed azithromycin 500  mg/day for 3  days after the examination. The fever went down and symptoms such as runny nose, headache, and sore throat disappeared within a week. However, he went to an outpatient clinic because his coughing did not stop and he could not sleep at night, especially because he coughed at bedtime. At the time of consultation, he had a body temperature of 96.8  °F (36.1  °C). He breathes 14 breaths/minute. Blood pressure 134/76, Sp02 98%, COVID-19 and influenza antigen rapid test were both negative. On auscultation, he had no abnormal breath sounds and did not hear any secondary noise. There was no abnormality in the findings on the chest X-ray. In the spirometry test, FVC: 94%, FEV1.0: 86%. He usually does not have paroxysmal coughing, wheezing, or dyspnea. There is no heartburn or coughing after eating. He has no dyspnea or coughing after exercise, no history of sinusitis, and no history of smoking.

Cough and Allergic Diseases

Question 1 What is the most probable diagnosis for this patient? A. Exacerbation of bronchial asthma after infection B. Prolonged cough after infection C. Sinus bronchial syndrome D. Chronic bronchitis E. Bronchiolitis Answer: B Prolonged cough after infection. Cough as a symptom is attributed to distinct diseases and is categorized as either acute (lasting ≤8 weeks) or chronic (lasting >8  weeks). Of course, these limits are arbitrary. Acute cough (due to common cold) usually lasts only 2–3  weeks. Common cold is the most common cause of cough and usually subsides spontaneously, in otherwise healthy persons, after 2–3 weeks. Enhancing the cough nerve sensitivity following diseases can cause acute cough. Various acute infections, (e.g., Mycoplasma pneumoniae and adenoviruses) however can elicit cough lasting more than 8 weeks; for example, Bordetella pertussis can cause cough lasting around 3  months. In otherwise healthy individuals, acute infections of the upper and/or lower airways, the most common cause of cough, are self-limiting. Medical history and physical examination are usually sufficient in the diagnosis, and over-the-counter (OTC, i.e., nonprescription) remedies for the treatment. However, a few special circumstances require an immediate, full diagnosis of acute cough. In contrast, immediate diagnostic workup is essential in all patients presenting with chronic (>8  weeks duration) cough; a chest radiograph and lung function test should be performed immediately. This is consistent with the recommendations in all published guidelines on cough. If the chest radiograph proves inconclusive, the lung function test is unremarkable and cough is the only presenting symptom, it will always be difficult to establish the diagnosis. A dry cough is annoying and destructive, but it has some common root causes, mainly allergies such as allergic rhinitis, colds, and the flu. Many guidelines define chronic cough as a cough that lasts for more than 8  weeks. As a result, chronic cough can lead to poor quality of life. The underlying cough reflex is the same whether the patient suffers from a cough associated with an allergy or infection. A cough begins when a special nerve that ends in the airways detects some disorder and informs the brain that something is happening in the respiratory system. When the brain processes enough signals to determine that something may be wrong, the brain sends back chemical signals to start coughing. Its lung-brain communication remains the same no matter what the cause of the cough. Allergy-related cough can be caused by inflammation of the airways caused by an inappropriate or overly sensitive immune response to relatively harmless

85

particles (such as pollen), whereas cold or flu-related inflammation is caused by a viral infection. If the cough is due to an allergy rather than a cold or flu, there are some differences in the timing of symptoms. If the allergen is present in certain weather conditions and not in other weather conditions, an allergy-related cough may begin in response to seasonal changes. Patients may also notice more coughing in some settings, but not in others. For example, a patient may feel good in the office but begins to cough at night due to household allergens such as animal dander and smoke. Allergy-­ related coughs can affect you for weeks or months at a time, and symptoms can vary in intensity from day to day. In the case of a cold or flu cough, the symptoms worsen in a few days and gradually relieve as they improve. Patients may notice some variation in timing symptoms such as a more severe night cough than during the day but may not have the same variability that is seen with allergic coughing. Patients may be able to decipher the cough by looking at the other symptoms they experience with the cough. Colds, flu, and allergies share common symptoms, such as sneezing, stuffy nose, and runny nose, but they usually differ in other ways. Allergies can cause itching of the eyes, palate, itching of the throat, and dark circles under the eyes. These bears, also known as allergy shiners, are associated with chronic untreated allergies. If the cough is due to a cold or flu, the patient may experience fatigue, pain, fever, and an upset stomach. Cough can change from dry to moist mucus-filled and vice versa as the patient develops from a cold or flu and recovers. Question 2 What is the most important pathophysiology of this patient? A. Eosinophilic inflammation B. Persistent purulent inflammation due to infection C. Sensitization to antigen D. Exposure to irritant chemicals E. Bronchial hyperresponsiveness Answer: E Eosinophilic inflammation is important as the pathophysiology of allergic inflammatory diseases of the airways, such as bronchial asthma, and it is important to distinguish it from purulent inflammation caused by neutrophils that control biodefense immunity. Also, lymphocytes are involved in two pathophysiologies, allergic inflammation and purulent inflammation. In this case, sensitization to the antigen is not related to the condition, as this patient does not have an allergic cough. When coughing persists after airway infection, it is characterized by persistent coughing even after the inflammation of the respiratory tract has subsided. “Bronchial sensitivity” is a term that refers to the hypersensitivity of the airways caused by specific antigen stimulation. The respon-

86

siveness of the airways to nonspecific stimuli such as exhaust fumes, cigarette smoke, or air pollution is called “bronchial hyperresponsiveness.” Accurate understanding of the difference in meaning between these two terms, “airway responsiveness” and “airway hyperresponsiveness,” can lead to airway infections such as acute bronchitis, or different airway inflammations such as bronchial asthma and chronic obstructive pulmonary disease (COPD). It is very important to understand the difference in pathophysiology of diseases and to make a differential diagnosis of various respiratory tract diseases. Question 3 Which test is appropriate for assessing BHR? A. Allergen-specific IgE test B. Methacholine inhalation test C. Kveim test D. Drug-induced lymphocyte stimulation test E. Pre- and post-ß2-agonist inhalation spirometry test Answer: B BHR can be assessed with a bronchial challenge test. Experimentally, BHR is expressed by the leftward shift of the concentration-response curves following aerosol administration of histamine or methacholine. These chemicals trigger bronchospasm in normal individuals as well, but people with BHR have a lower threshold. In asthmatic patients, BHR results in a significant decrease in the provocative concentration of histamine or methacholine causing a 20% decrease in forced expiratory volume in 1 s (PC20). The cut-­ off value of PC20, which indicates increased BHR, is 8 mg/ mL for both histamine and methacholine. BHR referred to as airway hyperresponsiveness is the occurrence of excessive bronchoconstriction in response to a variety of inhaled stimuli, both chemical and physical. BHR is the expression of an exaggerated bronchopulmonary response associated with airway inflammation, involving vascular alterations, increase in bronchial secretions, recruitment, and activation of inflammatory cells. Widely used as an objective measure of fluctuating airflow, BHR is considered as a hallmark, a defining feature and most characteristic clinical feature of asthma. However, BHR is also found in a spectrum of other lung diseases from COPD to cystic fibrosis. It is often detected in atopic individuals, in patients with rhinitis but without pulmonary symptoms, in smokers and ex-smokers, smoking history of 20 pack-years, or in smokers older than 45 years) that suggest a serious underlying cause of cough. Also, after respiratory infections and following acute inhalation exposure to irritant chemicals. It is also found in asymptomatic nonsmoking members of the general population. From a clinical perspective, BHR testing has played an important role in the diagnosis of airway diseases. It can serve as a

S. Yoshida

predictor of future lung disease and disease progression, and it has been shown to be an independent risk factor for irreversible loss of lung function in patients with and without pulmonary symptoms. In addition, the effect of treatments on BHR provides a practicable measure for titrating medication doses in asthmatics and others, and from the perspective of pathophysiology, study of BHR provides insights into the capacity of the normal lung to control airway function and the pathological mechanisms that lead to airway dysregulation and disease. BHR is an important feature of asthma and is characterized by a nonspecific exaggerated response to bronchoconstrictor agents, such as histamine and acetylcholine. As mentioned above, BHR is expressed by the leftward shift of the concentration-response curves following aerosol administration of histamine or methacholine. These chemicals trigger bronchospasm in normal individuals as well, but people with BHR have a lower threshold. In asthmatic patients, BHR results in a significant decrease in the provocative concentration of histamine or methacholine causing a 20% decrease in PC20. BHR is a hallmark of asthma but also occurs frequently in people suffering from COPD. In the Lung Health Study, BHR was present in approximately two-­ thirds of patients with nonsevere COPD, and this predicted lung function decline independently of other factors. Kveimtest is a specific intradermal reaction for diagnosing sarcoidosis. Pre- and post-ß2-agonist inhalation spirometry is a test to evaluate airway reversibility. Question 4 What is this patient’s first-line drug? A. Inhaled corticosteroids B. Inhaled anticholinergic agent C. Short-duration antitussives D. Antileukotrienes E. Inhaled long-acting β2 agonist Answer: C It is probable that the respiratory tract infection did not prolong because the respiratory tract infection developed 4 weeks ago, and azithromycin was effective. However, it is estimated that airway hyperresponsiveness is high in about 10% of the population. That is, the airways are highly responsive to nonspecific stimuli, and the respiratory tract infection can damage the airway mucosal epithelium due to severe cough, resulting in prolonged cough and sometimes sputum over several months. Patients often go to a medical institution because their cough does not go away even after taking over-the-counter antitussives and they cannot sleep due to severe coughing. In general, abuse of antitussives interferes with sputum discharge and may cause atelectasis, so its instant use during coughing is not recommended, especially in children. However, long-term coughing after an airway

Cough and Allergic Diseases

infection can damage the walls of the bronchial mucosa, which can lead to more severe and prolonged coughing. Persistent cough causes multiple damages to the airway mucosa, which can lead to further coughing, leading to prolonged airway mucosal damage, leading to a vicious cycle. Furthermore, there is concern about the possibility of recurrent airway inflammation due to secondary infection of the damaged airway mucosa. Especially, patients with BHR are expected to treat persistent cough by promoting healing of airway mucosal damage. Antitussives may be indicated for prolonged coughing that may interfere with daytime activity, or insomnia for more than 4 weeks. It aims to suppress the severe cough reflex by keeping the bronchial wall at rest and repairing damaged airway mucosa. The appropriate treatment is to prescribe antitussives and expectorants that stop coughing, promote bronchial mucosal repair, and promote sputum output for the minimum required period. Repeated and prolonged damage to the airway mucosa due to prolonged cough causes the damaged bronchial wall to become fibrotic through substrate formation, resulting in remodeling of the bronchi and bronchioles. The development of airway remodeling due to mechanical stress to the airways exerted by long-standing coughing may result in a vicious cycle of cough and remodeling, and imply the importance of control of coughing. Question 5 What is this patient’s second-line drug? A. Inhaled corticosteroids B. Inhaled anticholinergic agent C. Short-duration antitussives D. Antileukotrienes E. Inhaled long-acting β2 agonist Answer: A Inhaled corticosteroids may relieve inflammation, but their anti-inflammatory effect on persistent severe airway inflammation and damage to the airway mucosal epithelium during the acute phase is inadequate. If necessary, concomitant use of expectorants and short-term cough suppressants may also be effective in relieving acute symptoms. In addition, if the respiratory tract infection persists, such as yellow sputum being excreted at the first diagnosis, it may be necessary to consider antibiotics such as azithromycin. Even if these treatments cure bronchial inflammation, increased airway hypersensitivity can prolong coughing. That is the clinical course of this case. If cough persists after the airway infection has completely healed, short-term administration of cough medications to rest the bronchi to promote repair of the damaged airway mucosa, and inhaled corticosteroids for the treatment of potentially related persistent inflammation may be effective for treating prolonged and complicated cough.

87

Case Presentation 2 28-year-old female. She had a history of childhood asthma until she was under the age of five, but she has never had an asthma attack since she was an adult. She has been allergic to cedar pollen for 15 years and has symptoms such as runny nose, nasal congestion, sneezing, eye congestion, tearing, and itching every year from late January to mid-April. She went to my family physician’s office every year. From late January, her symptoms, such as runny nose, nasal congestion, and sneezing, as well as coughing, caused the patient to purchase and take a common cold medicine at a drugstore. As all her symptoms disappeared quickly, she self-diagnosed that she had acute upper respiratory tract inflammation and decided to continue taking her common cold medicine for some time. While taking the cold medicine, the symptoms disappeared, but when the medicine was stopped after 4 weeks or more after the onset, symptoms such as runny nose, nasal congestion, sneezing, coughing, and redness in the eyes recurred. It was also accompanied by hyperemia and itching. Her family doctor referred her to a respiratory department, especially because she had a severe dry cough without sputum. According to her findings at the time of her visit, she had a body temperature of 96.8F (36.0  °C). She breathes 16 breaths/minute. She had a blood pressure of 122/70, Sp02 of 95%, and a negative COVID-19 rapid antigen test. On auscultation, she had no abnormal breath sounds and she did not hear the second respiratory sounds. Her chest X-ray findings showed no abnormalities. Her spirometry test showed FVC: 102% and FEV1.0: 82%. The FEV1.0 improvement test with short-­acting bronchodilators (SABA) was negative (3  weeks after an acute, often viral airway infection and resolves after 3%) +

+

+

±

±





+

±





±

++

CVA cough variant asthma, AC atopic cough, NAEB non-asthmatic eosinophilic bronchitis, AHR airway hyperresponsiveness, SABA short-acting beta agonist, LABA long-acting beta agonist, SOB shortness of breath a  Defined by the presence of at least one positive serum-specific IgE or skin test response to common aeroallergens

Cough and Allergic Diseases

to the emergency walk-in clinic. Her COVID-19 PCR test was negative.

91

intestines (small intestine), so you cannot get the nutrients. Celiac disease can cause a variety of symptoms, including diarrhea, abdominal pain, and bloating. It is not associated Question 11 with symptoms such as respiratory tract infections. What are the most likely illnesses in this patient? Hypersensitivity pneumonitis is an immune system disorder in which the lungs become inflamed as an allergic reac A. Charg-Strauss syndrome tion to inhaled microorganisms, animal and plant proteins or B. Cystic fibrosis chemicals. It may develop with changes in the environment, C. Celiac diseases but there are no recurrent respiratory tract infections or D. Hypersensitivity pneumonitis genetic predisposition. E. Diffuse panbronchiolitis Diffuse panbronchiolitis (DPB) is an idiopathic inflammatory disease that is well recognized in Japan and primarily Answer: B affects respiratory bronchioles, causing progressive purulent Cystic fibrosis (CF) is an inherited disease of the exocrine and severe obstructive respiratory disease because it is one of glands affecting primarily the gastrointestinal and respira- the diseases that require differential diagnosis from cystic tory systems. It leads to chronic lung disease, exocrine pan- fibrosis and is considered to be a type of chronic bronchitis creatic insufficiency, hepatobiliary disease, and abnormally with multiple granulomatosis formation. If left untreated, high sweat electrolytes. CF is the most common life-­ DPB progresses to bronchiectasis, respiratory failure, and threatening genetic disease in the white population. In the death. US, it occurs in about 1/3300 white births, 1/15,300 black births, and 1/32,000 Asian American births. It is rarely seen Question 12 in Asians. In the case of mixed-race children, such as this Which is the most frequent symptom of this disease? patient, the frequency of occurrence is different, and in the case of mixed-race with a race with a low incidence, such as A. Cleft lip palate Japanese, the problem is that the diagnosis rate is low. B. Autism spectrum Approximately 50% of patients with cystic fibrosis are adults C. Hiatal hernia in the United States because advances in treatment have D. Nasal polyps increased the life expectancy of people with cystic fibrosis. E. Funnel chest The incidence of cystic fibrosis is similar in boys and girls. Treatment is supportive through aggressive multidisciplinary Answer: D care along with small-molecule correctors and potentiators Fifty percent of patients not diagnosed through newborn targeting the cystic fibrosis transmembrane conductance reg- screening present with pulmonary manifestations, often ulator protein defect. Because of improved treatment and life beginning in infancy. Recurrent or chronic infections maniexpectancy, about 54% of patients in the US with CF are fested by cough, sputum production, and wheezing are adults. CF is carried as an autosomal recessive trait by about common. Cough is the most troublesome complaint, often 3% of the white population. Although the lungs are generally accompanied by sputum, gagging, vomiting, and disturbed histologically normal at birth, most patients develop pulmo- sleep. Intercostal retractions, use of accessory muscles for nary disease beginning in infancy or early childhood. Mucus respiration, a barrel-chest deformity, digital clubbing, cyaplugging and chronic bacterial infection, accompanied by a nosis, and a declining tolerance for exercise occur with dispronounced inflammatory response, damage the airways, ease progression. Upper respiratory tract involvement ultimately leading to bronchiectasis and respiratory insuffi- includes nasal polyposis and chronic or recurrent ciency. The course is characterized by episodic exacerba- rhinosinusitis. tions with infection and progressive decline in pulmonary function. Question 13 Churg-Strauss syndrome is a disorder characterized by Which is correct for this patient? inflammation of blood vessels. This inflammation limits blood flow to organs and tissues and can sometimes cause A. Decreased airway mucus secretion permanent damage. This condition is also known as eosino- B. Increased sweating philic granulomatosis (EGPA) with polyangiitis. The most C. Present with chronic diarrhea common symptom of Churg-Strauss syndrome is bronchial D. Exocrine pancreatic function is normal asthma. E. Impaired glucose tolerance Celiac disease is a condition in which the immune system attacks your tissues when you eat gluten. This damages your Answer: A

92

S. Yoshida

In typical cases, meconium ileus often occurs shortly after birth. After that, poor digestion and malabsorption due to exocrine pancreatic insufficiency occurred, and respiratory tract infections were repeated, resulting in respiratory failure. The reabsorption of chloride ions in the sweat glands is impaired, resulting in high salinity in sweat. Organs are damaged and their severity vary, but in some patients only a single organ is damaged. Meconium ileus is found in 40–50% of CF patients. The highly viscous mucus impedes meconium excretion and causes impaired passage at the terminal ileum. Respiratory symptoms are found in almost all CF patients. After birth, highly viscous mucus accumulates in the bronchioles, and when pathogenic bacteria settle, bronchiolitis and bronchitis are repeated, resulting in respiratory failure. It causes purulent sputum production, coughing, and dyspnea. It is characterized by persistent infection with Pseudomonas aeruginosa. Exocrine pancreatic insufficiency is found in 80–85% of CF patients. The small pancreatic duct is occluded by an acidic secretion with a high protein concentration, and the pancreatic parenchyma gradually falls off. Changes begin in the womb, typically around the age of two with exocrine pancreatic insufficiency, resulting in steatorrhea, malnutrition, and underweight. Imaging findings often present with pancreatic atrophy or fat replacement. Cholestasis-type cirrhosis is found in 20–25% of CF patients. Question 14 Which is the respiratory complication of this disease? A. B. C. D. E.

Bronchial asthma Pulmonary fibrosis Recurrent pneumothorax Bronchiectasis Lymphangioleiomyomatosis (LAM)

Answer: D Pulmonary complications include pneumothorax, nontuberculous mycobacterial infection, hemoptysis, allergic bronchopulmonary aspergillosis (ABPA), and right heart failure secondary to pulmonary hypertension. Abortive forms can manifest in adulthood for the first time through cough, bronchial infections, and bronchiectasis. LAM is a rare pulmonary disease characterized by low-­ grade cancer cell growth in the smooth muscle tissue of the lungs and abdomen. LAM affects almost only women around 35 years. Bronchiectasis is defined by permanent and abnormal widening of the bronchi. As bronchiectasis is an acquired disorder, its pathophysiology is commonly described as distinct phases of infection and chronic inflammation. Bronchiectasis is also characterized by mild to moderate airflow obstruction. The interaction between these phases establishes a vicious circle in which the end result is the destruction of the bronchi and the accompanying clinical

symptoms. Pulmonary damage is probably initiated by a diffuse obstruction in the small airways by abnormally thick mucus secretions. Bronchiolitis and mucopurulent plugging of the airways occur secondary to obstruction and infection. Chronic inflammation secondary to the release of proteases and proinflammatory cytokines by cells in the airways also contributes to lung injury. Airway changes are more common than parenchymal changes, and emphysema is not prominent. About 50% of patients have bronchial hyperreactivity that may respond to bronchodilators. In patients with advanced pulmonary disease, chronic hypoxemia results in muscular hypertrophy of the pulmonary arteries, pulmonary hypertension, and right ventricular hypertrophy. The lungs of most patients are colonized by pathogenic bacteria. Early in the course, Staphylococcus aureus is the most common pathogen, but as the disease progresses, Pseudomonas aeruginosa is frequently isolated. A mucoid variant of P. aeruginosa is uniquely associated with CF and results in a worse prognosis than nonmucoid P. aeruginosa. In the US, the prevalence of methicillin-resistant S. aureus (MRSA) in the respiratory tract is now about 27%. Patients who are chronically infected with MRSA have a more rapid decline in pulmonary function and lower survival rates than those who are not. Colonization with Burkholderia cepacia complex occurs in about 2.6% of patients and may be associated with more rapid pulmonary deterioration. Nontuberculous mycobacteria, including Mycobacterium avium complex and M. abscessus, are potential respiratory pathogens. Prevalence varies with age and geographic location and probably exceeds 10%. Differentiating infection from colonization can be challenging. Other common respiratory pathogens include Stenotrophomonas maltophilia, Achromobacter xylosoxidans, and Aspergillus species. Question 15 Which assay is valuable in diagnosing this disease? A. HER2 gene test B. BRCA gene test C. DaTscan D. MIBG-myocardial scintigraphy E. CFTR gene test Answer: E Diagnosis of CF is by sweat test or identification of cystic fibrosis-causing mutations in patients with a positive newborn screening test result or characteristic clinical features. CF is caused by a mutation in the CFTR (Cystic fibrosis transmembrane conductance regulator gene). The CFTR protein is the major anion channel of the luminal organs throughout the body. In CF, CFTR dysfunction impairs the transport of chloride (chloride ions) and water through the epithelium/mucosa of the airways, intestines, pancreatic ducts, bile ducts, sweat ducts, and vas deferens. As a result,

Cough and Allergic Diseases

the mucus/secretion in the lumen becomes excessively viscous, the lumen is occluded and infection is likely to occur, resulting in damage to multiple organs. There are more than 1900 genetic mutations reported so far, which vary by race and country. Even patients with the same gene mutation have different organs and severity of damage, and there are many unclear points in the mechanism of pathogenesis. The responsible gene has been localized on the long arm of chromosome 7. It encodes a membrane-associated protein called the cystic fibrosis transmembrane conductance regulator (CFTR). The most common gene mutation, F508del, occurs in about 85% of CF alleles; >2000 less common CFTR mutations have been identified. CFTR is a cyclic adenosine monophosphate (cAMP)-regulated chloride channel, regulating chloride, sodium, and bicarbonate transport across epithelial membranes. A number of additional functions are considered likely. Disease manifests only in homozygotes. Heterozygotes may show subtle abnormalities of epithelial electrolyte transport but are clinically unaffected. The CFTR mutations have been divided into six classes based on how the mutation affects the function or processing of the CFTR protein. Patients with class I, II, or III mutations are considered to have a more severe genotype that results in little or no CFTR function, whereas patients with 1 or 2 class IV, V, or VI mutations are considered to have a milder genotype that results in residual CFTR function. However, there is no strict relationship between specific mutations and disease manifestation, so clinical testing (i.e., of organ function) rather than genotyping is a better guide to prognosis. CFTR mutations can be frameshift (deletion or insertion in a DNA sequence that shifts the way a sequence is read) or nonsense (stop) mutations. Human epidermal growth factor receptor 2 (HER2) is a member of the epidermal growth factor receptor family having tyrosine kinase activity. Dimerization of the receptor results in the autophosphorylation of tyrosine residues within the cytoplasmic domain of the receptors and initiates a variety of signaling pathways leading to cell proliferation and tumorigenesis. Amplification or overexpression of HER2 occurs in approximately 15–30% of breast cancers and 10–30% of gastric/gastroesophageal cancers and serves as a prognostic and predictive biomarker. HER2 overexpression has also been seen in other cancers like ovary, endometrium, bladder, lung, colon, and head and neck. The introduction of HER2-directed therapies has dramatically influenced the outcome of patients with HER2-positive breast and gastric/ gastroesophageal cancers; however, the results have been proved disappointing in other HER2 overexpressing cancers. One of the most important risk factors for breast cancer is family history of the disease, indicating that genetic factors are important determinants of breast cancer risk. About 5–10% of breast cancers are thought to be hereditary, caused by abnormal genes passed from parent to child. A number of

93

breast cancer susceptibility genes have been identified, the most important being BRCA1 and BRCA2. However, it is estimated that all the currently known breast cancer susceptibility genes account for less than 25% of the familial aggregation of breast cancer. In this chapter, we review the evidence for other breast cancer susceptibility genes arising from twin studies, pedigree analysis, and studies of phenotypes associated with breast cancer, and the progress towards finding other breast cancer susceptibility genes through linkage and association studies. Taken together, the available evidence indicates that susceptibility to breast cancer is mediated through variants in many genes, each conferring a moderate risk of the disease. Such a model of susceptibility has implications for both risk prediction and for future gene identification studies. In 2011, the US Food and Drug Administration (FDA) approved a brain imaging test called DaTscan (DAT-SPECT: Dopamine transporter-single photon emission computed tomography) to help diagnose Parkinson’s disease (PD). For some people, DaTscan can be a useful addition to the doctor’s examination in diagnosing Parkinson’s. A DaTscan is an imaging technology that uses small amounts of a radioactive drug to help determine how much dopamine is available in a person’s brain. A SPECT scanner is used to measure the amount and location of the drug in the brain. Iodine 123 (123I) metaiodobenzylguanidine (MIBG) is the first-line functional imaging agent used in neuroblastoma imaging. MIBG uptake is seen in 90% of neuroblastomas, identifying both the primary tumor and sites of metastatic disease. The addition of single photon emission computed tomography (SPECT) and SPECT/computed tomography to 123 I-MIBG planar images can improve identification and characterization of sites of uptake. During scan interpretation, use of MIBG semiquantitative scoring systems improves description of disease extent and distribution and may be helpful in defining prognosis. Therapeutic use of MIBG labeled with iodine 131 (131I) is being investigated as part of research trials, both as a single agent and in conjunction with other therapies. 131I-MIBG therapy has been studied in patients with newly diagnosed neuroblastoma and those with relapsed disease.

2 Conclusion Treatment of cough should always seek a causal treatment. However, if this approach is not possible on the first visit, such as an acute viral respiratory infection, or if it tends to be delayed, such as tuberculosis, cough treatment tends to always be empirical. Symptomatic treatment is unavoidable, especially if the causative approach, such as an acute respiratory infection, is not possible at the first visit, or if the test tends to be delayed due to cough due to a chronic condition. By coating the cough receptors in the throat,

94

mucus removers are thought to have antitussive effects. Cough syrup, lozenges and drops, and honey share sugar as a common ingredient. Efficacy, if any, is time-limited to contact between the sugar and the receptor. This is usually 20–30 min. Systemic α-adrenaline agonists for nasal congestion are popular in the United States but are virtually unused internationally. Fixed combinations with older anticholinergic agents and centrally active antihistamines (i.e., chlorpheniramine or dexbrompheniramine) are not readily available in Europe. In addition, there is a lack of evidence of their effectiveness from randomized controlled trials. Antibiotics are only effective against cough caused by a bacterial infection characterized by purulent sputum (purulent bronchiectasis, bronchiectasis, exacerbation of COPD, purulent rhinitis, sinusitis). Antibiotics are not indicated for acute bronchitis. Inhaled and nasal corticosteroids and oral leukotriene antagonists reduce asthma, eosinophil bronchitis, postinfection cough with BHR, and rhinitis cough. Local anesthetics negate the electrophysiological activity of receptors and afferents (such as during bronchoscopy). They are increasingly used off-label in idiopathic cough and palliative medicine. Drugs that affect the central mechanism of cough (antitussives) include systemic morphine or codeine, as well as natural and synthetic derivatives (dextromethorphan, dihydrocodeine, noscapine, pentoxyverine). Some nonaddictive herbal remedies (Ribwort Plantain, Drosera rotundifolia) claim a central antitussive effect, which has not been proven in clinical studies. Opiates are recommended for effective symptomatic treatment of dry debilitating cough. They have a limited effect on the treatment of cough caused by the common cold. Expectorants are the most common drugs used for respiratory disease, such as ambroxol and N-acetylcysteine, which reduce the irritation of cough receptors by accumulating mucus by “coughing”). Efficacy is difficult to assess because there is no suitable method. Inconsistent evidence of the relative efficacy of various expectorants is present throughout the published literature. Symptomatologic use of expectorants is recommended to relieve cough in the case of mucous secretions (COPD and bronchiectasis). Many patients also report positive subjective efficacy using self-medication for acute bronchitis. Combined with phytotherapy, it can reduce the duration of acute coughing or colds. In bronchiectasis of CF, inhaled Dornase α relieves cough. Inhaled anticholinergic agents (i.e., ipratropium and tiotropium) are thought to reduce mucus production. However, their antitussive effects are inconsistent. Theophylline and β2-adrenergic agents increase mucous fimbria clearance but have no effect on coughing. Despite being a clinical routine in both hospital and outpatient care, and rehabilitation, there is insufficient evidence of the effectiveness of physiotherapy for cough. Pulmonary rehabilitation may be

S. Yoshida

indicated depending on the pathophysiology of the disease. The purpose of physical therapy is (1) For patients with a productive but ineffective cough, use effective coughing techniques to increase sputum. (2) To spontaneously suppress unproductive cough. (3) Acapella® (DHD ­ Healthcare, Wampsville, NY, USA), Flutter® (Desitin/ ScandipharmVarioRaw SA, Birmingham, AL, USA), RC Cornet® (BoniCur, East Court, UK). Quite a lot of involvement in allergy and immunology has emerged in the pathology of cough. In some cases, it has become clear that the mechanism of allergy and immunology is involved in the pathophysiology of diseases that were previously thought to be unrelated to allergy and immunology. Future progress in research should be ardently expected.

Bibliography 1. Irwin RS.  Introduction to the diagnosis and management of cough: ACCP evidence-based clinical practice guidelines. Chest. 2006;129(1 Suppl):25S–7S. 2. Irwin RS, et  al. Overview of the management of cough. Chest guideline and expert panel report. Chest. 2014;146(4):885–9. 3. Cockcroft DW, Davis BE. Mechanisms of airway hyperresponsiveness. J Allergy Clin Immunol. 2006;118:551–9. 4. Tarlo SM, et al. Diagnosis and management of work-related asthma: American College of Chest Physicians Consensus Statement. Chest. 2008;134:1S. 5. Irwin RS, et al. Chronic persistent cough in the adult: the spectrum and frequency of causes and successful outcome of specific therapy. Am Rev Respir Dis. 1981;123(4 pt 1):413–7. 6. Pratter MR.  Chronic upper airway cough syndrome secondary to rhinosinus diseases (previously referred to as postnasal drip syndrome): ACCP evidence-based clinical practice guidelines. Chest. 2006;129(1 Suppl):63S–71S. 7. Reddel HK, et al. American Thoracic Society/European Respiratory Society Task Force on Asthma Control and Exacerbations. An official American Thoracic Society/European Respiratory Society statement: asthma control and exacerbations: standardizing endpoints for clinical asthma trials and clinical practice. Am J Respir Crit Care Med. 2009;180:59–99. 8. Mukae H, et al. The Japanese respiratory society guidelines for the management of cough and sputum (digest edition). Respir Investig. 2021;59(3):270–90. 9. Morice AH, et al. The diagnosis and management of chronic cough. Eur Respir J. 2004;24:481–92. 10. Morice AH, et al. ERS guidelines on the assessment of cough. Eur Respir J. 2007;29:1256–76. 11. Wilkes J. ACCP provides updated recommendations on the management of somatic cough syndrome and tic Cough. Am Fam Physician. 2016;93:416. 12. Irwin RS, Rosen MJ, Braman SS. Cough. A comprehensive review. Arch Intern Med. 1977;137:1186–91. 13. Irwin RS, Madison JM. The diagnosis and treatment of cough. N Engl J Med. 2000;343(23):1715–21. 14. Kardos P.  Management of cough in adults. Breathe. 2010;7:122–1332. 15. Bolser DC.  Cough suppressant and pharmacologic protussive therapy: ACCP evidence-based clinical practice guidelines. Chest. 2006;129:238S–49S.

Hypersensitivity Pneumonitis Ria Gripaldo and Avanthika Thanushi Wynn

Abbreviations

1 Introduction

AEP Acute eosinophilic pneumonia ANA Antinuclear antibody ANCA Antineutrophilic cytoplasmic antibody ARDS Acute respiratory distress syndrome BAL Bronchoalveolar lavage CBC Complete blood count CEP Chronic eosinophilic pneumonia CXR Chest radiograph EGPA Eosinophilic granulomatosis and polyangiitis HP Hypersensitivity pneumonitis HR Heart rate HRCT High resolution computed tomography IgE Immunoglobulin E IgG Immunoglobulin G IL Interleukin ILD Interstitial lung disease IPF Idiopathic pulmonary fibrosis MPO Myeloperoxidase antibody MUC5B Mucin 5b NTM Nontuberculous mycobacterium OSA Obstructive sleep apnea Th1 T helper cell type 1 Th17 T helper cell type 17 Th2 T helper cell type 2 UIP Usual interstitial pneumonia

Hypersensitivity pneumonitis, previously termed extrinsic allergic alveolitis, is a complex immune-mediated syndrome that occurs in susceptible individuals who have had an occult or overt exposure to antigens small enough to reach the alveoli ( Th1 response

IL-5

IL-4, IL-13 GM-CSF

Release of cytokines, chemokines and growth factor Neutrophilic inflammation Mast cell degranulation

Further bronchial damage and airway remodeling IgE B cell

IgG

Eosinophilic inflammation

IgA

Fig. 1  Pathogenesis of ABPA—In genetically susceptible individuals and those with impaired local lung immunity, Aspergillus fumigatus spores are not cleared by alveolar macrophages. Growing (active) conidia disrupt the epithelial barrier and activate the innate immunity through antigen recognition by dendritic cells. A Th2 skewed inflammation occurs with release of cytokines that mediate eosinophilic

BRONCHIECTASIS PULMONARY FIBROSIS

inflammation and IgE production by B cells. Continued inflammation ultimately leads to neutrophilic inflammation and further bronchial damage characterized by pulmonary fibrosis and bronchiectasis. PAMP (pathogen-associated molecular pattern), PRR (pattern recognition receptor, Th1 and 2 (T helper cells type 1 and 2, respectively), IL (interleukin)

M. Majzoub and R. Joks

110

6 Radiographic Abnormalities Seen in ABPA • Transitory pulmonary infiltrates—fleeting shadows (early). • Lobar collapse. • Mucus impaction. • Bronchiectasis on HRCT (high-resolution CT scan). Bronchiectasis involving large central airways with a predilection for the upper lobes are diagnostic of ABPA. HAM: high attenuation mucoid impaction is pathognomonic for ABPA, usually seen on HRCT. Case 2 A 60-year-old man with mild persistent asthma and allergic rhinitis on maintenance allergen immunotherapy with ­ragweed and dust mite presenting to your clinic for his monthly subcutaneous injection. Patient is not feeling well today. He is coughing with the production of yellowish-brown tinged sputum. He also reports feeling more tired than usual. Symptoms have been going on for two weeks now. He is using albuterol inhaler 2–3 times daily and reports medication adherence. Patient just completed a course of oral steroids 4 weeks ago. Over the past 6 months, he was treated twice in the ED for asthma exacerbations and completed four courses of oral steroids. Fluticasone propionate/Salmeterol was added to his regimen with minimal improvement. He is on Montelukast too. Prior to that patient had 1–2 exacerbations per year usually around ragweed season treated with oral steroids. On your exam patient has scattered wheezing and oxygen saturation is 96% on room air. Given that his asthma is uncontrolled, you decided to order some tests to aid you with diagnosis. Initially, you performed a skin prick test and was reactive to Aspergillus fumigatus. Question 1 Given Aspergillus fumigatus reactivity on skin prick, your patient is less likely to have something else other than ABPA. A. True B. B- False

Answer and Explanation Correct answer is B Aspergillus fumigatus reactivity on skin prick aids but does not confirm the diagnosis of ABPA.

7 Differential Diagnosis 1. SAFS: Severe Asthma with Fungal Sensitization. Occurs exclusively in severe asthma. 2. Asthma or cystic fibrosis exacerbation secondary to bacterial or viral pneumonia. 3. Acute or chronic eosinophilic pneumonia. 4. Churg-Straus Syndrome. Question 2 What is the single test that will help you rule out ABPA? A. Nonreactive Aspergillus skin prick testing B. Normal chest radiography C. Low total serum IgE level D. Normal eosinophil count on CBC Answer and Explanation The correct answer is C

8 Diagnosis of ABPA Absence of sensitization to Aspergillus excludes ABPA. Presence of Aspergillus-specific IgE alone is insufficient for the diagnosis. Question 3 Your patient’s total IgE is 1280 IU/ml ruling out SAFS. He has asthma and skin reactivity to Aspergillus fumigatus. Which of these is required for the definitive diagnosis of ABPA? A. Absolute eosinophil count >500 on CBC B. Aspergillus fumigatus-specific IgE > 0.35 KUA/ L C. Aspergillus fumigatus-specific IgG > 27 mgA/L D. Bronchiectasis on CT scan Answer and Explanation The correct answer is B

Allergic Bronchopulmonary Aspergillosis

111

9 Diagnostic Criteria of ABPA Modified ISHAM Criteria for ABPA Diagnosis in Asthmatics (2020)

All of the following: Asthma Aspergillus Fumigatus specific IgE > 0.35 KUA/L 3- Serum Total IgE > 500 IU/ml

ABPA diagnosis in cystic fibrosis Clinical deterioration (increased cough and sputum production, wheezing, decreased pulmonary function) Total serum IgE >1000 KU/L (unless receiving systemic steroids) Elevated IgG antibodies to aspergillus (or precipitating antibodies) Sensitivity to aspergillus fumigatus (positive skin prick test or elevated serum aspergillus-specific IgE) Radiographic abnormalities: Pulmonary infiltrates, mucus plugs

10 Staging

II—Remission

Radiographic findings Homogeneous infiltrates, tree in bud findings, mucus plugs, consolidations, lobar collapse, bronchiectasis No infiltrates

Question 4 You diagnosed your patient with ABPA, what is the treatment of choice? A. B. C. D.

Inhaled corticosteroids Antifungals Systemic steroids Exposure prevention

11 Management

1. Oral Corticosteroids: • Mainstay of treatment. • Common strategy: 2  weeks of daily prednisone 0.5  mg/kg, followed by 6–8  weeks of alternate day therapy and then tapering by 5–10 mg every 2 weeks. • More aggressive approaches with higher doses of oral steroids or pulsed IV methylprednisolone have been used too. • High doses of [inhaled corticosteroids] alone have no role in the management of ABPA-S and should not be used as first-line therapy. 2. Antifungals: • Adjunctive treatment. • Recommended for steroid-dependent patients and relapses. • Benefits: –– Decreases the burden of fungal colonization. –– Significantly reduces total serum IgE, sputum eosinophils. –– Improves symptoms. –– Reduces the requirement for prolonged high-­ dose systemic corticosteroids

III— Exacerbation

As in stage I

IV—Steroid-­ dependent asthma V—End stage-fibrotic

No infiltrates Atelectasis, hyperinflation Scarring, hyperinflation, fibro-cavitary lesions

Normal or elevated

10.2 Radiological Staging Clinical features Serologic Pt meets all criteria except for central bronchiectasis ABPA-CB Serologic + central bronchiectasis ABPA-CB-­ Central bronchiectasis + pulmonary ORF fibrosis/ground glass opacities, fibro-cavitary lesions

Mucus Impaction on chest CT

Total serum IgE Elevated

Normal or elevated (less than stage I) Elevated (double that of stage II) Elevated or normal

Stage ABPA-S

Eosinophil count >500 cell/ml Bronchiectasis on chest CT

Answer and Explanation The correct answer C

10.1 Clinical Staging Stage I—Acute

2 or more of the following: Aspergillus specific lgG >27 mg A/L

Airflow obstruction Mild

Moderate Severe

11.1 Treatment

112

M. Majzoub and R. Joks

• Usual regimen: Itraconazole 200  mg twice daily for 4–6 months then tapered over the next 4–6 months. • Alternatives: voriconazole, posaconazole. 3. Biologics: • Possible benefits of omalizumab and dupilumab based on randomized control trials and studies. Question 5: You initiated oral steroids on your patient. You contact him after 2 weeks and he reports improvement in his symptoms. You schedule a follow up visit in 4 weeks. Which of these is the most sensitive indicator of remission? A. Decline in serum total IgE of 35% after 6 weeks B. Resolution of pulmonary infiltrates on HRCT after 4–8 weeks of initiation of steroid treatment C. Normalization of eosinophil count on CBC D. Nonreactive skin testing for A fumigatus Answer and Explanation The correct answer is A

• • • •

Monitoring and Follow Up The most sensitive indicator of disease progression is serial measurements of total IgE. A decline in serum total IgE of 35% is considered diagnostic of achieving remission of ABPA. Doubling of serum total IgE is considered diagnostic of relapse of ABPA, especially in CF patients. Patients are considered in remission when they remain without pulmonary infiltrates and/or eosinophilia for 6 months after oral steroid withdrawal.

How to Follow Up? • Obtain Total serum IgE every 6–8 weeks after the initiation of oral steroids and for 1 year thereafter. • Obtain chest imaging, either by CXR or HRCT after 4–8 weeks of initiation of oral steroid therapy to assess resolution of infiltration.

• Spirometry is a useful tool to objectively assess response to therapy.

Key Points • Always think of ABPA when you have a patient with uncontrolled asthma. • Diagnosis: asthma or cystic fibrosis, elevated total and Aspergillus-specific serum IgE. • Absence of Aspergillus sensitization excludes ABPA. Bronchiectasis of large airways with upper lobe involvement is characteristic of ABPA. Oral Steroids are the mainstay of treatment. Antifungals are adjunctive. • Total serums IgE: 35% decrease indicates remission. Doubling indicates relapse.

Bibliography 1. Hew M, Douglass JA, Tay TR, O’Hehir RE.  Middleton’s allergy e-book, 9th Edition - Chapter 58: Allergic bronchopulmonary aspergillosis, hypersensitivity pneumonitis, and epidemic thunderstorm asthma. Elsevier - OHCE; 2019. 2. Greenberger PA, Bush RK, Demain JG, Luong A, Slavin RG, Knutsen AP.  Allergic bronchopulmonary aspergillosis. J Allergy Clin Immunol. 2014;2(6):703–8. 3. Latgé JP, Chamilos G.  Aspergillus fumigatus and aspergillosis in 2019. Clin Microbiol Rev. 2019;33(1):e00140–18. https:// doi.org/10.1128/CMR.00140-­18. PMID: 31722890; PMCID: PMC6860006 4. Bains SN, Judson MA.  Allergic bronchopulmonary aspergillosis. Clin Chest Med. 2012;33(2):265–28. 5. Saxena P, Choudhary H, Muthu V, Sehgal IS, Dhooria S, Prasad KT, et al. Which are the optimal criteria for the diagnosis of allergic bronchopulmonary aspergillosis? A latent class analysis. J Allergy Clin Immunol Pract. 2021;9:328–35.

Food Allergy Catherine A. Popadiuk and Doerthe A. Andreae

Case 1 Four-month-old monozygotic twin girls are referred to an allergist after their pediatrician became concerned about a peanut allergy in both twins. No testing was completed by the pediatrician. Both twins were born at term, to a healthy 37-year-old mother without complications. There is no family history of atopic or allergic disease. Moderate eczema has been noted in both children since the age of 2 months. Their parents have been controlling flares with topical steroids that are used about twice weekly in addition to daily lukewarm baths and application of a thick moisturizing cream after bathing. About 6 weeks prior to the visit with the allergist, one twin developed hives around her mouth and neck after being kissed by her grandmother who was eating peanuts at the time. She developed no other symptoms and the hives resolved within a few minutes of receiving diphenhydramine 1 mg/kg. She had not ingested peanuts or tree nuts prior to that incident. Neither of the twins has eaten peanut or tried any tree nuts since this event because of parental concern about an allergy to peanut and tree nuts. Question 1 Which of the following symptoms are commonly associated with the presentation of a food allergy? A. Urticaria and angioedema B. Vomiting and diarrhea

C. A. Popadiuk Division of Pulmonary, Allergy and Critical Care, Department of Medicine, Penn State College of Medicine, Milton S. Hershey Medical Center, Hershey, PA, USA e-mail: [email protected] D. A. Andreae (*) Division of Allergy and Immunology, Department of Dermatology, University of Utah, Salt Lake City, UT, USA e-mail: [email protected]

C. Rhinorrhea, conjunctivitis, and sneezing D. Wheezing E. All of the above The correct answer is E Question 2 Which of the following scenarios warrant further testing for an IgE mediated food allergy? A. All infants with moderate to severe atopic dermatitis B. Patients with symptoms suggestive of anaphylaxis within an hour of ingesting a highly allergenic food C. The sibling of a child with atopic dermatitis who has not had any symptoms concerning a food allergy D. A patient with lactose intolerance E. A and B F. All of the above The correct answer is E Awareness of food allergies has increased within the last few decades, especially in the Western hemisphere. Incidence varies across age groups and in populations worldwide. Recent studies have documented the prevalence of food allergies as high as 10% in most populations worldwide with food allergies far more prevalent in children. By survey sampling, the prevalence of food allergy in US adults has been estimated at 10.8% with the most common allergens reported in order of prevalence as shellfish, peanut, cow’s milk, tree nuts, and finned fish. By contrast, food allergen prevalence in US children is estimated at 7.6% with 40% of these children having more than one food allergy. The most common food allergens reported in US children include peanut and cow’s milk followed by shellfish, tree nuts, egg, and finned fish. For comparison, in recent European survey studies, prevalence of food allergy in adults was as high as 5.6% in Switzerland and lowest at 0.3% in Greece with just as wide a variation in the most common food allergen. Also, unlike the

© The Author(s), under exclusive license to Springer Nature Switzerland AG 2022 M. Mahmoudi (ed.), Absolute Allergy and Immunology Board Review, https://doi.org/10.1007/978-3-031-12867-7_11

113

114

US findings, fruits and vegetables predominated as the major causative food allergens across Europe. Numerous studies have sought to identify risk factors for the development of food allergies and understanding of this has improved in recent decades. The most studied and best-­ described risk factors include skin barrier compromise as in conditions like atopic dermatitis as well as age at first ingestion of a highly allergenic food. Other risk factors to consider which may or may not be modifiable include the intestinal microbiome, exposure to environmental allergens, and maternal dietary habits during pregnancy and lactation. In the simplest of explanations, food allergy is the result of the loss of immunologic tolerance to an ingested food. The clinical manifestations we typically associate with a food allergy are mediated by IgE, a Type I hypersensitivity reaction. While the loss of immunologic tolerance may result in IgE mediated, non-IgE-mediated, or mixed IgE and non-­ IgE-­mediated reactions, the focus of this chapter will be on IgE mediated food allergies. Sensitization to a food allergen is the process of developing food antigen-specific IgE. Repeated exposure to the allergen then induces an IgE-­ mediated hypersensitivity reaction. Symptoms manifest quickly following ingestion of the culprit food. We will first briefly discuss the development of tolerance and sensitization followed by a clinical presentation and diagnosis of food allergies. Tolerance can be induced via the gastrointestinal tract or often through the skin, respiratory system, and oral cavity. For the purposes of this chapter, we will focus on the gastrointestinal tract and skin. In the gastrointestinal tract, food proteins are broken down by gastric acid and digestive enzymes leaving behind a few food proteins usually in the form of peptides which are transported across the gut lumen to dendritic cells. Dendritic cells will internalize and present these peptides as food antigens to naïve T cells in the lymphatic tissue of the gut. Under regular circumstances, this process leads to tolerance of the food proteins. If this process causes T cell release of cytokines leading to B cell proliferation, isotype switching, and the formation of IgE to food antigens it will lead to sensitization to the food proteins, now considered food allergens. Loss of immunologic tolerance also occurs due to breakdown in skin barriers. The twins presented in the above case both suffer from atopic dermatitis (eczema). Skin breakdown especially in patients with atopic dermatitis allows for sensitization to occur via the skin as opposed to the GI tract due to the breakdown of filaggrin which allows for bacterial colonization or infection, epidermal water loss, and allergen penetration through the skin leading to sensitization. Sensitization allows for the development of IgE antibodies to a food allergen, usually a protein. IgE will bind to high-affinity receptors on mast cells and basophils. When this IgE encounters a food allergen, it mediates cross linking

C. A. Popadiuk and D. A. Andreae

of mast cells or basophils which then release mediators leading to a hypersensitivity reaction. These mediators include histamine, tryptase, leukotrienes, prostaglandins, and platelet-­activating factors. Together, they are responsible for the signs and symptoms of allergic reactions including urticaria, angioedema, and smooth muscle contraction which may cause wheezing and/or abdominal pain. Diagnosis of food allergy can be complex. In the US, guidelines for diagnosis and management of food allergies are published by the National Institute of Allergy and Infectious Diseases (NIAID) to promote best practices nationwide. As discussed above, food allergy is usually suspected when typical signs and symptoms of an immediate IgE mediated hypersensitivity reaction occur rapidly following ingestion of a likely food allergen. IgE-mediated reactions are usually quick in onset (minutes to almost 2 h after ingestion) and can manifest with a variety of symptoms depending on the organ system involved. As previously discussed, symptoms are triggered by an IgE-mediated reaction and typically affect the skin, gastrointestinal system, respiratory, and/or cardiovascular system. When the onset of symptoms occurs within minutes to 2 h following ingestion of an allergenic food it is highly suspicious of an IgE-mediated food allergy and warrants further investigation by an allergist. A thorough history is imperative to correctly diagnose a food allergy. Clinical history and evidence of IgE-mediated sensitization to the food or foods in question are required for establishing a diagnosis of food allergy. For example, a child who is otherwise well presenting with a history of urticaria and vomiting within 30 min of ingestion of cashew for the first time would be concerning for a new onset tree nut allergy. However, a child who previously tolerated tree nuts now presenting with symptoms of viral illness for several days including vomiting and urticaria would be less concerning for a new onset food allergy and his/her presentation is more consistent with hives triggered by viral illness. Because of a high probability of false positive results, food allergy screening is not recommended and testing for any food allergy should only be performed in the appropriate clinical scenario based on history. In our case, Twin A presents with new onset urticaria in the setting of what the parents believe is her first exposure to peanut without other symptoms; leading to concern for a food allergy. It is therefore imperative to note, that all testing for confirmation of a food allergy needs to be properly based on a thorough history which is suggestive of a food allergy. Due to the twins’ concerning history in the setting of moderate atopic dermatitis, skin prick testing for peanuts is performed in the clinic: Twin A, because of her history of peanut reaction and Twin B, because of her moderate atopic dermatitis. Results are reflected in the table below along with positive (histamine) and negative (diluent) controls:

Food Allergy

Twin A Twin B

Peanut SPT (mm) 5/15 15/45

115 Diluent (mm) 0/0 2/5

Histamine (mm) 10/50 20/50

Serum IgE to Peanut (kU/L) 0.8 34

As discussed in the above section, all patients with symptoms suggestive of an IgE-mediated reaction following ingestion of a culprit food warrant further testing. In addition to the presence of moderate to severe atopic dermatitis in infants, current guidelines advise further testing even prior to the introduction of egg and peanut in certain cases. We will explore these indications as well as avenues for testing in the next paragraphs. When the history is concerning for an IgE-mediated food allergy, further testing to confirm the presence of said allergy as well as to assess the extent of the allergy should be ­pursued. This is typically performed by an allergist. When the diagnosis is in question, a food allergy can be confirmed or excluded with an oral food challenge. An oral food challenge consists of administering the food in question to a patient suspected of having an allergy to that food in incremental amounts while being monitored for signs and symptoms of an IgE-mediated reaction, which if present, are diagnostic for a food allergy. Oral food challenges remain the gold standard for diagnosis of food allergy; however, most food allergies can be diagnosed on the basis of history and skin prick and/or serum IgE testing to the suspected allergen without an oral food challenge. Diagnostic testing for food allergies includes skin prick and/or serum IgE testing for the suspected allergen. Skin prick testing, as the name implies, involves the cutaneous administration of the allergen extract followed by a bloodless skin prick in order to allow antigen exposure to cutaneous mast cells. This is administered along with positive (histamine) and negative (saline) controls. A resulting wheal and flare at the area of allergen extract application is a measure of food sensitization relative to the size of the negative control. A wheal of 3 mm or more than the negative control is considered a positive test result indicative of sensitization to that particular food allergen. Skin prick testing has been the preferred method for aid in food allergy diagnosis since the 1950s. It should be noted that the specificity and sensitivity of skin prick testing vary by age, study, prevalence of selected food allergy in a given population and by technique. It is also subject to operator error in terms of administration and result interpretation. Current guidelines provide cut-off limits for the size of the wheal/flare that should be taken into consideration when determining if a patient is a candidate for an oral food challenge based on testing results and history. In a similar manner, serum IgE testing for a food allergen is also widely utilized. Current guidelines do not recommend

serum IgE testing alone as a means for diagnosing food allergies. Serum IgE testing has been used since the 1990s and is easily reproduceable. The serum IgE immunoassay, more commonly known as ImmunoCAP testing, is the most commonly used platform. In this form of allergy testing, serum antibodies bind to the allergen and are then quantified with enzyme-labeled anti IgE.  Results are reported in allergen-­ specific kilounits per liter (kU/L) and indicate sensitization to the food allergen tested. Atopic comorbidities such as sensitization to aeroallergens or atopic dermatitis may decrease the pretest probability of serum IgE testing and this should be taken into account when interpreting the results. Sensitivity and specificity vary depending on the food tested. An advantage of serum IgE testing is the possibility of food allergen component testing. Foods contain many allergenic proteins and serum IgE testing allows for the assessment of many of these allergenic proteins in a specific food individually, which skin prick testing does not. Some of these proteins found in certain allergenic foods are elevated due to cross-sensitization with pollens in a patient with concomitant seasonal allergic rhinitis. We will discuss pollen food allergy syndrome also known as oral allergy syndrome later on in this chapter. Component testing will allow the clinician to risk stratify and separate those patients at risk for a severe allergic reaction to food versus those who are cross-­sensitized with pollens but not at high risk for a severe food allergy reaction. While both skin prick testing and serum IgE testing have limitations, when used together they provide useful information for determining the severity of food allergies in an individual patient. Going back to our case, the skin prick testing to peanut is positive for both twins. In order to better assess the extent of peanut allergy in each twin, serum IgE testing for peanut is obtained. Twin A’s serum IgE level to peanut is mildly elevated at 0.8 kU/L and Twin B’s is elevated at 34 kU/L. As discussed above, the serum IgE level and skin prick testing must be considered together with clinical history in determining if an oral food challenge to evaluate for the presence or absence of a food allergy is appropriate. Twin A’s skin prick and serum IgE testing are within acceptable limits to offer an oral challenge to peanut; however, her sister’s results are not. The family chooses to pursue an oral food challenge to peanut for Twin A and she passes. Peanut is introduced into this twin’s diet regularly and she tolerates it without any adverse reactions. Question 3 Commonly known allergenic foods include which of the following? A. Peanuts B. Egg C. Fruits

C. A. Popadiuk and D. A. Andreae

116

D. Tree Nuts E. A, B, D F. All of the above The correct answer is E Question 4 What is the most appropriate management of a confirmed food allergy? A. Strict avoidance of the food the patient is allergic to B. Continued consumption of the food in small amounts C. Serial testing with consumption of the food in small amounts D. Immunotherapy The correct answer is A Peanut, egg, and tree nuts are among the most widely prevalent food allergens in the Western hemisphere. According to recent guidelines, the most common food allergens include cow’s milk, egg, peanut, tree nuts, shellfish, finned fish, wheat, and soy listed in order of prevalence. It should be noted this list will differ among regions and worldwide as the prevalence of food allergy is studied most often in Western nations and is dependent on the foods most commonly eaten by a specific population. These eight foods listed above are responsible for more than half of all food allergies in the United States. In recent years, peanut has emerged as more prevalent, especially in children. Studies cite the prevalence of childhood food allergy as around 7% with peanut as the most common food allergy followed by cow’s milk. Peanut allergy has been evaluated closely over the last few decades, especially with respect to comorbid atopic dermatitis which is the case for our twin infants presented in this chapter. Atopic dermatitis is characterized by an erythematous, papulovesicular rash that usually develops in early infancy. Over time, we have learned about its association with other atopic diseases. Of note, roughly 85% of patients with atopic dermatitis have elevated IgE levels and more often than not these IgE antibodies are to food allergens. Studies have demonstrated that food ingestion can lead to worsening of atopic dermatitis and skin clearing when foods the patients are sensitized to (determined either by skin prick testing or serum IgE testing) were removed from the diet. While initial management guidelines recommended preemptive food avoidance, studies have since demonstrated that avoidance does not prevent the development of food allergy even in patients with severe atopic dermatitis. Rather, in these patients, sensitization to the food might often occur first through the breakdown of the skin which is the result of severe uncontrolled atopic dermatitis. Further studies promote early introduction (before sensitization occurs) of

a­llergenic foods into the diets of infants even those with atopic dermatitis as a means of preventing food allergy. However, this too has come with its own challenges. Perhaps the most well-known study regarding peanut allergy and atopic dermatitis is the Learning Early About Peanut Allergy or LEAP. In this study, infants with severe atopic dermatitis were randomized to two groups—either early peanut consumption or peanut avoidance. The researchers followed these infants from age 11  months through 4-years old, at which time they were tested for sensitization to peanut. The study ultimately concluded that introducing peanut early into the infant’s diet correlated with a decreased prevalence of peanut allergy later. In a similar manner, the Beating Egg Allergy Trial (BEAT) evaluated the early introduction of egg in infants with severe atopic dermatitis. Based on these studies, infants with moderate to severe eczema are now tested for sensitization to egg and peanut in order to preserve a window of opportunity for early introduction of these foods to prevent the development of a food allergy. However, avoidance of the foods in patients who are allergic presents a narrow window for introduction and testing must be completed often for the best chance of preventing the development of food allergy. Getting back to our case, the family of the twins is subsequently lost to allergy follow-up due to several moves; however, return to an allergist’s care when the twins are 5-years old. The parents are eager to see if they can introduce peanut into Twin B’s diet. She undergoes skin prick and serum IgE testing which is depicted in the table below along with positive (histamine) and negative (diluent) controls:

Twin B

Peanut SPT (mm) 10/30

Diluent (mm) 0/0

Histamine (mm) 15/35

Serum IgE to Peanut (kU/L) 18

Twin B’s skin prick testing and serum IgE testing remain too elevated to safely perform an oral food challenge to peanut. She repeats this testing at a 1-year follow up, but her skin prick test and serum IgE levels remain elevated. She does not outgrow her peanut allergy. Her sister continues to tolerate peanut. Once a food allergy is confirmed by diagnosis, the food must be strictly avoided to prevent a life-threatening reaction. Children often outgrow food allergies and serial testing for sensitization whether by skin prick testing or serum IgE testing can be performed to see if the child can safely tolerate an oral food challenge. An oral food challenge is the gold standard of food allergy diagnosis and will determine if the child has or has not outgrown the food allergy. Adults typically do not outgrow food allergies. All patients with a food allergy should be instructed on strict avoidance measures and appropriate use of an epinephrine autoinjector to treat anaphylaxis.

Food Allergy

Immunotherapy is the incremental administration of increasing doses of the allergen with the ultimate goal of raising the threshold dose which will cause an allergic response. In a patient with food allergies, oral immunotherapy (OIT) can prevent life-threatening reactions. Oral immunotherapy has recently been introduced as a treatment for peanut allergy and is meant to reduce the chance of a life-threatening allergic reaction to peanut by achieving sustained unresponsiveness. It does not make peanut safe for peanut-allergic patients to eat at will. The largest, most well-known study to date, ultimately lent itself to the FDA approval of peanut OIT in 2020. Study participants were patients between ages 4 and 17 years with known peanut allergy and randomly assigned to receive the peanut OIT or placebo—67% of the participants receiving OIT were able to consume a minimum of 600  mg peanut protein without symptoms at the end of the study and a follow-on study demonstrated continued safety and desensitization up to ­ 2000 mg peanut protein. This study and the subsequent FDA approval of peanut OIT, generic name AR 101, are a tremendous breakthrough in the management of food allergy. OIT has been investigated as a treatment option for other food allergens, but more studies are needed to assess efficacy and safety. Question 5 Twin B, now in kindergarten, has decided to trade lunches with a friend who has a peanut butter and jelly sandwich. After two bites, she begins gagging and develops hives on her face and arms. What is the likely diagnosis and best treatment option? A. She is probably choking, umbilical thrusts B. She is having an allergic reaction, cetirizine 5  mg by mouth C. She is having an allergic reaction, her epinephrine autoinjector should be administered by an adult immediately D. She probably does not like the taste, watchful waiting The correct answer is C This patient is known to be allergic to peanut and has been witnessed ingesting part of a peanut butter and jelly sandwich. She has developed generalized hives and appears to be choking; the involvement of two organ systems meets the criteria for anaphylaxis and epinephrine, usually in the form of an autoinjector, should be administered immediately to stop the reaction. The patient should then be monitored for any worsening symptoms or a biphasic reaction. Food allergy is best managed by avoidance of the allergenic food and immediate treatment of any allergic reactions caused by accidental ingestion. Of course, avoidance is not always easily done. Consumers and parents of pediatric patients with food allergens as well as caregivers (babysit-

117

ters, coaches, teachers, etc.) will need to familiarize themselves with food labels to avoid cross-contamination or accidental ingestion. Food labeling laws in the United States mandate that all major food allergens are identified. This includes milk, egg, peanut, tree nuts, fish, crustaceans, wheat, and soy. It should be noted that many allergenic foods are labeled with ambiguous phrases such as “may contain” or “processed in a facility with” which are known to be ignored by about 40% of patients with food allergies. It should be noted that while traveling, differences in labeling laws elsewhere in the world including the European Union, may create even higher risk for the patient. Despite strict adherence to a food avoidance diet, cross-­ contamination can occur. This is especially a concern at schools, daycare centers, and camps where children often eat together. This can also occur at places where food is prepared by a person unfamiliar with the patient’s food allergy such as a communal meal among neighbors or at a restaurant. In addition, contact with food other than ingestion can lead to less severe reactions such as skin rashes in food handlers who are allergic to what they are preparing. Other potential sources of contact include poor cleaning of food preparation surfaces, sharing utensils without cleaning, and kissing which can trigger a reaction caused by accidental ingestion when an allergen is transferred in saliva. Uncertainties in food preparation and handling of all kinds necessitate ready identification of an allergic reaction and preparation of the patient and family members or at times, facility staff to intervene appropriately. As alluded to above, even patients adhering to an elimination diet can encounter many factors putting them at risk of accidental ingestion. Therefore, the patient, as well as family and caregivers, must be knowledgeable about recognizing the signs and symptoms of an allergic reaction and understand how to intervene in order to prevent or treat a life-threatening reaction including anaphylaxis. Recognizing an allergic reaction is critical to early intervention which can be life-saving for a patient with a food allergy. Symptoms can appear discretely but progress rapidly and range from localized urticaria, a tingling sensation in the lips or tongue, swelling of the tongue, difficulty breathing, wheezing, skin rashes, eczema flares, vomiting, diarrhea, and/or lightheadedness. Anaphylaxis must be recognized promptly, and intramuscular epinephrine should be administered immediately. The typical dose is 0.01 mg/kg (to a maximum dose of usually 0.5  mg) delivered intramuscularly, though commercial epinephrine autoinjectors apply standardized doses for pediatric and adult patients. Parents and other caregivers should be educated on the appropriate use and administration of intramuscular epinephrine for pediatric patients. All adults with food allergies should be taught to recognize their symptoms and to use an epinephrine autoinjector.

118

Risk factors for predisposition to anaphylaxis in a food allergic patient have been well documented in the literature. Between 1994 and 2006, 63 fatalities from food allergy were studied by the Food Allergy and Anaphylaxis Network/ American Academy of Allergy, Asthma and Immunology. The majority of these fatalities included foods introduced to the patient at a restaurant. Most of these patients had had a previous reaction to the same food implicated in the fatal reaction. The two most common risk factors in this population were concomitant asthma and age less than 30 years old. Teenagers and young adults are likely to exhibit more risky behaviors and/or be away from home without parental supervision. Oftentimes, they do not have access to their epinephrine autoinjector or unfortunately, their peers do not know how to administer it or otherwise assist. Lastly, peanut allergy was implicated in 87% (55 out of 63) of these fatalities. Avoidance of peanuts is not always an easy task given peanuts are used in a variety of cuisines worldwide. This study demonstrates the need for proper education of the patient and family by the allergist and primary care clinician on food allergy avoidance measures as well as early recognition and treatment of allergic reactions. While anaphylaxis must be recognized and treated early, it is possible for patients with food allergy to have more minor reactions following accidental ingestion of the allergen. These can manifest as more subtle symptoms often skin rashes or a few hives in a patient who has had skin contact with the food allergen. Antihistamines can be given to patients with minor symptoms such as localized urticaria. These patients should continue to be monitored for progression of their symptoms and the administration of epinephrine should not be delayed if there is any concern for anaphylaxis. Case 2 A 16-year-old boy is evaluated for possible food allergies. Over the last 3 years, he has noticed numbness and tingling of the lips and tongue when eating fresh fruits, particularly raw apples, pears, and peaches. He used to enjoy these fruits regularly, but his parents have removed them from his diet out of concern for a food allergy. The symptoms do not occur when he eats the fruits in cooked form such as peach cobbler and apple pie. While eating these fruits he has never developed hives, lip-, or tongue-swelling, wheezing, difficulty breathing, vomiting, or diarrhea. His symptoms resolved almost immediately after using an oral antihistamine. The patient has chronic rhinitis and ocular pruritus that are present year-round but worsen in the springtime. This becomes especially bothersome as he is a baseball catcher for his high school team and has difficulty playing when his symptoms flare. His symptoms are fairly well controlled when he remembers to use his over-the-counter oral antihistamine and steroid nasal spray. He is otherwise healthy and does not take any other medications. There is no family history of other atopic diseases, asthma, or medication allergies.

C. A. Popadiuk and D. A. Andreae

Question 6 What is the next appropriate step in this patient’s work up/ what is his likely diagnosis? A. Skin prick testing for the implicated foods/food allergy B. No further workup/food intolerance C. Skin prick testing to perennial and seasonal allergens/pollen food syndrome and chronic allergic rhinitis and conjunctivitis D. Skin prick testing for perennial and seasonal allergens/ nonallergic rhinitis The correct answer is C Question 7 Which of the following pollens are implicated in pollen food allergy syndrome? A. Bermuda grass B. Mugwort C. Birch D. Ragweed E. All of the above The correct answer is E Question 8 The patient undergoes skin prick testing which determines he is sensitized to various trees, grasses, and weeds including birch pollen. He is started on a daily intranasal steroid and as-needed oral antihistamine to treat his allergic rhino-­ conjunctivitis. What is the best treatment option for his food sensitization? A. Complete avoidance of the foods causing his pollen food allergy syndrome B. Consuming these foods in the cooked form only C. Prescribing an epinephrine autoinjector D. No absolute avoidance is necessary but may be indicated based on symptoms The correct answer is D The patient is suffering from allergy symptoms worsening in the spring. This is concerning for seasonal allergic rhinitis. His lip pruritus while ingesting certain raw fruits and vegetables is indicative of pollen food allergy syndrome (also known as oral allergy syndrome) which is caused by initial sensitization to plant proteins from inhaled pollens. Pollen food allergy syndrome is an IgE-mediated process; however, symptoms are most often limited only to the lips and mouth (areas which come into direct contact with the food) and very rarely progress to more systemic symptoms including anaphylaxis. Patients present with seasonal allergic rhinitis as seen in our case, initially tolerating all fruits

119

Food Allergy

and vegetables without adverse reactions. A few years after the onset of the environmental allergies, ingestion of the cross-reactive raw plant foods starts to lead to symptoms such as tingling or itching limited to the mouth, lips, throat, and/or tongue. Patients can usually tolerate the same plant-­ derived food cooked without these symptoms since the protein has been denatured. It is important to note a distinction between foods sensitized through the gut versus the respiratory tract, as food allergens whose initial sensitization is through the gastrointestinal tract are stable to heat and digestive enzymes therefore producing immediate type symptoms including anaphylaxis. However, food allergens whose initial ­sensitization is through inhalation as in pollen food allergy syndrome are cross-reactive with pollens and generally susceptible to degradation by heat and digestive enzymes making systemic symptoms rare. One of the most commonly implicated aeroallergens leading to pollen food allergy syndrome is birch pollen. Birch pollen is cross-reactive with a variety of fruits and vegetables including apple, pear, sweet cherry, peach, plum, apricot, celery, carrot, kiwi, hazelnut, almond, mango, and chili pepper. Sensitization to birch pollen through inhalation therefore can manifest later in life as pollen food allergy syndrome to any number of these fruits, vegetables, or tree nuts. Cross-­ reactivities of common foods implicated in pollen food allergy syndrome are summarized in the Table 1. Because of the underlying mechanism of prior sensitization to pollen, pollen food allergy syndrome is not often seen Table 1  Examples of cross-reactivity patterns between specific pollen and selected natural foods Birch

Grass

Ragweed

Mugwort

Apple Pear Cherry Plum Apricot Celery Carrot Kiwi Mango Chili pepper Hazelnut Peanut Watermelon Tomato Potato Kiwi Orange Watermelon Cantaloupe Zucchini Cucumber Banana Celery Carrot Mango

in young children, but rather predominates in adolescents and adults. Studies suggest a prevalence of about 10% in US adults though can vary based on geographic location both in the US and worldwide depending on the presence or absence of certain aeroallergens. For example, in areas with a high prevalence of birch trees, prevalence of pollen food allergy syndrome in the population has been estimated as high as 90%. Patients with pollen food allergy syndrome are initially sensitized through inhalation of pollen and then develop symptoms to foods due to the cross-reactivity with pollen. While the risk of anaphylaxis in patients with pollen food allergy syndrome is low, about 3% of these patients may develop a systemic reaction and 1.7% develop anaphylactic shock. For this reason, treatment practices can vary, though roughly 70% of allergists practicing in the US prescribe an epinephrine autoinjector for patients with pollen food allergy syndrome due to a history of symptoms, mostly those involving the lower respiratory tract or pharynx, concomitant asthma, facial angioedema and/or hives. Current practice parameters do recommend prescribing an epinephrine autoinjector to patients with severe symptoms such as laryngeal swelling or respiratory distress in addition to avoidance of the raw food. Our patient did not have severe symptoms and therefore was educated on the risks of systemic reactions including anaphylaxis and advised to avoid the food in the raw form if symptoms were bothersome. He may continue to ingest the foods in cooked form.  ocial Aspects of Food Allergy S Food allergies affect children, adolescents, and adults. They can have a significant impact on the quality of life and the socialization of children and adolescents among peers. Management and treatment are a team approach and must include not only the patient and family, but their allergist and numerous other players including teachers, coaches, food handlers, and other personal caregivers. We will end this chapter briefly by looking at the social aspects of food allergy with special regard to food allergy management. Question 9 Recalling our first case, one twin did not outgrow her peanut allergy. When she reaches school age, which factors would contribute to a potentially fatal reaction for the patient at school? A. Use of school lunches instead of homemade lunches B. Lack of school staff to recognize anaphylaxis C. Lack of a food allergy action plan D. All of the above The correct answer is D

120

Question 10 Your peanut allergic patient has now started middle school. She and her parents report bullying because of her food allergy, and you proceed to counsel the family. Which of the following statement regarding the social aspects of food allergy is incorrect? A. A third of children with food allergy have experienced bullying B. Open communication and education are effective strategies to address bullying C. Bullying is rarely targeted at food allergic patients because the disease is not visible D. Allergic reactions due to bullying with the food allergen have been reported E. Addressing food allergy-related bullying requires a team approach The correct answer is C As discussed earlier, food allergy has become more prevalent in recent decades, especially in Western societies. The increase in prevalence suggests a need for increased awareness of prevention and management of adverse reactions due to food allergies. Studies on near-fatal and fatal reactions caused by anaphylaxis from food allergies have concluded that a lack of adequately recognizing anaphylaxis leads to a delay in epinephrine administration. This, in combination with inadequate or sometimes incomplete prevention strategies have contributed to near-fatal and fatal outcomes. Children and teenagers are at an elevated risk of potentially life-threatening reactions due to multiple factors. Young age and lack of appropriate education or carelessness around the severe consequences of a food allergy as well as a lack of understanding or misreading of food labels confounded by the desire to fit in with a peer group can lead to unsafe eating habits. Peer pressure may increase risky behaviors leading to severe and potentially fatal consequences. When and if these patients, by choice or accident, eat the food to which they are allergic, they may become severely ill and unable to self-­ administer epinephrine. Friends may be unfamiliar with the presentation of a severe allergic reaction and/or the use of an epinephrine autoinjector making them unable to assist. Parents and teenagers need to be educated on food allergies and the risks associated with ingestion of the food as well as the need for access to an epinephrine autoinjector in case of an accidental ingestion. For younger children, the risk of a reaction from accidental ingestion is higher in settings like schools and daycares due to food sharing which is increased by the child’s lack of understanding. Significant adverse reactions in school-aged children are most often attributed to an adult’s inability to recognize and appropriately manage a reaction, stressing the need for formal teaching. The aller-

C. A. Popadiuk and D. A. Andreae

gist plays a critical role in educating patients and their caregivers so they may educate others in the child’s life.Higher rates of anxiety have been reported in patients with food allergies, especially children with food allergies and their caregivers This includes anxiety, depression, social isolation, and bullying. In the United States, bullying is defined as harassment with the intention to harm another person, usually taking place when there is an imbalance of power between two people. It can involve physical or and/or verbal attacks intended to harm.Lieberman et al. conducted a study in 2010 specifically surveying caregivers of children with food allergy about the bullying associated with food allergy in children and adults. The investigators found that roughly one quarter (24%) of 353 respondents indicated they or their child have been a victim of bullying due to their food allergy and the most common food allergy reported in cases of bullying was peanut. Excluding children under 5-years old, the group least likely to be bullied, the number of respondents who reported bullying because of food allergy increased to 35.2% overall. The most common setting for bullying to occur was in school (82.4%). It should be noted that the majority of food allergic persons included in this study were school-aged children between the ages of 4 and 11 years old (55%).Lieberman’s findings are not unique as more recent studies have demonstrated similar findings. The FORWARD study compared bullying in black children with food allergy to bullying in white children with food allergy. Surveys were completed by 252 families and the study found that 18.7% of all families (black and white) of children ages 4–15 years old reported their child had been bullied because of their food allergy. However, the majority of food bullying was targeted in older children (>11 years) at 33.3% among both black and white families combined. No significant racial differences related to bullying were identified in the study. Among children with food allergies, survey studies have identified reports of social exclusion, being forced to eat, or make skin contact with the allergic food or taunting with the allergic food.Bullying because of food allergy has been correlated with a decreased quality of life in children with food allergies and their caregivers. Parents, teachers, and other adults who care for children with food allergies need to be aware of the risk of bullying so they can appropriately intervene as necessary. This is yet another aspect of food allergy that the primary care physician and allergist will need to assist their patients in managing.We have presented in this chapter a brief introduction to the epidemiology, pathogenesis, diagnosis, treatment, and social implications of food allergy. Further research in the field will expand our knowledge and understanding of food allergy laying the groundwork for more innovative treatments. A basic understanding of the concepts presented in this chapter is vital for any physician or other clinician type involved in the management of patients with food allergy.

Food Allergy

Bibliography 1. Calvani M, Anania C, Caffarelli C, Martelli A, Miraglia Del Giudice M, Cravidi C, et  al. Food allergy: an updated review on pathogenesis, diagnosis, prevention and management. Acta Biomed. 2020;91(11-S):e2020012. 2. Sicherer SH, Sampson HA. Food allergy: a review and update on epidemiology, pathogenesis, diagnosis, prevention, and management. J Allergy Clin Immunol. 2018;141(1):41–58. 3. Lopes JP, Sicherer S.  Food allergy: epidemiology, pathogenesis, diagnosis, prevention, and treatment. Curr Opin Immunol. 2020;66:57–64. 4. Gupta RS, Warren CM, Smith BM, Jiang J, Blumenstock JA, Davis MM, et  al. Prevalence and severity of food allergies among US adults. JAMA Netw Open. 2019;2(1):e185630. 5. Gupta RS, Warren CM, Smith BM, Blumenstock JA, Jiang J, Davis MM, et al. The public health impact of parent-reported childhood food allergies in the United States. Pediatrics. 2018;142(6) 6. Sicherer SH, Warren CM, Dant C, Gupta RS, Nadeau KC.  Food allergy from infancy through adulthood. J Allergy Clin Immunol Pract. 2020;8(6):1854–64. 7. Lyons SA, Burney PGJ, Ballmer-Weber BK, Fernandez-Rivas M, Barreales L, Clausen M, et al. Food allergy in adults: substantial variation in prevalence and causative foods across Europe. J Allergy Clin Immunol Pract. 2019;7(6):1920–8.e11. 8. Du Toit G, Roberts G, Sayre PH, Bahnson HT, Radulovic S, Santos AF, et al. Randomized trial of peanut consumption in infants at risk for peanut allergy. N Engl J Med. 2015;372(9):803–13. 9. Sampson HA, O'Mahony L, Burks AW, Plaut M, Lack G, Akdis CA.  Mechanisms of food allergy. J Allergy Clin Immunol. 2018;141(1):11–9. 10. Oriel RC, Wang J.  Diagnosis and management of food allergy. Pediatr Clin N Am. 2019;66(5):941–54. 11. Nowak-Wegrzyn A, Burks AW, Sampson HA. Reactions to foods. Middleton’s allergy, principles and practice. 9th ed. Philadelphia, PA: Elsevier; 2020. 12. LaHood NA, Patil SU.  Food allergy testing. Clin Lab Med. 2019;39(4):625–42. 13. Boyce JA, Assa'ad A, Burks AW, Jones SM, Sampson HA, Wood RA, et  al. Guidelines for the diagnosis and management of food allergy in the United States: summary of the NIAID-sponsored expert panel report. Nutr Res. 2011;31(1):61–75. 14. Sampson HA, Aceves S, Bock SA, James J, Jones S, Lang D, et al. Food allergy: a practice parameter update-2014. J Allergy Clin Immunol. 2014;134(5):1016–25.e43. 15. Bettcher CM, Rockwell PG, Ravikumar R.  Managing food allergy in children: an evidence-based update. J Fam Pract. 2020;69(7):336;8;40;43 16. Sampson HA. The evaluation and management of food allergy in atopic dermatitis. Clin Dermatol. 2003;21(3):183–92. 17. Fisher HR, Du Toit G, Bahnson HT, Lack G.  The challenges of preventing food allergy: lessons learned from LEAP and EAT. Ann Allergy Asthma Immunol. 2018;121(3):313–9.

121 18. Wei-Liang Tan J, Valerio C, Barnes EH, Turner PJ, Van Asperen PA, Kakakios AM, et  al. A randomized trial of egg introduction from 4 months of age in infants at risk for egg allergy. J Allergy Clin Immunol. 2017;139(5):1621–8.e8. 19. Sicherer SH, Lack G, Jones SM.  Food allergy management. Middleton’s allergy, principles and practice. 9th ed. Philadelphia, PA: Elsevier; 2020. 20. Vickery BP, Vereda A, Casale TB, Beyer K, du Toit G, Hourihane JO, et al. AR101 Oral immunotherapy for peanut allergy. N Engl J Med. 2018;379(21):1991–2001. 21. Vickery BP, Vereda A, Nilsson C, du Toit G, Shreffler WG, Burks AW, et  al. Continuous and daily oral immunotherapy for peanut allergy: results from a 2-year open-label follow-on study. J Allergy Clin Immunol Pract. 2021;9(5):1879–89. 22. Wood RA. Oral immunotherapy for food allergy. J Investig Allergol Clin Immunol. 2017;27(3):151–9. 23. Muñoz-Furlong A, Weiss CC.  Characteristics of food-allergic patients placing them at risk for a fatal anaphylactic episode. Curr Allergy Asthma Rep. 2009;9(1):57–63. 24. Kim JH. Insights into pediatric pollen food allergy syndrome. Clin Exp Pediatr. 2020;63(12):483–4. 25. Sicherer SH. Clinical implications of cross-reactive food allergens. J Allergy Clin Immunol. 2001;108(6):881–90. 26. Jeon YH.  Pollen-food allergy syndrome in children. Clin Exp Pediatr. 2020;63(12):463–8. 27. Carlson G, Coop C. Pollen food allergy syndrome (PFAS): a review of current available literature. Ann Allergy Asthma Immunol. 2019;123(4):359–65. 28. Young MC, Muñoz-Furlong A, Sicherer SH. Management of food allergies in schools: a perspective for allergists. J Allergy Clin Immunol. 2009;124(2):175–82, 82.e1–4; quiz 83-4 29. Ravid NL, Annunziato RA, Ambrose MA, Chuang K, Mullarkey C, Sicherer SH, et  al. Mental health and quality-of-life concerns related to the burden of food allergy. Psychiatr Clin North Am. 2015;38(1):77–89. 30. Services USDoHaH.  What is Bullying https://www.stopbullying. gov/bullying/what-­is-­bullying. 31. Egan M, Sicherer S.  Doctor, my child is bullied: food allergy management in schools. Curr Opin Allergy Clin Immunol. 2016;16(3):291–6. 32. Lieberman JA, Weiss C, Furlong TJ, Sicherer M, Sicherer SH.  Bullying among pediatric patients with food allergy. Ann Allergy Asthma Immunol. 2010;105(4):282–6. 33. Brown D, Negris O, Gupta R, Herbert L, Lombard L, Bozen A, et  al. Food allergy-related bullying and associated peer dynamics among black and white children in the FORWARD study. Ann Allergy Asthma Immunol. 2021;126(3):255–63.e1. 34. Cooke F, Ramos A, Herbert L.  Food allergy-related bullying among children and adolescents. J Pediatr Psychol. 2022;47(3): 318–26. 35. Herbert L, Shemesh E, Bender B.  Clinical management of psychosocial concerns related to food allergy. J Allergy Clin Immunol Pract. 2016;4(2):205–13.

Eosinophilic Disorders Taha Al-Shaikhly, Matthew R. Norris, Heather Stern, and Gisoo Ghaffari

Abbreviations DOCK8 Dedicator of Cytokinesis 8 EGID Eosinophilic gastrointestinal disorder EGPA Eosinophilic granulomatosis with polyangiitis EoE Eosinophilic esophagitis HE Hypereosinophilia HEFA Familial hypereosinophilia HER Reactive hypereosinophilia HES Hypereosinophilic syndrome HEUS Hypereosinophilia of undetermined significance L-HES Lymphocytic variant of hypereosinophilic syndrome M-HES Myeloid variant of hypereosinophilic syndrome STAT3 Signal transducer and activator of transcription-3 Th2 Type-2 helper

1 Introduction Eosinophils are white blood cells of myeloid origin, contain granules, which produce many molecules and chemicals through which they participate in host defense against certain infections and are involved in various allergic and immunological responses. Understanding the nature, function, and regulatory mechanisms of eosinophils is essential to allergists-­ immunologists. Eosinophilic disorders are most commonly secondary to an underlying disease process. Eosinophils could also be the primary source of the disorder, T. Al-Shaikhly · M. R. Norris · H. Stern · G. Ghaffari (*) Division of Pulmonary, Allergy and Critical Care Medicine, Department of Medicine, Pennsylvania State College of Medicine, Penn State Health-Milton S Hershey Medical Center, Hershey, PA, USA e-mail: [email protected]; mnorris3@ pennstatehealth.psu.edu; [email protected]; [email protected]

most commonly due to aberrant production of eosinophilopoietic cells. Recognizing the differences between these and understanding the molecular basis of these conditions are important for proper diagnosis and management of these conditions, which are frequently encountered by practicing allergists-immunologists. In this chapter, we will discuss eosinophil biology, definition of eosinophilia and its severity, common causes of ­secondary eosinophilic disorders, and eosinophilic gastrointestinal disorders with a particular focus on eosinophilic esophagitis and hypereosinophilic syndromes. These will be discussed through case-based questions. Two cases are presented, with five questions related to each and explanations of the answers both briefly and in more detail. Case 1 A 26-year-old white male presents to the allergy clinic for evaluation of a few years’ history of intermittent difficulty swallowing particularly with solid foods. His symptoms, which were initially sporadic, are getting worse in the past 6 months. He was recently at a family dinner when for the first time, food got stuck in his throat while eating turkey. Induction of vomiting did not resolve the symptom. He was taken to an emergency department where a food bolus was removed via endoscopy, and he was told that his esophagus looked inflamed. He had experienced milder episodes while at college after eating peas, pork, beef, and chicken. These symptoms do not occur every time while eating one of these foods and are not associated with any ocular, nasal, respiratory, or dermatologic symptoms. His medical history included childhood asthma not requiring daily inhaler therapy and seasonal rhinitis for which he takes antihistaminic pills on as-needed basis. He has a history of eczema, which cleared by age 5 and had an IgE-mediated milk allergy until the age of 7. His mother and sister have asthma and allergic rhinitis. He is a medical student, does not smoke, drinks alcohol in moderation, and does not use any illicit drugs. He lives in an apartment, which has carpeting, and last year during the pandemic, he obtained a pet dog.

© The Author(s), under exclusive license to Springer Nature Switzerland AG 2022 M. Mahmoudi (ed.), Absolute Allergy and Immunology Board Review, https://doi.org/10.1007/978-3-031-12867-7_12

123

124

T. Al-Shaikhly et al.

He does have occasional heartburn and regurgitation of food. He admits to being the last person to finish his meals when eating with others and tends to drink plenty of water with meals to help with swallowing foods. After the visit to the emergency department, he was referred to a gastroenterologist for an esophagogastroduodenoscopy. Gross endoscopy revealed linear furrows and esophageal biopsy showed intraepithelial eosinophilic infiltration (20 eosinophils per high power field in proximal esophagus, 25  in mid esophagus, and 30  in distal esophagus). No eosinophilic inflammation was identified in either gastric or duodenal biopsies taken during the procedure. H-pylori was not identified. He was told that he most likely has an allergic condition known as eosinophilic esophagitis and was referred to an allergist to further help with the management plan. Question 1 The patient remembers from basic science years in medical school, that eosinophils are seen in parasitic infestations and is wondering if that is the reason for this pathological finding. The allergist explains to him the nature of eosinophils and how he might have gotten them in his esophagus. Which of the following is incorrect? A. Eosinophils are primarily tissue leukocytes and can be normally seen in some tissues, including the esophagus B. Eosinophilic granules produce chemicals which are involved in protection against parasitic infestations C. Increased number of eosinophils are commonly seen in allergic conditions D. Eosinophils can be seen in an increased amount in tissues without being at higher levels in the circulation E. Interleukin-5 is an important cytokine in the process of eosinophilopoiesis Answer: A Eosinophils are normally found in many tissues including many parts of the gastrointestinal tract; however, normally the esophagus should be devoid of any eosinophils.

2 Eosinophil Biology

and in humans, they have a tissue-to-blood ratio of approximately 100:1. They have a circulating half-life of approximately 8–18 h while their life span in tissue ranges from 2 to 5 days, and this may be prolonged by cytokines. Eotaxins, which are chemokines that bind to the eosinophil chemokine receptor CCR3, play a key role in the recruitment of eosinophils into tissues. Eosinophils contain several types of granules. The most important are specific or secondary granules, which contain major basic protein (MBP), eosinophil cationic protein (ECP), eosinophil-derived neurotoxin (EDN), and eosinophil peroxidase which are capable of inducing tissue damage. Charcot-Leyden crystals, which can be found in tissues where there is eosinophilic inflammation, are located in the primary granules of eosinophils. Eosinophils also produce lipid mediators including cysteinyl leukotrienes, prostaglandins, and platelet-activating factor (PAF) that play a role in allergic disease, specifically related to their effects on vascular permeability, vascular and bronchial smooth muscle tone, and chemotaxis. Even when eosinophils are no longer present in a specific tissue, the presence of their granules and mediators is evidence of their existence. To detect evidence of prior eosinophil degranulation in addition to intact eosinophils, immunofluorescent identification can be used, which is more sensitive than other staining techniques.

3 Eosinophilia Eosinophilia is an increase in the number of eosinophils in blood and/or tissue. Greater than 450–550 cells/μL in the periphery is typically considered elevated. Although knowing the absolute eosinophil count is more important, greater than 5% of eosinophils on a differential is generally considered abnormal. The severity of eosinophilia does not necessarily predict end-organ damage; however, tissue damage is more likely to occur when the peripheral eosinophil count is greater than 1500 cells/μL.

4 Secondary Versus Primary Eosinophilic Disorders

When evaluating a patient with eosinophilia, a practical Eosinophils are derived from multipotent CD34 hematopoi- approach involves considering if the eosinophilia is secondary etic stem cells in the bone marrow. Eosinophil differentiation to another pathologic process or if it is related to a primary from the myeloid lineage is dependent upon the transcription process in which the eosinophils are the main source of tissue factors GATA-1, PU.1, and the C/EBP family, and is and/or organ damage. Common secondary causes of eosinoinstructed by the eosinophil-promoting cytokines including philia include atopic conditions, chronic rhinosinusitis, druginterleukin-3 (IL-3), interleukin-5 (IL-5), and granulocyte-­ induced eosinophilia, and infection-related eosinophilia. macrophage colony-stimulating factor (GM-CSF). The Significant eosinophilia in a patient with asthma should raise eosinophil life cycle includes bone marrow, blood, and tissue concern for possible allergic bronchopulmonary aspergillosis stages. However, eosinophils are primarily tissue-dwelling, or eosinophilic granulomatosis with polyangiitis. +

Eosinophilic Disorders

Drug-induced eosinophilia can occur with a prescription or nonprescription medications including herbal supplements. In areas of the world where helminth infections are uncommon, persistently elevated eosinophil levels are most commonly attributed to drug reactions. Drug-induced eosinophilia may occur alone or in conjunction with other manifestations, such as rash and/or fever. Various types of drug reactions can be associated with eosinophilia including but not limited to drug-induced hepatitis, which can occur with tetracyclines or semisynthetic penicillins, and drug-induced interstitial nephritis, which has been associated with cephalosporins or semisynthetic penicillins. Drug reaction with eosinophilia and systemic symptoms (DRESS) is a severe adverse drug reaction that can have a heterogeneous clinical presentation associated with an extensive skin rash. It often presents with fever and lymphadenopathy, and organ failure can result. Commonly implicated medications include antibiotics, anticonvulsants, and allopurinol. Helminth infections are a leading cause of eosinophilia worldwide. Strongyloides is a common culprit. Other helminths that can be contracted in the United States without foreign travel include Trichinella, Ascaris, hookworm, and visceral larva migrans. Ectoparasites (particularly scabies), certain fungi (such as coccidioidomycosis and cryptococcosis), and rarely viruses (such as human T cell lymphocytic virus-1) can result in eosinophilia while bacterial infections are not typically associated with eosinophilia. Eosinophilia may occur in the setting of certain malignancies such as Hodgkin’s lymphoma, adenocarcinomas, and thyroid cancer whereas some malignancies are eosinophil-­ derived, such as acute and chronic eosinophilic leukemias. Systemic mastocytosis is another condition that can be associated with peripheral eosinophilia, which occurs in up to 28% of patients with the condition. Eosinophils can also be present on bone marrow biopsy even if there is not significant peripheral eosinophilia. There are various immunologic disorders associated with peripheral and/or tissue eosinophilia including but not limited to Omenn Syndrome, DOCK8 deficiency, and Wiskott-Aldrich Syndrome. Lastly, connective tissue and autoimmune disorders can also be associated with eosinophilia. Eosinophils can be found in specific organs without being present in the circulation. Organ-specific eosinophilia can be seen in many tissues including the respiratory and gastrointestinal tracts, skin, and mucous membranes.

5 Eosinophilic Gastrointestinal Disorders Eosinophils can be found normally in all parts of the gastrointestinal tract, except for the esophagus. The upper limit of normal in the various sections of the gastrointestinal tract is

125

not well-defined. However, their presence in excess is almost always abnormal and raises a broad differential diagnosis. Eosinophilic gastrointestinal disorders (EGIDs) are diseases characterized by eosinophil-rich inflammation of the gastrointestinal tract. Symptoms of these diseases are often heterogeneous and nonspecific. EGIDs can have a significant impact on quality of life and are associated with the need for extensive dietary restrictions. Current literature supports the role of immunological, environmental, and genetic factors in the pathogenesis of EGIDs.

6 Eosinophilic Gastritis, Eosinophilic Gastroenteritis, and Eosinophilic Colitis The remaining EGIDs are poorly characterized and infrequently encountered in clinical practice. They are pathologically characterized by eosinophilic infiltration of their respective tissue of the gastrointestinal tract, with symptoms often relating to the tissue level affected. There are the mucosal, the muscularis, and the serosal forms of EGID.  Using eosinophilic gastritis and gastroenteritis as an example, the mucosal form will be mostly characterized by nausea, vomiting, abdominal pain, and malabsorption syndromes, such as anemia and protein-losing enteropathies. The muscularis form often results in thickening of the bowel wall that results in decreased motility and obstructive symptoms. The serosal form, a relatively rare variant compared to the mucosal and muscularis forms, is characterized by higher peripheral eosinophil counts and ascites. The differential diagnoses for these conditions can be extensive, and there is no consensus on an ideal treatment approach. With eosinophilic colitis, in particular, the accumulation of eosinophils in the colon has a very wide differential, and their presence should prompt investigation of secondary causes of eosinophilic inflammation, such as inflammatory bowel diseases. Question 2 Which of the following may have conferred a predisposition to developing eosinophilic esophagitis as the cause of dysphagia? A. Patient’s gender B. Patient’s race C. Patient’s age D. Patient’s history of atopy E. All of the above Answer: E All of the epidemiologic factors provided, and the patient’s history of atopy puts him at higher risk of developing eosinophilic esophagitis. Over 75% of patients with EoE

126

T. Al-Shaikhly et al.

will have at least one atopic disease. History of asthma, allergic rhinitis, food allergy, and atopic dermatitis are common among EoE patients.

There have been reports of 3–5% risk of developing EoE in patients receiving oral immunotherapy for the treatment of food allergy. In these patients, remission of the EoE will occur upon cessation of the immunotherapeutic antigen

7 Epidemiology and Risk Factors for Eosinophilic Esophagitis

Question 3 The student then asks his physician if there are any other steps to confirm the diagnosis. Which of the following statements is correct regarding EoE?

While EoE can be diagnosed at any age, most patients are diagnosed in their third decade of life. There is a strong male predominance in EoE with an estimated males-to-females ratio of 3:1. A recent meta-analysis showed a pooled prevalence of 53.8 per 100,000 inhabitants for male patients compared to 20.1 per 100,000 for female patients. Patients are also more likely to be Caucasian. A study of over 7000 patients with EoE found that 89% were Caucasian, 6.1% African American, and 5.6% Asian. EoE in Hispanic patients is thought to be rare, with the prevalence of EoE in Caucasian patients being sevenfold higher than in Hispanic patients. Socioeconomic status is not associated with the diagnosis of EoE in either adults or children. Furthermore, over 75% of patients with EoE will have at least one atopic disease, with asthma, allergic rhinitis, and eczema being more common among EoE patients There appears to be a genetic component to EoE, as 10% of parents of EoE patients have a history of esophageal strictures. Dizygotic twins have a 22% disease concordance, and 2.4% of siblings of patients with EoE have EoE, which is higher than the risk of EoE in the general population (0.05%). Studies of monozygotic and dizygotic twins suggest genetic risk variants account for 15% of the phenotypic variation of disease risk. Although many candidate genes are under investigation, three genes responsible for producing thymic stromal lymphopoietin (TSLP), calpain-14, and eotaxin-3 have been identified as being altered in association with EoE. CCL26 is the most highly expressed gene in the EoE transcriptome and is induced by IL-13. As with many atopic diseases, the impact of altered immune system stimulation and microbiome disruption at an early age appears to increase the risk of developing EoE. Children with EoE had higher odds (odds ratio, 3.2) of being born via cesarian section when compared to controls. Antibiotic use in the first year of life was also found to be associated with 3.5 times higher odds of developing EoE. Although breastfeeding has not consistently been found to correlate with protection against EoE development, a case-control study identified a possible protective role of breastmilk in a subset of patients. Formula feeding, neonatal intensive care unit admission, prematurity, maternal fever, C-section, antibiotic use in infancy, and acid suppressant use in infancy have all been found to be early risk factors for the development of EoE, with antibiotic and proton pump inhibitor use in infancy showing the most consistent correlation.

A. Regardless of his symptoms, presence of more than 15 eosinophils/hpf in any part of esophagus is sufficient for diagnosis B. History of food impaction along with furrowing in the esophagus is sufficient for diagnosis C. History of dysphagia, with more than 15 eosinophils/hpf, which is confined to esophagus, and exclusion of H-pylori are consistent with diagnosis of EoE D. Treatment with high-dose PPI is required to exclude GERD before making the diagnosis of EoE E. GERD and EoE are separate entities and they do not coexist together Answer: C EoE is a clinicopathological diagnosis. Clinical symptoms of esophageal dysfunction along with pathological findings of at least 15 eos/hpf in various parts of the esophagus are required for diagnosis. Except for GERD which may coexist, other causes of esophageal eosinophilia should be excluded either by history or through endoscopic findings.

8 Eosinophilic Esophagitis Eosinophilic esophagitis (EoE) is a clinicopathologic diagnosis. It is defined by the presence of symptoms attributed to esophageal dysfunction (including dysphagia and food impaction in adults or feeding intolerance and gastroesophageal reflux disease-like symptoms in children) and eosinophil-­ predominant inflammation of ≥15 eosinophils per high power field (HPF). EoE has emerged over the past 30 years as one of the leading causes of food impaction and dysphagia in adults and children. It was first described in 1978 by Landres and colleagues as an isolated case of vigorous achalasia with marked smooth muscle hypertrophy and eosinophilic infiltration of the esophagus. Case reports ­continued to accumulate over the next decade, culminating in the 1989 case series entitled “Esophageal Asthma—an episodic dysphagia with eosinophilic infiltrates” by Attwood and colleagues.” They went on to publish their description of these adult patients with dysphagia, normal pH monitoring, and esophageal eosinophilia in 1993, titled “Esophageal

127

Eosinophilic Disorders

Eosinophilia with dysphagia. A distinct clinicopathological syndrome.” There were rapid discoveries of this novel disease through the 1990s, with the first guidelines for EoE being written in 2007. These guidelines were first updated in 2011 and most recently in 2020. These will be discussed in further detail in this chapter.

9 Clinical Presentation of Eosinophilic Esophagitis The clinical presentation of EoE is nonspecific and symptoms vary by age of onset (Fig. 1). Infants and toddlers tend to present with food refusal, feeding difficulties, vomiting, and/or failure to thrive. Older children will present with nausea, vomiting, regurgitation, heartburn, and abdominal pain. The most common presenting symptoms in adults are food impaction, chest pain, and dysphagia, with dysphagia being the most common presentation of EoE in patients ≥18  years of age. This progression of symptoms is well documented, with some questions as to whether adults have a late-onset disease (i.e., a different clinical phenotype of EoE) or have asymptomatic chronic inflammation from childhood that goes unrecognized. There is growing evidence to support the second phenotype of EoE affecting patients 65  years of age or older. These patients have a higher likelihood of having gastric acid reflux and are less likely to be male. Fig. 1  Clinical presentation of eosinophilic esophagitis varies by age

10 Diagnosis of Eosinophilic Esophagitis EoE is a clinicopathological diagnosis and both clinical symptoms and pathological criteria should be present at the same time. The diagnosis of EoE should be considered if the patient has a history of dysphagia and personal or family history of atopy, with or without peripheral eosinophilia. Positive allergy skin or serologic testing is not needed to diagnose EoE, as a large fraction of patients with EoE lack allergen sensitization. The gold standard for diagnosing EoE remains tissue biopsy of the esophagus showing increased intraepithelial eosinophilic inflammation. Although upper endoscopy is diagnostic in >80% of patients with mucosal involvement, upto 10% of endoscopies in patients with EoE can appear normal, particularly in children Traditionally, at least 5 biopsies are obtained at multiple esophageal levels to maximize the sensitivity of identifying intraepithelial eosinophilia. One or more of the biopsies must show eosinophil-­ predominant inflammation of at least ≥15 eosinophils/HPF. Proton pump inhibitor-responsive esophageal eosinophilia (PPI-REE), an entity described in the 2011 consensus guidelines, was later eliminated from the most recent consensus guidelines. PPI-REE was recategorized as being part of the EoE continuum. To this end, transcriptional profiling of PPI-­ REE patients demonstrated a near-identical gene expression profile when compared to PPI-unresponsive EoE ­counterparts. Therefore, a trial of high-dose PPI therapy is no longer required before making an EoE diagnosis.

Clinical Presentation of Eosinophilic Esophagitis

Infants/ Toddlers

Children

Adults

Key Symptoms by Age Food Refusal Feeding Difficulty Vomiting Failure to Thrive

Nausea Vomiting Regurgitation Heartburn Abdominal Pain

Food Impaction Chest Pain Dysphagia

128

Question 4 The student is curious to know how the patient developed the condition, anything he could have done to prevent that from happening and if his history of other allergic conditions has anything to do with this diagnosis. Which statement is incorrect about the pathogenesis of EoE? A. Genetic, epigenetic, and environmental factors play roles in the pathogenesis B. Th-2 inflammatory cytokines are responsible for many of the findings including eosinophilic inflammation C. EoE is an IgE-mediated allergic condition with food antigens being the main culprit D. TGF-ß could be responsible for fibrostenotic changes in the esophagus E. IL-13 causes downregulation of fillagrin, a protein involved in the epithelial barrier function Answer: C Although food antigens can trigger an inflammatory response, EoE is not an IgE-mediated food allergy.

11 Pathogenesis of Eosinophilic Esophagitis There has been a tremendous amount of research on the pathogenesis of EoE, particularly on the role of genetic and environmental factors. The pathogenesis of EoE appears to be primarily related to impaired esophageal epithelial barrier function that results in enhanced Th2 stimulation and antigen stimulation susceptibility. Many EoE patients have evidence of dilated inter-epithelial spaces, altered epithelial barrier function, and downregulation of adhesion molecules, such as desmoglein-1, and barrier function proteins, such as filaggrin and zonulin-1. Collectively, these histologic changes may result in enhanced antigen presentation of food antigens. The resulting increase in pro-inflammatory cytokine release (e.g., TSLP) leads to a Th2-predominant release of IL-4, IL-5, IL-13, and TGF-ß, which results in further disruption of the epithelial barrier, tissue remodeling, and promotes eosinophil infiltration. IL-4 release results in increased T cell differentiation into Th2. IL-13 release induces the secretion of eotaxin-3 from epithelial cells. Additionally, IL-13 downregulates the expression of the adhesion molecule desmoglein-­1 and the barrier function protein filaggrin. Increased eosinophil and mast cell recruitment results in the release of TGF-ß, which promotes fibrotic changes in the esophagus. EoE pathogenesis does not appear to be IgE-mediated because (1) skin testing does not consistently identify food triggers, (2) biologic therapy with omalizumab, a monoclonal antibody that binds to the free human immunoglobulin E,

T. Al-Shaikhly et al.

has not improved EoE symptoms and (3) EoE has been recreated in IgE-null mice. The non-IgE-mediated role for food antigens in EoE pathogenesis is supported by symptom responsiveness to dietary elimination challenges and recurrence of symptoms upon food reintroduction in both adult and pediatric populations. Question 5 The physician then explains how to treat this condition to prevent future episodes of food impaction and the outcome of this diagnosis. Which of the following statements is correct regarding the treatment of EoE? A. Treatment with omalizumab, which targets the IgE-­ mediated allergic response, is effective B. PPI is the first line of treatment for many individuals regardless of presence or absence of symptoms of acid reflux C. Dietary modifications are only effective in children, but not in adults D. Since he just had an endoscopy with dilatation, no further treatment is needed E. Barrett’s esophagus can be seen in both GERD and EoE Answer: B Based on the recent guidelines for treatment, PPI is the first line of treatment for many individuals.

12 Treatment of Eosinophilic Esophagitis The goals of managing EoE patients include (1) alleviating the inflammatory process, (2) providing symptomatic control, and (3) preventing fibrotic sequela. Ideally, the patient should be connected with an interdisciplinary team inclusive of an allergist, a gastroenterologist, and a nutritionist. The three medical treatment modalities currently available include dietary therapy (elemental diet or selective dietary elimination), PPI, and swallowed steroids. In 1995, a landmark study by Kelly and colleagues sought to determine the effect of diet on eosinophilic esophagitis. In this study, 10 children with EoE who were unresponsive to PPIs or fundoplication, attained clinical and histological remission while consuming an amino acid-based formula. Symptoms and esophageal eosinophilia recurred with the reintroduction of allergenic foods, establishing a role for food antigens in the pathogenesis of the disease. Subsequent single-armed, observational studies demonstrated that ~94% of pediatric patients and ~ 80% of adult patients on an elemental diet will experience histologic remission. Harms of elemental diets in EoE include social isolation, cost of elemental formula, the burden of repeat endoscopies on food reintroduction, and the development of oral motor skill defi-

Eosinophilic Disorders

cits. Elemental diets are therefore designed for short-term use only. For these reasons, the 2020 practice parameters for EoE suggest that patients who wish to avoid the challenges of adherence to an elemental diet and prolonged process of dietary reintroduction may reasonably decline this treatment option. To overcome the challenges associated with an elemental diet, less-restrictive variations of elimination diets have been proposed. A six-food elimination diet of the most commonly implicated food triggers results in symptom resolution in approximately 68% of patients based on 10 single-arm, observational studies. These foods are those made with cow’s milk (~75% of patients require continued elimination), wheat (~26% of patients), egg (~17% of patients), soy (~10% of patients), peanut (~6% of patients)/tree nuts and seafood (fish and shellfish). After complete cessation of all six foods for 8–12 weeks, a repeat endoscopy is needed to assess for histopathological remission. If EoE is resolved, foods are then reintroduced individually, followed by repeat endoscopy. The 2020 practice parameters for EoE carry the same notation as for the elemental diet, specifically that patients may decline the six-food elimination diet if they wish to avoid the challenges of adherence to it and the requirement for multiple endoscopic procedures. More liberal variations of the six-food elimination diet include a four-­ food elimination diet (milk, wheat, eggs, soy) and a milk-only elimination diet. There have been attempts to use allergy testing to guide dietary elimination; however, there is only very low-quality evidence to support such an approach. Coupled with the limited accuracy of current food allergy testing, the 2020 guidelines state that patients may reasonably prefer an alternative therapeutic approach. Although PPI therapy no longer plays a role in the diagnostic evaluation of patients with suspected EoE, it can induce histologic remission in approximately one-third of patients. Although the mechanism behind such histologic remission has yet to be elucidated, in vitro studies showed that PPIs, independently of their acid suppressive effect, decrease IL-13-induced eotaxin-3 production, which may also account for their anti-inflammatory benefit. In patients with symptomatic esophageal eosinophilia, the 2020 guideline conditionally recommends using PPIs over no treatment. That said, direct comparison of PPIs to other therapeutic modalities is limited because patients who were responsive to PPIs were excluded from earlier clinical trials based on the previous categorization of PPI-responsive esophageal eosinophilia, the term which is no longer used. Topical glucocorticoid therapy, having an established role in asthma, is being increasingly recognized for its ability to target the inflammatory basis of EoE.  Based on several double-­ blinded, placebo-controlled studies assessing the benefit of topical steroids (budesonide or fluticasone), approximately 67% of patients can experience histologic

129

remission. For this reason, practice parameters recommend using topical steroids over no treatment. In studies ≤3 months, there was no increased risk of adverse events in patients treated with topical steroids compared to placebo, but long-term studies are still lacking. Although no FDA-­ approved treatments exist in the United States, the European Medicines Agency (EMA) approved a budesonide tablet for EoE in 2018. The use of systemic glucocorticoids to treat EoE has further been shown to have similar therapeutic results at the expense of a greater side effect profile. In patients who develop EoE-associated dysphagia secondary to stricture formation, the 2020 practice parameters recommend endoscopic dilation since 87% of these patients will experience symptomatic improvement with such intervention. Other therapeutic modalities under investigation include the use of biologic therapies, montelukast, cromolyn sodium, immunomodulators, and anti-TNF therapies. Although some literature exists supporting their use, the 2020 guidelines recommend their use only in the setting of a clinical trial.

13 Prognosis of Eosinophilic Esophagitis Although 81% of patients diagnosed with EoE in childhood experience regression of their symptoms as young adults, both prospective and retrospective studies indicated that spontaneous remission of EoE is uncommon. Treatment is necessary as untreated EoE is associated with fibrostenotic complications; untreated EoE has not been associated with Barret’s esophagus or premalignant state. As the guidance for management continues to evolve, the 2020 American Gastroenterological Association and the Joint Task Force on practice parameters for the management of EoE serve as the most up-to-date consensus on best practices. It is important to recognize that there is no FDA-­ approved treatment for eosinophilic esophagitis. It is also important to note that all therapeutics evaluated are applicable to short-term treatments only as current evidence is based on trials extending from 2 to 16 weeks. Long-term therapy with medicaments, such as topical steroids and PPIs has not been studied—either for efficacy or long-term side effects. Although side effects such as nutritional deficiencies can be managed in a patient on dietary restriction, long-term use has a significant impact on quality of life in other conditions for which it is used. Case 2 A 54-year-old man presented for evaluation of a 6-month history of worsening pruritic skin rash. The rash is ­maculopapular and was unresponsive to topical corticosteroids. He was evaluated by a dermatologist and skin biopsy was performed revealing eosinophilic infiltration. Complete

130

blood count revealed an absolute eosinophil count (AEC) of 1600 cells/μL. Six weeks later, his AEC increased to 4600 cells/μL.  Patient-reported no respiratory or gastrointestinal symptoms. He is an active smoker with a 10-pack year smoking history. He reported no history of travel outside the United States. Physical examination revealed a diffuse maculopapular rash involving the trunk. No hepatosplenomegaly was noted. Chest X-ray showed no evidence of parenchymal lung abnormality. His cardiac Troponin I was within normal limit. Echocardiographic evaluation showed a normal ejection fraction with no cardiac motion abnormalities. His liver enzymes were within normal limits. Serum B12 level was 400  pg/μL (normal, 200–1100  pg/μL). Quantitative immunoglobulin levels including IgG, IgM, and IgA levels were within normal limits, but his serum IgE level was elevated at 1100 IU/L. Lymphocyte subsets analysis by flow cytometry showed a CD3+ T cell count of 1000 cells/μL with CD4+ count of 1000 and CD8+ of 200/μL. The patient was started on prednisone 1 mg/kg/day and repeat AEC 3 days later was 300 cells/μL. His skin rash improved. Question 1 Which of the following is the most accurate statement? A. This patient has hypereosinophilia of undetermined significance B. This patient has hypereosinophilic syndrome C. This patient has idiopathic hypereosinophilia D. This patient has clonal hypereosinophilia Answer: B This patient has hypereosinophilic syndrome as evidenced by his peripheral blood hypereosinophilia and the presence of end-organ damage (cutaneous involvement). This patient most likely has a form of reactive hypereosinophilia. Hypereosinophilia describes the presence of hypereosinophilia in the absence of end-organ damage.

14 Hypereosinophilia and Hypereosinophilic Syndromes Hypereosinophilia is defined as an absolute eosinophil count of 1500 cells/microliter on two occasions at least 4  weeks apart and/or tissue hypereosinophilia. Based on the underlying basis of eosinophilia, hypereosinophilia is classified into four major categories: (1) reactive or secondary (HER) driven by eosinopoetic cytokines, (2) primary or clonal HE (HEN), (3) familial (HEFA), and, lastly, (4) idiopathic in which no underlying basis for eosinophilia exists (HEUS) (Table  1). Patients with hypereosinophilia and end-organ damage or

T. Al-Shaikhly et al. Table 1  Mechanism of the different classes of hypereosinophilia (HE) and hypereosinophilic syndromes (HES) Mechanism of HE Excess IL-5 production

HE classification HES variant HER Associated HES Overlap HES L-HES Clonal expansion of eosinophils HEN M-HES Unknown HEUS Idiopathic HES Unknown HEFA Familial HES Abbreviations: HER reactive or secondary, L-HES lymphocytic variant of HES, HEN neoplastic or clonal HE, M-HES myeloid variant of HES, HEFA familial HE, HEUS HE of undetermined significance

dysfunction attributable to tissue HE are said to have hypereosinophilic syndrome (HES). Therefore, other causes for organ damage should be excluded before making the diagnosis of HES. HES is often diagnosed in the fifth decade of life (median age at diagnosis, 45 years) and the disorder has an overall slight male predominance (M:F ratio is 1.3). Eosinophils can cause tissue damage by promoting fibrosis and the clinical manifestations of HES vary depending on the organ(s) involved. The most frequently affected organs in HES are the skin, gut, and lungs with gastrointestinal involvement being more common in pediatric patients whereas pulmonary involvement is more common in adult patients. Serious manifestations such as those secondary to neurological and cardiovascular involvements are relatively less frequent. Further, eosinophils can promote thrombosis, yet another sign of end-organ damage, which is thought to be related to the excess expression of tissue factor in the eosinophils of patients with HES. These various causes of HE and the wide clinical manifestations of HES call for a thorough evaluation to identify evidence of end-organ damage and the basis of HE, as timing and type of therapy differ by the extent of tissue damage and the underlying variant of HES. Question 2 Which of the following is the most likely diagnosis in the patient described above? A. B. C. D.

L-HES M-HES Associated HES EGPA

Answer: A The patient is likely to have L-HES. His age at onset, the indolent course (6-month duration), predominant cutaneous manifestation, elevated IgE level, and the presence of a likely abnormal T cell clone as suggested by CD3+ count being less than the sum of CD4+ and CD8+ are all feature suggestive of L-HES.

131

Eosinophilic Disorders

15 Hypereosinophilic Syndrome Variants Associated variant of HES: In this IL-5-dependent variant of HES, the cause of hypereosinophilia is secondary to a well-defined allergic, immunologic, infectious, or neoplastic disease and that resolution is expected with the treatment of such underlying cause (Table  2). While it is arguable that these patients represent a separate entity, their clinical presentation can be indistinguishable from patients with idiopathic HES.  Among the common causes of the associated variant of HES are drug hypersensitivity, helminthic infection, sarcoidosis, and neoplasms, such as lymphoma. In the pediatric age group, HE can be secondary to atopic diseases, such as atopic dermatitis or allergic asthma, or can occur due to primary immunodeficiency disorders, such as hyper IgE syndrome, due to dominant-negative mutation in STAT3, CARD11, or recessive mutation in DOCK8. Other rare etiologies of associated HES variant are IgG4-related disease that may have overlapping clinical features with L-HES (see below) and sarcoidosis. Elevated IgE level is common to many of the secondary causes of HE and is a useful clinical clue, but resolution of hypereosinophilia with the treatment of the underlying cause is necessary to confirm the diagnosis of this variant of HES (Table 3). Myeloid Variant of HES (M-HES): In this variant of HES, excess production of eosinophils by the bone marrow is independent of IL-5 and results from clonal expansion of mature eosinophils. The World Health Organization (WHO) categorizes M-HES as chronic eosinophilic leukemia. Interstitial deletion in chromosome 4q12 that results in FIP1L1-PDGFRA rearrangement, a fusion protein with a constitutive tyrosine kinase activity or less common cytogenetic abnormalities, such as rearrangements involving the PDGFRB, FGFR1, or JAK2 genes, account for the clonal expansion of eosinophils in M-HES.  Clonal hypereosinophilia can also be observed in other myeloproliferative disorders, such as BCR-ABL1-positive chronic myeloid leukemia, Table 2  Causes of associated variant of HES Most common  • Helminthic infections (e.g., strongyloidiasis, toxocariasis, scabies)  • Drug hypersensitivity  • Neoplasms (e.g., lymphoma, solid tumors) Less common  • Atopic diseases especially in children  • Allergic bronchopulmonary aspergillosis  • Primary immunodeficiency disorders (e.g., omen syndrome, job syndrome, DOCK8 deficiency)  • Human immunodeficiency virus (HIV) infection  • Graft-versus-host disease  • Sarcoidosis  • IgG4-related disease  • Inflammatory bowel diseases

Table 3  Key features of the different HES variants Variant of HES Key features Associated Elevated IgE levels, resolves variant with treatment of the underlying cause Overlap Single organ involvement (often variant gastrointestinal and pulmonary), positive ANCA (in EGPA) Lymphoid Predilection for cutaneous variant involvement, abnormal T cell clone, elevated TARC, and IgE levels, possibly elevated IgM levels, episodic angioedema (in Gleich syndrome), risk of T cell lymphoma transformation Myeloid Male predominance, aggressive variant course, corticosteroid-resistant, cardiac/neurologic involvement, cytopenia, promyelocytes on peripheral blood smear, hepatosplenomegaly, elevated B12 levels, elevated tryptase levels Familial Positive family history, mostly variant asymptomatic (diagnosis of exclusion) Idiopathic No specific features, negative variant family history (diagnosis of exclusion)

Response Response to to steroid anti-IL5 N/Aa N/Ab

+++

+++

++

++

−/+



No data available

No data available

+++

++

Systemic glucocorticoid therapy may not be necessary Anti-IL5 therapy use and response will be dependent on the underlying cause a

b

polycythemia rubra vera, and systemic mastocytosis. M-HES has a male predominance, an aggressive clinical course with higher odds of cardiac and neurological involvement, and may be unresponsive to glucocorticoids. Akin to other myeloproliferative disorders, patients often have hepatomegaly, splenomegaly, and abnormalities in other cell lines, such as anemia, thrombocytopenia, and elevated serum vitamin B12 levels (Table 3). Higher serum tryptase level is noted in these individuals; however, levels are not as high as in systemic mastocytosis patients. Bone marrow biopsy with cytogenetic study is key to establish the diagnosis and to guide therapeutics. Lymphocytic or Lymphoid Variant (L-HES): In this IL-5-dependent variant of HES, there is an abnormal clone of T cells producing excess amount of IL-5 cytokine which drives the overproduction of eosinophils (i.e., clonal Th2 cells). The immunophenotype of such an aberrant IL5-­ producing T cell clone is variable, but CD3-CD4+ is the most common. Other reported immunophenotypes include CD3+CD4+CD7− and CD3+CD4−CD8− TCRαβ+. Unlike M-HES, L-HES equally affects men and women and has an indolent course with the prediction for skin and soft tissue involvements. Cutaneous manifestations are variable and

132

range from diffuse pruritus, urticarial plaques, and macular to maculopapular rash. Lymphadenopathy, gastrointestinal, and pulmonary involvement are also common in these patients. In keeping with the cytokine profile of this variant of HES, elevated IgE and TARC levels are observed in these patients who may additionally exhibit atopic diathesis (Table 3). A unique subset of the lymphocytic variant HES, characterized by cyclic angioedema and eosinophilia, is known as Gleich syndrome. Clonal expansion of CD32 + CD41+ T cells and elevated serum IgM levels are observed in this unique subset of L-HES patients. L-HES should be distinguished from IgG4-related disease, a rare cause of associated HES variant, that exhibits overlapping clinical phenotype, but the presence of elevated IgG4 levels and the absence of aberrant T cell phenotype support the former. Diagnosis of L-HES remains challenging. Lymphocyte subset analysis by flow cytometry may offer a clue to the presence of an abnormal T cell clone, such as CD3-CD4+, but T cell receptor rearrangement study remains necessary to detect the presence of T cell clonality. However, the sensitivity of such testing remains limited. While the clinical course of lymphoid variant HES is indolent in the majority of patients, it is complicated by those patients being at risk of developing T cell lymphoma Overlap Variant of HES: This is yet another IL-5-­ dependent HES variant in which hypereosinophilia exists in conjunction with a single organ eosinophilic disorder, such as eosinophilic gastrointestinal disorder, eosinophilic pneumonia, or eosinophilic skin diseases. Eosinophilic granulomatosis with polyangiitis (EGPA) is a unique subset of overlap HES, in which blood vessel involvement results in multisystem manifestations (Table 3). As these single organ eosinophilic disorders only occasionally manifest hypereosinophilia, the presence of such high levels of peripheral blood eosinophils makes them earn the “overlap” title. Because it is often difficult to rule out clonal eosinophilia in these patients, bone marrow biopsy is advisable, and biopsy of the involved organ is often necessary to establish a definitive diagnosis Familial Hypereosinophilia (HEFA) and Familial HES: This rare variant of HES is inherited in an autosomal dominant fashion (MIM 131400). Genome-wide linkage analysis mapped familial hypereosinophilia to chromosomal region 5q31q33 where a cluster of cytokine genes including genes for IL-5, IL-3, and GM-CSF are located. The levels of these cytokines including serum level of IL-5 are normal in these patients. Eosinophils from patients with familial hypereosinophilia exhibit normal morphology; however, there are increased serum levels of eosinophil granular proteins and increased surface expression of CD69, CD25, and HLA-DR, which are markers of eosinophil activation. The observed increased markers of eosinophil activation and serum eosinophil granular protein levels are less than what is observed in

T. Al-Shaikhly et al.

other variants of HES, which may explain why patients with familiar hypereosinophilia are often asymptomatic (Table 3). End-organ damage including endomyocardial fibrosis and neurological involvement has been reported in few affected family members. Factors associated with disease progression in this small subset of patients with familial hypereosinophilia remain unknown. Familial HES is a diagnosis of exclusion and should be considered when other family members report hypereosinophilia Idiopathic Variant of HES: A thorough evaluation to identify a cause for HES is non-revealing in over 50% of patients, and hence these cases are known as idiopathic HES. Idiopathic HES should be distinguished from HEUS, in which persistent hypereosinophilia exists without end-­ organ damage and without features to suggest familial or an alternative basis for hypereosinophilia such as HEN or HER. Watchful waiting in HEUS to identify early signs and symptoms of end-organ damage is necessary. Symptomatic patients with idiopathic HES require treatment to prevent progressive end-organ damage Question 3 Which of the following is a feature of L-HES? A. Elevated serum tryptase level B. Elevated serum B12 level C. Elevated serum TARC level D. Low IgM level Answers: C Elevated TARC level. In keeping with the type-2 cytokine profile of L-HES, elevated IgE and TARC levels are observed in these patients. Question 4 Which of the following is the best test to confirm the diagnosis of L-HES? A. Bone marrow biopsy and cytogenetic study for FIP1L1-­ PDGFRA rearrangement B. T cell receptor rearrangement study C. Serum IgG4 level D. Gain of function mutation in a gene that codes for CD113 Answer: B T cell receptor rearrangement study will assist in detecting the presence of T cell clonality, a unique feature of L-HES. Bone marrow biopsy and cytogenetic study for FIP1L1-­ PDGFRA rearrangement will assist in diagnosing M-HES. L-HES should be distinguished from IgG4-related disease, a rare cause of associated HES variant, that exhibits overlapping clinical phenotype, but the presence of elevated

Eosinophilic Disorders

133

IgG4 levels and the absence of aberrant T cell phenotype support the former. Gain of function mutation in a gene that codes for CD113 or c-kit mutation is associated with systemic mastocytosis.

sensitivity (i.e., HER) should be sought to prevent disease progression. This evaluation includes a careful history and directed laboratory testing. Particular attention should be paid to allergic disorders, travel, and drug history. Atopic diseases, such as allergic asthma and atopic dermatitis can Question 5 cause hypereosinophilia, especially in children (Table  2). In discussing treatment options for patients with HES, which Remote or recent travel history, especially to tropical regions of the following HES variants is least responsive to is relevant. Both prescription and nonprescription medicacorticosteroids? tions should be reviewed in relationship to disease onset and unnecessary medications should be discontinued. For those A. Lymphocytic variant HES where no obvious secondary cause for hypereosinophilia or B. Overleap variant HES if treatment of the presumed underlying cause failed to C. Myeloid variant HES resolve patient’s eosinophilia, evaluation of underlying D. Idiopathic variant HES myeloid or lymphocytic variant HE should be sought especially when there are clinical features to support these variAnswer: C ants (Table  3). Peripheral blood smears, serum levels of Myeloid variant HES can be resistant to steroids. All vitamin B12, and tryptase levels should be checked. The other variants are responsive to corticosteroids. presence of immature granulocytes, elevated B12, or tryptase levels may support the presence of a myeloproliferative process. Peripheral blood analysis for FIP1L1-PDGFRA by 16 Approach to Patients fluorescence in situ hybridization (FISH) or RT-PCR should with Hypereosinophilia and HES be considered. Bone marrow biopsy and cytogenetic studies may also be needed when the suspicion remains high. Patients presenting with hypereosinophilia should be evalu- Lymphocyte subset analysis by flow cytometry and T cell ated for the presence of end-organ dysfunction (Fig. 2). This receptor rearrangement may identify abnormal T cell clones, evaluation includes performing detailed history, physical typical of L-HES. In patients with an otherwise HEUS, watchexamination, and diagnostic testing. Diagnostic tests to eval- ful waiting with interval follow-up for end-organ damage uate for underlying end-organ damage may include electro- can be advised. cardiogram, echocardiograph, cardiac enzymes, chest x-ray In patients with evidence of end-organ damage, urgent and/or computerized tomography (CT) of the chest, pulmo- treatment is necessary for those with life-threatening maninary function testing, and liver and renal function tests. In festations, such as cardiac, pulmonary, or neurologic involvepatients with hypereosinophilia with no evidence of end-­ ment, and corticosteroids are often the first line of therapy organ damage, identifying and addressing an underlying (prednisone 1 mg/kg/day). Because systemic corticosteroids treatable cause such as helminthic infection or drug hyper- are associated with increased mortality in patients with Fig. 2  General approach to patients with hypereosinophilia. Abbreviations: AEC absolute eosinophil count, HES hypereosinophilic syndrome

AEC ≥ 1,500 cells/µL on 2 occasions 4 weeks apart (i.e., hypereosinophilia)

History, physical examination and laboratory testing to identify end-organ damange/ dysfunction No end organ damange Identify and treat obvious causes (e.g., drug hypersensitivty, helminthic infections)

Evaluate for myeloid variant and for abnormal lymphcyte clone

presence of end-organ damange = HES

Empiric treatement with corticosteroid +/- anti-helminthic

Work up to identify the underlying HES variant (Table 3)

Treatment per HES variant

134

strongyloidiasis, those with a travel history outside the United States (even if it was remote), should also receive empiric anti-helminthic therapy with a 2-day course of ivermectin. When the clinical suspicion of M-HES is high (Table 3), imatinib (± corticosteroids for those with cardiac involvement) should be given as a first-line agent. For HES patients without life-threatening features, or those initiated on empiric therapy, further treatment can be tailored toward the underlying cause of hypereosinophilia or the variant of HES. Treatment of associated HES variant is directed toward the underlying cause. In patients with suspected drug hypersensitivity, the offending drug should be discontinued, and in patients with helminthic infections, an anthelminthic agent, such as ivermectin, should be attempted. Neoplasms, such as lymphoma, should be treated according to established guidelines. If no response is observed with proper treatment of the presumed underlying cause of HE, alternative variants of HES should be considered. As discussed above, in the M-HES, there is clonal expansion of eosinophils that is independent of the IL-5 cytokine and is driven by acquired molecular abnormalities that drive eosinophil proliferation and expansion. The IL5-independent nature of M-HES explains why these patients are also resistant to anti-IL-5 therapy with mepolizumab or benralizumab. This survival advantage created by the underlying molecular abnormalities explains why these patients tend to be resistant to corticosteroid therapy and that M-HES should be suspected when high-dose corticosteroid fails to decrease the eosinophil count within 48 h. Alternatively, the approach to M-HES is dictated by the underlying cytogenetic abnormality and whether the HE is part of a WHO-defined myeloid neoplasm. In those with myeloid neoplasm with eosinophilia and abnormality of PDGFRA, PDGFRB, FGFR1, or JAK2, the resultant cytogenetic abnormality with intrinsic tyrosine kinase activity offers a therapeutic opportunity. For example, in the most commonly identified cytogenetic abnormality of FIP1L1-PDGFRA (F/P-positive M-HES), imatinib results in universal hematological and molecular remission in these patients. In some of the F/P-positive patients, a cure was attained (i.e., molecular-free remission even after cessation of imatinib). In those without cytogenetic abnormality, imatinib in high doses may be attempted, but standard treatment for acute myeloid leukemia should be offered. Patients with other WHO-defined myeloid neoplasms are treated according to the standard guidelines for these neoplasms, but this is beyond the scope of this chapter. Patients with lymphocytic and idiopathic variants of HES are both responsive to steroids, but relapse is common. Patients who fail to respond to corticosteroids, develop

T. Al-Shaikhly et al.

debilitating side effects, or require a prolonged course, would benefit from the use of a steroid-sparing agent, such as hydroxyurea, cyclosporine, azathioprine, INF-α, or more recently, mepolizumab. As a last resort, alemtuzumab, an anti-CD52 monoclonal antibody, may be considered. The relative safety profile of mepolizumab makes it an attractive steroid-sparing agent in these patients. In a sub-analysis of the international, randomized, double-blind, placebo-­ controlled clinical trial to investigate the efficacy of mepolizumab as a corticosteroid-sparing agent in F/P-negative HES, mepolizumab at a dose of 750 mg intravenously was found to be effective as a corticosteroid-sparing agent in these patients. Mepolizumab at a dose of 300 mg subcutaneously every 4  weeks is an FDA-approved treatment for patients with F/P-negative HES (>6  months) including patients with lymphocytic variant HES. A phase II study of benralizumab, anti-IL5 receptor alpha monoclonal antibody, resulted in clinical and hematological response in four patients with L-HES and six other patients with idiopathic variants; however, three patients with L-HES had relapse. Topical therapies for single organ eosinophilic disorders, such as eosinophilic gastrointestinal disorders with hypereosinophilia, may be insufficient, and systemic corticosteroids may be necessary. Patients with single organ eosinophilic disorders with HE are responsive to medium doses of corticosteroids (1.0 mcg/ml results in an excessive number of false positive results. Question 4 False positive results with insect testing with a non-culprit insect is most likely due to: A. Identical protein structures among insects. B. Enzyme activity of venom proteins. C. Carbohydrate determinants common to different insects. D. Specific IgG to venom antigens. Answer and Explanation Answer: C The most effective way of evaluating insect allergy is with skin testing, although false positive and negative results do occur. Specific-IgE antibodies for common carbohydrate epitopes shared among insects may result in multiple positive tests that are not clinically relevant. Many of the insects within the same taxonomy family will have venom antigen similarities, for example, between yellow jackets and hornets, but the venom components are not identical. Proteins such as hyaluronidase are common to multiple families, albeit the hyaluronidases do differ. It is preferable to distinguish between true sensitivity and cross-reactivity and treat with the relevant insect reagent. Thus, in some countries sting challenge is performed to verify the importance of a specific insect. However, the negative impact on patient comfort, patient and physician anxiety, liability, and logistics

166

(for example, access to stinging insects) limit the use of this strategy in clinical practice in the US. Another approach, not commercially accessible in the US but available in some other countries, is component resolved diagnosis (CRD) in cases of difficult interpretation of sensitization-based Vespula and Polistes specific IgE to whole venom. For example, testing with the purified component Ves v 5 and Pol d 5 may identify which is the culprit insect. Another test used to distinguish true and false positive results is in vitro inhibition of specific IgE or CAP-inhibition test (ImmunoCap®). It is a quantitative assay that measures inhibition of allergen-­ specific IgE in serum based upon the efficiency of one venom, mixed with the patient’s serum prior to the assay, to inhibit reactivity to another venom. Antibodies with greater avidity for a specific inhibiting venom suggest the inhibiting venom is from the culprit insect. IgG antibodies do not result in positive venom allergy testing. The enzymatic activity of venom components may be responsible for toxic reactions but not allergy. Question 5 Which of the following is an indication for lifelong venom immunotherapy? A. Wasp allergy B. Systemic reaction during venom immunotherapy C. Asthma D. Calcium channel blocker therapy of coexisting hypertension

D. S. Imanirad and D. Ledford

related exposure. Approximately 3  years of venom immunotherapy reduces the risk of significant anaphylaxis by more than 80%, and this protection endures for 3–5 years or longer following discontinuation of immunotherapy. Despite the persistence of a positive skin test result while on venom immunotherapy, about 80–90% of patients will not experience a systemic allergic reaction to a subsequent insect sting after 3–5 years of immunotherapy. Repeat skin testing is not required for consideration of stopping immunotherapy. Relapse is less likely with 5  years than 3  years of venom immunotherapy. The duration is not yet defined for immunotherapy with IFA whole-body extract. More favorable immunotherapy responses generally occur in children, those with mild or moderate systemic reactions and those with history of Vespula allergy. A shared decision-making discussion with individuals at greater risk of reoccurrence of sensitivity associated with anaphylaxis should be documented concerning lifetime immunotherapy. Honeybee sting anaphylaxis relapse is more likely, possibly due to the higher amount of venom injected by honeybees during a sting compared to other insects, such as wasps. Another reason may be that some honeybee allergens (e.g., Api m 3 and Api m 10) are of insufficient concentration in commercially purified venoms.

1 Summary

Systemic venom reactions following insect sting can be mild or life-threatening anaphylaxis, with involvement of Answer and Explanation two or more organ systems. Testing and venom immunoAnswer: B therapy is only indicated in case of a severe allergic reaction There are risk factors for post immunotherapy occurrence or anaphylaxis or risk factors for potentially fatal reactions of sting anaphylaxis. These factors include a life-threatening with future insect stings. The American Academy of Allergy reaction prior to initiating immunotherapy, systemic reac- and Immunology (AAAAI) and American College of tions to insect immunotherapy or to field stings during Allergy, Asthma, and Immunology (ACAAI) 2016 Joint immunotherapy, anaphylaxis to honeybee stings, and mast Task Force insect sting allergy practice parameter recomcell disorders (Table  3). Aside from these exceptions, mends that testing should be repeated in 3–6 months if ini3–5 years of venom immunotherapy would be sufficient to tially negative in patients with a high pretest probability of reduce the risk of life-threatening anaphylaxis from specific insect sting allergy. Baseline serum tryptase level should be Hymenoptera venom. Insect sting immunotherapy is an effi- considered in cases of severe allergic reactions, especially cient method of treating people with severe immediate sys- with hypotension without urticaria or with a convincing histemic reactions. The treatment gradually diminishes the tory and no detectable specific IgE.  An elevated baseline specific IgE to the treatment allergens via multiple mecha- tryptase may increase the risk of sting reactions. Insect sting nisms. Immunotherapy is performed with purified venoms testing is performed with commercially available purified from most insects of interest and whole-body extract from venoms, except whole-body extracts are used for IFA. LongIFAs. Insect sting immunotherapy is indicated if there is a term management of systemic insect allergy includes precompelling history of anaphylaxis following the sting and scription of epinephrine autoinjectors, instruction in the specific IgE identified to the culprit insect. Insect immuno- timing and technique of epinephrine use, venom immunotherapy will reduce the size of local reactions and may be therapy if relevant specific IgE is detected, instructions as to considered in subjects with incapacitating large local reac- when to seek emergency assistance, and education about tions and inability to avoid stings, for example, in a work-­ measures to avoid future stings.

Insect Allergy

Bibliography 1. Alfaya Arias T, Soriano Gómis V, Soto Mera T, Vega Castro A, Vega Gutiérrez JM, Alonso Llamazares A, et al. Key issues in hymenoptera venom allergy: an update. J Investig Allergol Clin Immunol. 2017;27(1):19–31. 2. Baker TW, Forester JP, Johnson ML, Stolfi A, Stahl MC. The HIT study: Hymenoptera identification test: how accurate are people at identifying flying insects? Ann Allergy Asthma Immunol. 2014;113(267) 3. Bilò MB, Tontini C, Martini M, Corsi A, Agolini S, Antonicelli L. Clinical aspects of hymenoptera venom allergy and venom immunotherapy. Eur Ann Allergy Clin Immunol. 2019;51(6):244–58. 4. Casale TB, Burks AW. Clinical practice. Hymenoptera-sting hypersensitivity. N Engl J Med. 2014;370(15):1432–9. 5. Castells M. Diagnosis and management of anaphylaxis in precision medicine. J Allergy Clin Immunol. 2017;140(2):321–33. 6. Golden DBK, Tabatabaian F, Müller-Gierok U, Lockey RF. Indications for and preparing and administering Hymenoptera

167 vaccines. In: Allergens and allergen immunotherapy. 6th ed. Boca Raton, FL: CRC Press/Taylor and Francis Group; 2020. p. 423–35. 7. Golden DBK. Insect allergy. In: Burks AW, Holgate ST, O’hehir R, Broide DH, Bacharier LB, Hershey GKK, Peebles RS, Middleton E, editors. Middleton’s allergy: principles and practice. Amsterdam: Elsevier; 2020. 8. Golden DBK, Demain J, Freeman T, Graft D, Tankersley M, Tracy J, et al. Stinging insect hypersensitivity: a practice parameter update 2016. Ann Allergy Asthma Immunol. 2017;118(1):28–54. 9. Hemmer W, Wantke F. Insect hypersensitivity beyond bee and wasp venom allergy. Allergol Select. 2020;4(01):97–104. 10. Niedoszytko M, Bonadonna P, Oude Elberink JNG, Golden DBK.  Epidemiology, diagnosis, and treatment of Hymenoptera venom allergy in mastocytosis patients. Immunol Allergy Clin N Am. 2014;34(2):365–81. 11. Ochfeld EN, Greenberger PA.  Stinging insect allergy and venom immunotherapy. Allergy Asthma Proc. 2019;40(6):372–5. 12. Tankersley MS, Ledford DK. Stinging insect allergy: state of the art 2015. J Allergy Clin Immunol Pract. 2015;3(3):315–22.

Latex Allergy Abeer Siddiqi and Anu Mallapaty

1 Introduction In this chapter, we will use a case-based approach to discuss the clinical manifestations, diagnosis, management, and clinical pearls of latex allergy. We will discuss the common presenting signs and symptoms followed by a discussion of the major allergens implicated in latex allergy. We will also review methods of diagnosis, describe latex-containing products, and discuss allergy to latexrelated foods.

2 Cases Case 1 A 35-year-old male, who works as a technician in a catheter-­producing plant, presents to your clinic with complaints of dyspnea, chest tightness, wheezing, and perennial rhinitis. Symptoms are worse on Fridays and improve over the weekend. Vitals include temperature of 99F, respiratory rate of 13 breaths per minute, heart rate of 76 beats per minute, blood pressure of 120/80 mm Hg, and oxygen saturation of 96% on room air. On physical examination, you note bilateral clear lung fields with no clubbing. Chest X-ray is unremarkable. You send the patient to obtain pulmonary function testing. Results are significant for an FEV1/FVC below 80% predicted, with a 20% improvement in FEV1 post albuterol.

A. Siddiqi · A. Mallapaty (*) Division of Allergy, Immunology and Rheumatology, Department of Pediatrics, Columbia University Irving Medical Center, New York, NY, USA Division of Pediatric Pulmonology, Allergy, Immunology & Sleep Medicine, Boston Children’s Health Physicians, Valhalla, NY, USA e-mail: [email protected]

Question 1 Which of the following types of allergen is likely implicated in this disorder? A. B. C. D. E.

Flour Metal Varnish Latex Wood dust

Explanation Answer: D While all the answers may be implicated in occupational asthma, it is essential to make the correlation with likely exposure at the workplace. Of these choices, latex is the most likely allergen in a catheter-producing plant. Latex or natural rubber latex (NRL) is derived from the tropical lactiferous rubber tree, Hevea braziliensis (of the botanical family Euphorbiaceae). Current reports indicate that every year over 12 million tons of NRL is produced. It is extracted as a fluid, with a milk-like consistency, by tapping the tree bark and is then used for the manufacturing of a wide assortment of medical and nonmedical materials and equipment. Only a limited number of these products are regulated by the FDA and several of these do not contain precautionary labeling. In the late 1980s, there were unexplained IgE-mediated immediate hypersensitivity reactions reported which included mild urticaria to angioedema and even life-­ threatening anaphylaxis. Later, these reactions were found to be likely attributed to trace amounts of NRL proteins present in a variety of materials. A decade later, in the 1990s, NRL proteins were also identified as potential airborne allergens causing rhinitis and occupational asthma through their binding to glove powder particles. Natural rubber latex (NRL) allergy reached an epidemic pattern during the late 1900s owing to the increased production and manufacturing methods of various NRL-containing

© The Author(s), under exclusive license to Springer Nature Switzerland AG 2022 M. Mahmoudi (ed.), Absolute Allergy and Immunology Board Review, https://doi.org/10.1007/978-3-031-12867-7_17

169

A. Siddiqi and A. Mallapaty

170

products, such as powdered gloves, condoms, and catheters. This condition was noted to be particularly prevalent in individuals with mucosal exposure to natural rubber latex (NRL), such as in those with spina bifida requiring repeated ureteral catheterizations with latex catheters, and in health care workers using natural rubber gloves. Health care workers developed occupational rhinitis and occupational asthma due to sensitization to NRL allergens. NRL-induced Occupational Asthma (OA) and Occupational Rhinitis (OR) were also seen in various other professions including hairdressers, food processors, and those who worked in greenhouses, glove manufacturing, laboratories, and the pharmaceutical industry. The reported data suggests that the prevalence of latex allergy in the general population is around 4.3%, with a higher prevalence seen in patients that are susceptible (e.g., 7.2% in patients with spina bifida and 9.7% in health care workers). Health care workers often present with symptoms of OA and OR as aforementioned, although there is a scarcity of reported literature correlating the incidence of asthma due to latex sensitization. Question 2 Which of the following allergens is the dominant sensitizer noted in this syndrome? A. Hev b 1 B. Hev b 2 C. Hev b 3 D. Hev b 6 E. Hev b 8 Explanation Answer: D Within highly purified or recombinant NRL, more than 15 allergens have been identified at a molecular level. Exposure to these proteins arises through airborne, percutaneous, or parenteral routes and is prevalent among individuals in frequent contact with natural rubber latex materials. In health care workers, airborne exposure is prominent and results from the attachment of latex proteins to dry lubricants that are used to facilitate donning of gloves, although sensitization rates have decreased with new manufacturing techniques. For patients, exposure is often parenteral and caused by leaching from latex materials used surgically. The pattern of IgE reactivity is dependent on the route of sensitization. Hev b 4, 5, 6.01, and 6.02 are the predominant sensitizers in health care workers. Hev b1, b3, and b5 are the immunodominant allergens for patients with spina bifida. The remaining allergens are considered to be minor.

The following table delineates these antigens. Allergen Hev b 1 Hev b 2 Hev b 3 Hev b 4 Hev b 5 Hev b 6 Hev b 7 Hev b 8 Hev b 9 Hev b 10 Hev b 11 Hev b 12 Hev b 13 Hev b 14 Hev b 15

Frequency of reactivity (%) 50–82 20–61 79 25–89 56–92 83 8–49 24 15 4

Biochemical name Rubber elongation factor Endo-1,3-beta-glucosidase Small rubber particle protein (SRPP) Lecithinase homologue Acidic protein Hevein precursor Patatin like protein Profilin Enolase Superoxide dismutase

3

Class I chitinase (PR-3)

24 63

Nonspecific lipid transfer protein type 1 (nsLTP1) Esterase

3–67

Hevamine

Unknown

Serine protease inhibitor

Question 3 How would you best approach diagnosing this patient? A. Latex skin testing with commercially available extracts B. A relevant medical history with Immunocap testing C. Patch Testing to Latex D. Strict avoidance of all potential workplace allergens E. Latex catheter challenge Explanation Answer: B There are various methods that may be employed in the evaluation of a patient with a likely allergic reaction to latex. In vitro assays for serum specific IgE antibody to latex are commercially available (e.g. Immunocap Assay). A relevant medical history and a positive Immunocap to latex at a level above 0.7 kU/L has a >98% specificity. However, it should be noted that 30% and 9% of asymptomatic atopic and non-­ atopic individuals respectively have positive ImmunoCap latex results. There are currently no commercially available latex skin testing extracts in the United States. Some allergists prepare their own extracts of unknown potencies by using latex gloves. One method involves soaking 2 fingers of a latex glove or a toy balloon in 5 mL of normal saline or buffer, and prick testing with the solution. This testing modality is not

Latex Allergy

standardized and can potentially expose the patient to high levels of latex protein. Patch testing to latex is not standard practice for diagnosing an IgE mediated latex allergy. Avoidance of all potential workplace allergens is unnecessary. A latex catheter challenge would be risky given history, and testing prior would be most prudent. Question 4 When counseling the patient regarding products to be avoided in the future, which of the following statements is accurate: A. Synthetic rubber should be avoided B. No need to strictly avoid latex C. Powdered natural rubber latex gloves may be used D. Avoid all products that may contain latex in the future E. Common household items such as balls do not contain latex anymore Explanation Answer: D Currently, the mainstay of therapy includes avoidance, which may significantly impact quality of life and employment for individuals. Patients should be counseled to decrease future exposure (e.g. avoidance of latex containing products in common household and personal care items including balloons, balls, condoms and gloves). In a latex-­ allergic patient who needs surgery, every provision should be made to obtain a latex-free operating suite to decrease the risk of anaphylaxis. Given the significant public health concern posed by NRL allergy, various policies have been put into place, including the recommendation made by the American Society for Testing and Materials to limit the amount of allergen protein to be less than 200 μg/m2 (approximately 50 μg/g of glove) for medical gloves, and substitution of powdered NRL gloves by gloves with a low content of protein and powder. Synthetic rubber can be safely used in a latex allergic patient. Question 5 What is the natural prognosis/treatment of this disorder? A. Patients will grow out of their latex allergy with continued exposure B. Unknown C. Patients maintain their sensitization to latex lifelong D. Sublingual immunotherapy is available for latex E. Subcutaneous immunotherapy is the gold standard for treatment of latex allergy

171

Explanation Answer: B The prognosis of this condition is still unclear. However, it is postulated that latex allergy may follow the same pattern as other drug allergens and result in decreased sensitivity over time as long as no further exposure. There are a few studies investigating specific immunotherapy for latex using latex allergen extract. Limited studies have shown that immunotherapy may reduce cutaneous and respiratory symptoms on re-exposure to natural rubber latex in IgE sensitized allergic individuals. However, the risk of systemic reactions remains high. Further studies in this area of research are required, and it is currently not routinely available. Case 2 A 4-year-old girl presents to the clinic with her grandmother. Grandmother reports that within 10 min of eating a spoonful of Kung Pao chicken from a new Chinese restaurant, her granddaughter complained of an itchy mouth and throat. No systemic involvement was noted and the symptoms resolved with one dose of an over-the-counter antihistamine. After reviewing the ingredients, chestnuts were the only item that the patient has not eaten and tolerated previously. You order specific serum IgE to chestnut and it is reported to be elevated. You suspect an oral allergy syndrome. Question 1 Which of the following allergens is known to be cross-­ reactive with chestnuts? A. B. C. D. E.

Latex Ragweed Birch Timothy grass Orchard grass

Explanation Answer: A Latex-fruit syndrome (LFS) is noted in approximately 30–50% of individuals that are sensitized to NRL allergens. An increasing number of plant sources have been associated with this syndrome including a variety of fruits, with the most commonly reported being avocado, banana, and kiwi. Potato, tomato, chestnut, olive, fig, herbs, and carrot have also been implicated. Clinical reactions to these foods vary from limited oral symptoms to life-threatening anaphylaxis. The diagnosis is based on clinical history. The current management includes avoidance of cross-reactive foods to which one has demonstrated clinical symptoms. Chestnut has not been reported to be cross-reactive with pollen protein.

172

A. Siddiqi and A. Mallapaty

Question 2 Which of the following are considered major pan-allergens responsible for Latex-fruit syndrome.

Question 4 What is the next best step:

A. Class 1 chitinases B. Class 2 chitinases C. Class 3 chitinases D. Class 5 chitinases E. Class 4 chitinases

A. Start latex immunotherapy for this patient B. Obtain a history inquiring about latex exposure and potential clinical reactivity C. Perform latex skin testing D. Add latex to patient’s allergy list E. Instruct patient to continue eating chestnut even if symptom provoking

Explanation Answer: A There are 14 families of Pathogenesis Related (PR) proteins, of which 6 account for most of the cross-reactivity. Class 1 chitinases, which belong to the PR-3 and PR-4 families, have been identified as major pan-allergens responsible for latex-fruit syndrome. These are cross-reactive with hevein (Hev), namely Hev b2, Hev b7, Hev b8, and Hev b12, which are major allergens in latex. Allergens of PR Protein Group 2 (β-1,3-glucanase proteins) are responsible for the latex-fruit cross-reactivity seen between latex (Hev b2) and avocado, banana, chestnut, fig, and kiwi. Class 2/3/4/5 chitinases are not major pan-allergens implicated in latex-fruit syndrome.

Explanation Answer: B Currently, there are no longitudinal studies determining the likelihood of development of latex allergy in patients with food reactions that have been implicated in this syndrome or vice versa. Theoretically, there may be an increased risk of an allergic reaction with latex exposure. However, there is no strong recommendation for or against the evaluation of latex allergy in people with oral symptoms to commonly implicated foods without a history of reaction to latex. Thus, obtaining a detailed history of latex exposure and reactivity is the best next step.

Question 5 Question 3 The patient is due for vaccinations and develops localized Which of the following allergen protein in Chestnut is cross-­ hives and facial swelling within 30  min of an injection. reactive with Latex allergen Hev b 6.02. Which of the following vaccines is known to contain latex. A. Bos d 5 B. Gal d 1 C. Can f5 D. Ara h2 E. Cas s 5

A. Influenza, Fluarix Quadrivalent B. Pneumovax 23 C. Hepatitis B, Engerix-B D. Zoster, Shingrix E. HPV, Gardasil-9

Explanation Answer: E Latex allergen Hev b 6.02 is similar to PR-3 type proteins in chestnut (Cas s 5) and avocado (Pers a 1), and PR-4 type proteins (wound-induced proteins) found in tomato and potato (Win 1 and Win 2). Below table is a summary of the cross-reactive allergens present in Latex and fruits:

Explanation Answer: C Of these choices, Hepatitis B, Engerix-B is the only one that is latex containing, specifically the tip cap of the prefilled syringe. Dry natural rubber latex can be present in tip caps, vial stoppers, and syringe plungers. In 2003, a review of over 167,000 reported reactions in the VAERS database revealed 28 immediate-type allergic reactions to a vaccine in patients also reported to be latex-allergic. Although rare, there have been a few case reports of allergic reactions presumed to be related to latex contamination by medications stored in containers with rubber stopper vials. In theory, storage of liquid in a vial containing dry natural rubber latex may result in contact and release of latex allergens into the solution. The US Food and Drug Administration (FDA) has regulations regarding the labeling of natural rubber-containing medical devices, but these rules have not been applied to medication

Latex Hev b2 Latex Hev b6.02 Latex Hev b2, Hev b7, Hev b8, Hev b12

Cross-reactive Proteins β-1,3 glucanase Cas s 5, pers a 1, win 1, win 2 Class 1 chitinases

Bos d 5 is a cow’s milk component, gal d 1 is a heat stable egg component, Can f 5 is a cat component, Ara h 2 is a peanut component.

Latex Allergy

vial stoppers. The Center of Disease Control has an updated list of latex-containing vaccine packaging on their website, which health care practitioners should be encouraged to refer to. Vaccine vial stoppers and syringe plungers made of synthetic rubber do not pose a risk to latex-allergic persons.

3 Conclusion Over the last decade, multiple interventions including the substitution of powdered latex gloves with either low-protein latex gloves or non-latex gloves have dramatically decreased the incidence of sensitization to latex, resulting in a decline in sensitization by more than 50%. Key Points • Latex allergy continues to be a significant public health concern, particularly in high-risk individuals and health care workers. • Patients with suspected latex allergy should be evaluated based on clinical history and demonstration of clinical sensitization on skin prick testing or serum Immunocap testing. • The primary treatment modality remains avoidance and counseling. • Latex-fruit syndrome is noted in 30–50% of patients sensitized to latex and is due to cross-reactivity between plant and latex allergens.

Acknowledgements  We wish to confirm that there are no known conflicts of interest associated with this publication and there has been no significant financial support for this work that could have influenced its outcome. All authors listed have contributed sufficiently to the project to be included as authors, and all those who are qualified to be authors are listed in the author byline.

173

Bibliography 1. Archambault S, Malo J, Infante-Rivard C, et al. Incidence of sensitization, symptoms and probable occupational rhinoconjunctivitis and asthma in apprentices starting exposure to latex. J Allergy Clin Immunol. 2001;107:921–3. 2. Kogevinas M, Zock JP, Jarvis D, et  al. Exposure to substances in the workplace and new-onset asthma: an international prospective population-based study (ECRHS-II). Lancet. 2007;370:336–41. 3. Vandenplas O, Froidure A, Meurer U, et  al. The role of allergen components for the diagnosis of latex-induced occupational asthma. Allergy. 2016;71:840–9. 4. Vandenplas O, Raulf M.  Occupational latex allergy: the current state of affairs. Curr Allergy Asthma Rep. 2017;17:14. 5. Bousquet J, Flahault A, Vandenplas O, et al. Natural rubber latex allergy among health care workers: a systematic review of the evidence. J Allergy Clin Immunol. 2006;118:447–54. 6. Siracusa A, Folletti I, Gerth van Wijk R, et al. Occupational anaphylaxis–an EAACI task force consensus statement. Allergy. 2015;70:141–52. 7. Hamborsky J, Kroger A, Wolfe S. Epidemiology and prevention of vaccine-preventable diseases. In: Centers for Disease Control and Prevention. Washington, DC: Public Health Foundation; 2015. 8. Primeau M-N, Adkinson NF, Hamilton RG.  Natural rubber pharmaceutical vial closures release latex allergens that produce skin reactions. J Allergy Clin Immunol. 2001;107:958–62. 9. Russell M, Pool V, Kelso JM, et al. Vaccination of persons allergic to latex: a review of safety data in the vaccine adverse event reporting system (VAERS). Vaccine. 2004;23:664–7. 10. Slater JE.  Latex allergy. J Allergy Clin Immunol. 1994;94: 139–49. 11. Matricardi PM, Kleine-Tebbe J, Hoffmann HJ, et  al. EAACI molecular allergology user’s guide. Pediatr Allergy Immunol. 2016;27:1–250. 12. Makatsori M, Durham S, Calderon MA.  Specific immunotherapy for latex allergy. Cochrane Database Syst Rev. 2017;2017(4):CD009240. https://doi.org/10.1002/14651858. CD009240.pub2. PMCID: PMC6478135 13. Nucera E, Mezzacappa S, Buonomo A, Centrone M, Rizzi A, Manicone PF, Patriarca G, Aruanno A, Schiavino D. Latex immunotherapy: evidence of effectiveness. Postepy Dermatol Alergol. 2018;35(2):145–50. https://doi.org/10.5114/ada.2018.75235. Epub 2018 Apr 24. PMID: 29760613; PMCID: PMC5949543

Drug Allergy Schuman Tam

Management of drug allergy is commonly encountered in the specialty of Allergy and Immunology. Primary care practitioners frequently request assistance from the allergist and immunologist in managing patients with drug allergy. It is essential that a trained allergy and immunology specialist be knowledgeable in the condition to assist our colleagues in caring for the patients. The following two clinical cases demonstrate the essential clinical principles one has to master to be a board-certified allergist and immunologist. Case 1 A 67-year-old businessman presented to your clinic with a history of allergic reactions following flu shot and a steroid shot. In 1995, he received a steroid shot (believed to be Kenalog) into his shoulder joint and developed hives within half an hour without breathing problem; since then, he has not had any steroid shot. In 2017, he received Fluzone quadrivalent vaccine at a healthcare facility. Within minutes, he developed hives and then dyspnea and was given epinephrine for relief; since then, he has not had a flu shot. Past medical history included sleep apnea, hypercholesteremia, benign prostatic hyperplasia, anxiety, mild intermittent asthma, shellfish-induced anaphylaxis, and mild reaction to certain fresh fruits. Current medications at the time of presentation included Atorvastatin, Tamsulosin, Albuterol HHN, and Cetirizine. Prior drug reaction included Typhoid vaccine (injectable) induced urticaria, IV dye contrast-induced flushing, injectable Fluzone Quadrivalent induced anaphylaxis, and injectable triamcinolone induced urticaria. His son has allergic rhinitis and his daughter has shellfish allergy. His

S. Tam (*) University of California, San Francisco, CA, USA Asthma and Allergy Clinic of Marin and San Francisco, Inc., Greenbrae, CA, USA e-mail: [email protected]

social history, surgical history, and review of system were noncontributory. Physical examination was normal. He presented to your clinic in the morning requesting your advice on whether he should receive the COVID-19 mRNA vaccine scheduled for the same afternoon. His primary doctor said that he might be allergic to an inactive ingredient present in Triamcinolone Acetonide. Since this inactive ingredient was present in the COVID mRNA vaccine, he sought your advice. You investigated the inactive ingredients present in Fluzone Quadrivalent, Triamcinolone acetonide, and Typhoid vaccine. Your research reviewed that Fluzone Quadrivalent contains egg protein, octylphenol ethoxylate (Triton X-100), sodium phosphate-buffered isotonic sodium chloride solution, thimerosal, and sucrose. Triamcinolone acetonide contains sodium chloride, benzyl alcohol, carboxymethylcellulose, and polysorbate 80. Typhoid vaccine contains Vi polysaccharide, sodium chloride, disodium phosphate, and monosodium phosphate. Question 1 You informed the patient that polysorbate 80 which is present in the triamcinolone acetonide can cross-react with polyethylene glycol in the COVID 19 mRNA vaccine. The polysorbate 80 is also present in the Janssen COVID 19 adenovirus vector vaccine. You further told the patient the following (choose the best answer): A. The Quadrivalent Fluzone contains no polyethylene glycol and no polyethylene glycol derivative B. The Quadrivalent Fluzone contains polyethylene glycol or its derivative Answer: B Triton X-100, present in the Quadrivalent Fluzone, is a poly(ethylene glycol) derivative that is poly(ethylene glycol) in which one of the terminal hydroxy groups has been converted into the corresponding p-(2,4,4-trimethylpentan-3-yl) phenyl ether (Fig. 1).

© The Author(s), under exclusive license to Springer Nature Switzerland AG 2022 M. Mahmoudi (ed.), Absolute Allergy and Immunology Board Review, https://doi.org/10.1007/978-3-031-12867-7_18

175

S. Tam

176

O

H

R

O

x

Octylphenol ethoxylate in Fluzone Polyethylene Glycol

H

O n

Polysorbate 80 O

OH

O

O

O

O O

O

O

O

O

O

O O

O

O

O

O

O

O

O O

O

OH

OH

OH

OH

Fig. 1  Chemical structures of OctylPhenol ethoxylate (Triton X-100), polyethylene glycol, and polysorbate 80. All three chemicals contain polyether which contains multiple (-CH2CH2-O-) in repeat sequences. Octylphenol ethoxylate contains p-(2,4,4-trimethylpentan-3-yl)phenylether at one of its terminals and H on the other end. Polyethylene glycol

contains polyether (multiple-CH2CH2O- in sequence) terminating with H on one end and OH on the other end. Polysorbate 80 contains polyether with 1 big chemical structure, which contains polyether, on one end and another big chemical structure, which also contains polyether groups on the other end

It is important in managing drug allergy that if one encounters an unknown chemical, it is critical to look up the chemical structure. Pubchem is a good sauce to look up unfamiliar chemicals. For looking up inactive drug ingredients, a reliable sauce is DailyMed. In this case, Fluzone contains octylphenol ethoxylate (Triton X-100). Octylphenol ethoxylate is a derivative of polyethylene glycol and is structurally similar to polyethylene glycol. If the patient’s prior allergic reaction to Fluzone was secondary to a reaction to Triton x-100, he may react to the COVID 19 mRNA vaccine since it contains polyethylene glycol. He may also react to the Jansen adenovirus COVID 19 vaccine since Triton X-100 is also structurally similar to polysorbate 80.

D. Expanded skin testing using medications containing polyethylene glycol and polysorbate

Question 2 You discussed with the patient the risk and benefits of getting the COVID-19 mRNA vaccine. With his reliable history of Type I allergy (hives and dyspnea) to Fluzone Quadrivalent (containing a derivative of polyethylene Glycol) and Triamcinolone acetonide (containing polysorbate 80 which is similar to polyethylene glycol), you recommended: A. Routine vaccination with 15 min observation as the benefit of getting the COVID-19 mRNA vaccination is higher than the risk of potential allergic reaction B. Routine vaccination with 30 min observation because of medium risk of allergic reaction to COVID-19 mRNA vaccine C. Avoid COVID-19 vaccination even though the patient insists on getting the COVID-19 vaccination

Answer: D Expanded skin testing using medications containing polyethylene glycol and polysorbate. The benefit of getting the COVID-19 vaccine is to reduce the risk of mortality if he acquires the COVID-19 infection. In public health standpoint, the vaccination may help to prevent the spread of COVID-19 infection in the community. The risk, based on history, is high in that he had a typical immediate allergic reaction to the Fluzone quadrivalent vaccine containing polyethylene glycol derivative and triamcinolone acetonide containing polysorbate 80. In this situation, skin testing may be helpful as a negative response may indicate that the patient may be able to tolerate the COVID-19 vaccine. Although skin testing using medications containing polyethylene glycol and polysorbate is not standardized, a positive result using a recommended concentration of the drugs suggests the presence of polyethylene glycol or polysorbate IgE antibodies. A negative skin test result does not rule out an allergy. Care should be taken when administering the COVID vaccine with 30 min of observation and be prepared to treat anaphylaxis. A positive skin test response, together with a history of prior allergic reaction to the drug, suggests a true allergic reaction and the COVID vaccine should not be given in the usual manner. The skin test was performed urgently this morning before the patient’s scheduled COVID-19 mRNA vaccination in the afternoon. The skin test showed a negative response to both

177

Drug Allergy

the polyethylene glycol and polysorbate 80. You recommended the patient to proceed to get the vaccine with 30 min observation. The vaccine center in your hospital however declined to have the patient vaccinated in your clinic because of logistic reasons in getting the vaccine to you. You sent the patient to the vaccination center for administration. Question 3 You received a call from the vaccination center that the patient developed flushing, diffuse urticaria, and shortness of breath. You rushed to the vaccination center. The patient was in supine position as he was feeling dizzy. He was able to speak with you but with a respiratory rate of 32. You could not detect a radial pulse but his femoral pulse was present. You would administer: A. 0.3  mg Epinephrine subcutaneously into patient’s right upper arm B. 0.3 mg Epinephrine intramuscularly into the patient deltoid muscle C. 0.3 mg Epinephrine IV push D. 0.3  mg Epinephrine intramuscularly into patient’s anterior-­lateral mid-thigh E. Either 0.3 mg Epinephrine intramuscularly into patient’s anterior-lateral mid-thigh or 0.3 mg intramuscularly into patient’s deltoid muscle, depending on how the patient was dressed

site produces a higher and earlier peak in plasma epinephrine level than the earlier site. The greater absorption of epinephrine from the vastus lateralis muscle in comparison with the deltoid muscle is most likely due to the greater blood flow in the vastus lateralis. When epinephrine is injected into the vastus lateralis, peak serum epinephrine can be achieved in 10 min and the peak plasma epinephrine level is five times higher than when injected intramuscularly into the deltoid muscle and is three times higher than that achieved by injecting subcutaneously into the upper arm. In anaphylaxis due to medication or other causes, the appropriate site of injection for epinephrine is intramuscularly into the anterior-lateral mid-thigh. Intramuscular injection is also a preferred route compared to subcutaneous route in pediatric patients with anaphylaxis. Epinephrine IV push should not be given to a patient who is not having cardiac arrest. After appropriate administration of epinephrine, the patient improved and was transferred to the Emergency Department for further observation and treatment. The patient also received antihistamine and systemic steroid. Subsequently, he was discharged home.

Question 4 The patient returned to see you 2 months later. He said that his hospitalist obtained a serum tryptase about 2 h after the onset of his anaphylaxis to the COVID vaccine; the value was 8.2 ng/ml. His primary doctor obtained a serum tryptase Answer: D a month after his anaphylaxis and the number was 5.1 ng/ml. Intramuscularly into the patient’s anterior-lateral mid-­ You explained to the patient that: thigh. Table 1 shows that peak plasma epinephrine concentration is significantly higher after epinephrine is injected A. Since both numbers were 95%. The concentration of penicilloyl-polylysine, commercially available as Pre-Pen, utilized for both prick and intradermal skin test, is 6 x 10 -5 M (or 0.00006 M or 0.00006 mol/L). Therefore, B and D are incorrect. The standard concentration utilized for penicillin G prick and intradermal skin testing is 10,000 units per ml. Since the patient gave a history of allergic reaction to Amoxicillin, amoxicillin should be included in the skin testing panel. If this patient is selectively sensitive to amoxicillin and not penicillin (specific IgE antibody is directed against the R1 side chain (see Fig. 4), skin test can be positive for Amoxicillin and negative for Pre-Pen and penicillin G.  In this situation, the patient is allergic to the Amoxicillin, but is

183

able to tolerate penicillin. It is important to point out that the skin test for amoxicillin has not been validated. Therefore, oral challenge using amoxicillin is required if skin test is negative for amoxicillin, Pre-Pen, and penicillin G, in order to rule out amoxicillin allergy. The result of the skin test showed that the patient is sensitive to Amoxicillin by prick skin test but negative by both prick and intradermal methods for Pre-Pen and penicillin G. You challenged the patient orally with penicillin VK and showed tolerance. You determined that the patient is likely allergic to amoxicillin based on her past history of reaction and current result of skin test. Question 10 You advised the patient and her primary physician that the patient can take penicillin and should avoid amoxicillin. If she requires cephalosporin urgently in the future and skin test is not readily available, graded test dose administration of the appropriate cephalosporin can be given except the following cephalosporin: A. Ceftazidime, ceftriaxone, cefuroxime B. Cefadroxil, Cefprozil, Cefatrizine C. Cefaclor, cephalexin, cephradine D. A and B E. B and C Answer: E Cefadroxil, cefprozil, and cefatrizine share an identical R group (Fig.  4) with Amoxicillin. Cefaclor, cephalexin, and cephradine share a similar R group with Amoxicillin. Therefore, these antibiotics may have a higher risk of causing an allergic reaction in the patient; thus, test dosing protocol is not appropriate for this patient. However, the R groups of ceftazidime, ceftriaxone, and cefuroxime are dissimilar to the R group of Amoxicillin; test dosing protocol is appropriate as the risk of reaction is low. You informed the patient that her reaction to Allopurinol was likely Steven Johnson Syndrome or Toxic Epidermal Necrolysis. Testing for this reaction is not available and desensitization does not work for this drug and is considered dangerous. You were seeing this patient with a medical resident. Question 11 You asked your medical resident which genetic risk factor is associated with the development of Steven Johnson syndrome after taking Allopurinol: A. B. C. D.

HLA-B*13:01 HLA-B*15:02 HLA-B*58:01 HLA-B*57:01

S. Tam

184 Table 4  Human leukocyte antigen association with serious drug allergy syndrome. The table lists the five drugs that have HLA association with different types of serious drug allergy syndromes Drug Abacavir Allopurinol Carbamazepine Dapsone Vancomycin

HLA allele B*57:01 B*58:01 B*15:02; A*31:01 B*13:01 A*32:01

Drug allergy syndrome Abacavir hypersensitivity syndrome SJS/TEN/DRESS SJS/TEN/DRESS DRESS DRESS

Abacavir hypersensitivity syndrome: fever, malaise, nausea, diarrhea, skin rash DRESS drug reaction with eosinophilia and systemic symptoms, HLA human leukocyte antigen, SJS Stevens-Johnson syndrome, TEN toxic epidermal necrolysis

E. HLA-A*32:01 Answer: C HLA-B*58:01 is a risk allele associated with Allopurinol induced Steven Johnson syndrome, TEN (Toxic epidermal necrolysis), and DRESS (drug reaction with eosinophilia and systemic symptoms). HLA-B*13:01 is associated with Dapsone-induced DRESS. HLA-B*15:02 is associated with carbamazepine-induced Steven Johnson Syndrome, TEN, and DRESS.  HLA-B*57:01 is associated with Abacavir hypersensitivity syndrome. HLA-A*32:01 is associated with Vancomycin-induced DRESS.  One may want to commit Table 4 into memory for the board examination.

Bibliography 1. Banerji A, Wickner PG, Saff R, Stone CA, Robinson LB, Long AA, Wolfson AR, et al. mRNA Vaccines to Prevent COVID-19 disease and reported allergic reactions current evidence and suggested approach. J Allergy Clin Immunol. 2020;9(4):1423–37.

2. Broyles AD, Banerji A, Barmettler S, Biggs CM, Blumenthal K, Brennan PJ, Breslow RG, et  al. Practical guidance for the evaluation and management of drug hypersensitivity: specific drugs. J Allergy Clin Immunol. 2020;8(9S):S16–S122. 3. Broyles AD, Banerji A, Castells M. Practical guidance for the evaluation and management of drug hypersensitivity: general concepts. J Allergy Clin Immunol. 2020;8(9S):S3–S15. 4. DailyMed. NIH National Library of Medicine. https://dailymed. nlm.nih.gov/dailymed/. Accessed 7 Mar 2021. 5. Dreskin SC, Stitt JM.  Anaphylaxis. In: Burks AW, Holgate ST, O’Hehir RE, Broide DH, Bacharier LB, Hershey GKK, Peebles RS, editors. Middleton’s allergy principles and practice. Philadelphia, PA: Elsevier Inc.; 2020. p. 1228–46. 6. Konvinse KC, Trubiano JA, Pavlos R, James I, Shaffer CM, et al. HLA-A*32:01 is strongly associated with vancomycin-induced drug reaction with eosinophilia, and systemic symptoms. J Allergy Clin Immunol. 2019;144:183–92. 7. Mustafa SS, Ransey A, Staicu ML. Administration of a second dose of the moderna COVID-19 vaccine after an immediate hypersensitivity reaction with the first dose: two case reports. Ann Intern Med. 2021;174(8):1177–8. https://doi.org/10.7326/L21-­0104. Online ahead of print 8. PubChem. NIH National Library of Medicine. National Center for Biotechnology Information. https://pubchem.ncbi.nlm.nih. gov/#query=. Accessed 7 Mar 2021. 9. Simons FE, Roberts JR, Gu X, Simons KJ. Epinephrine absorption in children with a history of anaphylaxis. J Allergy Clin Immunol. 1998;101:33–7. 10. Simons FE, Gu X, Simons KJ.  Epinephrine absorption in adults: intramuscular versus subcutaneous injection. J Allergy Clin Immunol. 2001;108:871–3. 11. Solensky R, Phillips EJ. Drug allergy. In: Burks AW, Holgate ST, O’Hehir RE, Broide DH, Bacharier LB, Hershey GKK, Peebles RS, editors. Middleton’s allergy principles and practice. Elsevier Inc.; 2020. p. 1261–82. 12. Tam S.  Chemotherapy and biologic drug allergy. In: Mahmoudi M, editor. Allergy and asthma. Cham: Springer; 2019. https:// doi.org/10.1007/978-­3-­030-­05147-­1_24. Print ISBN: 978-3-030-­ 05146-4. Online ISBN: 978-3-030-05147-1. 13. Tam S. Drug allergy. In: Mahmoudi M, editor. Allergy and asthma, practical diagnosis and management. Switzerland: Springer International Publishing; 2016. p. 407–26.

Autoimmunity Jack Jeskey, Lauren Fill, Madiha Huq, Sandeep Sarkaria, Remie Saab, and Robert Hostoffer

Abbreviations

1 Introduction

ACLE ANA BMI BP BP180 BP230 CCLE CNS DIF DLE EM IgE IIF IVIG LE LP MAC PMLE PV SCLE SLE

Autoimmune disorders arise from a failure of self-tolerance, or immunologic unresponsiveness to an individual’s own antigens, in genetically susceptible individuals. The mechanisms that result in tissue damage in autoimmune diseases parallel the normal responses of adaptive immunity and may include autoantibodies, immune complexes and/or autoreacting T lymphocytes. The clinical manifestations of autoimmune disorders are extremely varied but tend to be long-lasting and progressive. Immune responses may be directed against a single tissue resulting in organ specific disease such as bullous pemphigoid (BP). In contrast, some autoimmune disorders target a widespread antigens resulting in diseases like systemic lupus erythematous (SLE). We aim to introduce the reader to a better understanding of some common autoimmune diseases, as well as appropriate treatment approaches.

Acute cutaneous lupus erythematous Anti-neutrophilic antibody Body mass index Bullous Pemphigoid Bullous Pemphigoid Antigen 180 Bullous Pemphigoid Antigen 230 Chronic cutaneous lupus erythematosus Central nervous system Direct Immunofluorescence Discoid lupus erythematous Erythema Multiforme Immunoglobulin E Indirect Immunofluorescence Intravenous immunoglobulins Lupus erythematous Lichen planus Membranolytic attack complex Polymorphic light eruption Pemphigus Vulgaris Subacute cutaneous lupus erythematous Systemic lupus erythematous

J. Jeskey · L. Fill · M. Huq · R. Saab · R. Hostoffer Division of Pulmonary, Critical Care and Sleep Medicine, UH Cleveland Medical Center, University Hospital’s Pediatric and Adult Allergy/Immunology Fellowship, Cleveland, OH, USA S. Sarkaria (*) Division of Pulmonary, Critical Care and Sleep Medicine, UH Cleveland Medical Center, University Hospital’s Pediatric and Adult Allergy/Immunology Fellowship, Cleveland, OH, USA Allergy/Immunology Associates, Inc., Mayfield Heights, OH, USA

Case 1 An 80-year-old male with a past medical history of hypertension and atrial fibrillation presented to the outpatient clinic for evaluation of a new rash. His rash had started 3 months prior, with small punctate lesions scattered over his body. Initially, the patient went to the emergency department and was given a course of oral prednisone, which did improve his symptoms for a brief time. He tried topical over-the-counter moisturizers which did not help. He saw dermatology 1 month after his rash started and was given a topical steroid cream which did not improve his symptoms. At the time of presentation, he had blisters extending from his neck to his toes. He reports that the blisters would burst and ooze a clear liquid. He denied any new household or personal products (Fig. 1). Further subjective history was negative for any autoimmune conditions to the patient’s knowledge. Family history was also unremarkable. The patient was a nonsmoker. His vitals identified a healthy body mass index (BMI) (23.7, normal range [NR] 18.5–24.9 kg/m2). The physical

© The Author(s), under exclusive license to Springer Nature Switzerland AG 2022 M. Mahmoudi (ed.), Absolute Allergy and Immunology Board Review, https://doi.org/10.1007/978-3-031-12867-7_19

185

186

J. Jeskey et al.

C. Mucous membrane pemphigoid D. Bullous pemphigoid E. Lichen planus

Fig. 1  Picture of the patient’s skin lesion

examination revealed an erythematous, macular rash with thin-walled blisters. No evidence of oral mucosal lesions or ocular findings. At his evaluation, the patient had a skin biopsy revealing subepidermal vesicles containing a dense infiltrate of inflammatory cells including eosinophils. Direct and indirect immunofluorescence (DIF and IIF) studies revealed linear deposition of IgG and C3 along the basement membrane. Question 1 Based on the biopsy results the most likely diagnosis would be: A. Erythema multiforme B. Pemphigus vulgaris

Answer: D The patient in this case is presenting with bullous pemphigoid, an autoimmune disorder characterized by blistering (Table 1). It presents with urticarial and eczematous lesions on the trunk and upper legs that progress to tense bullae. The most common age of onset for BP is 80-years old; however, drug-induced pemphigus can also be seen in the pediatric population, and young adults. BP is the most frequently encountered autoimmune blistering disease and is caused by autoantibodies directed against the hemidesmosomal proteins, BP antigen 180 (BP180) and BP antigen 230 (BP230). The diagnosis can be made with a perilesional biopsy with staining and direct immunofluorescence. A skin biopsy demonstrates subepithelial vesicles with mixed inflammatory infiltrate containing eosinophils. DIF microscopy shows linear IgG and/or C3 staining along the basement membrane zone. Erythema Multiforme (EM) is an acute inflammatory reaction of the skin, with rare involvement of the mucous membranes, that presents as erythematous macules that classically evolve into targetoid lesions on the extensor extremities. It represents a cell-mediated immune reaction associated with infections (mainly Herpes Simplex Virus), drugs, vaccinations, and autoimmune disorders. The histological features in EM are nonspecific with perivascular inflammation, interface dermatitis, and epidermal necrosis. Unlike BP, DIF is nonspecific and IIF would be negative, making this answer choice incorrect.

Table 1  Characteristics of common blistering skin disorders Disease Erythema Multiforme

Clinical features Sudden onset. Variable lesions: Erythematous macules, target lesions, vesicles. Located on extensor extremities → spread centripetally. Mucous membranes are sometimes involved No sudden onset. Painful, flaccid bullae with erosions on skin and mucosa. Nikolsky sign is positive

Histology Autoantibody target Nonspecific. Perivascular Nonspecific inflammation, interface dermatitis, epidermal necrosis with intra and subepithelial vesicular formation

Immunofluorescence Nonspecific

Intraepithelial vesicles with acantholytic keratinocytes, intact basal layer

Intercellular, reticulated IgG and C3

Mucous membrane pemphigoid

No sudden onset. Vesiculoulcerative lesions involving the mucosa only. May lead to scarring

Perivascular inflammation with subepithelial vesicles

Bullouspemphigoid

No sudden onset. Pruritic, tense Perivascular infiltrate with bullae with rare mucosal involvement eosinophils and subepithelial vesicles No sudden onset. Pruritic, flat topped Hyperkeratosis, papules and plaques with involvement hypergranulosis, interface of the skin (flexor extremities), hair, dermatitis, saw-toothed nails, and mucous membranes epithelial vacuolation with apoptosis

Pemphigus vulgaris

Lichen planus

Epidermal desmosomes (Desmoglein-1 and Desmoglein-3) BP230, BP180, α6-integrin, Laminin 5, Laminin 6, b4-integrin, type 7 collagen BP230, BP180

None/unknown

Linear IgG and IgA along basement membrane

Linear IgG and C3 along basement membrane Fibrin deposits along the basement membrane

Autoimmunity

Pemphigus Vulgaris (PV) is a chronic, autoimmune blistering disease of the skin and mucous membranes. In contrast to BP, PV presents at an earlier age (40–60 years old) with painful, flaccid bullae that rupture easily to form erosions. In addition, the mucosa is almost always involved compared to BP. The primary defect in PV is IgG autoantibodies directed against keratinocyte adhesion proteins resulting in loss of cell adhesion or acantholysis. In turn, DIF demonstrates intercellular deposits of IgG in a reticular pattern around keratinocytes making this an incorrect answer choice. Mucous Membrane Pemphigoid is a rare, antibody-­ mediated blistering disorder that only affects the mucous membranes, making this answer choice incorrect. In addition, although DIF would demonstrate linear bands of IgG deposited along the basement membrane, IIF is usually negative. Lichen planus (LP) is an idiopathic inflammatory disorder of the skin. The skin lesions of LP are characterized by the “6P’s” including purple, pruritic, planar, polygonal, papules, and plaques with an overlying reticulated, fine scale known as Wickham striae. The wrist and ankles are common sites of involvement, but any location may be affected including the hair, nails, and mucous membranes. Characteristic histological features of LP include a band-like lymphocytic infiltrate along the dermal-epidermal junction, saw-toothed epithelial vacuolation, and apoptosis with a thickened granular cell layer and stratum corneum making this an incorrect answer choice. In addition, LP will have a negative IIF.

187

vous system (CNS) and skin. This may be the common feature which connects the clinical manifestations of BP to the CNS. It is postulated that any insult to the CNS can trigger increased levels of anti-BP180. Increases in these levels have been found to correlate with the severity of dementia in patients with Alzheimer’s disease. There is conflicting data regarding BP association with malignancy. Two Japanese studies found higher rates of malignancy in their BP subjects when compared to age-­ matched controls. One study demonstrated a rate of 5.8% of their BP subjects with malignancies including lymphoma, gastric, colorectal, prostate, lung, and uterine cancers. In addition to its association with neurologic disorders and malignancy, BP also has been associated with an increased thrombotic risk. BP promotes a dysregulated immune response mediated by Th1 and Th2 cells resulting in an increased synthesis of IL-1B, TNF-α, IL-5, IL-6, IL-8, IL-10, and IL-15. The production of these pro-inflammatory cytokines upregulates vascular endothelial growth factor and E-selectin which results in endothelial cell activation. These patients also have been found to have increased circulating levels of prothrombin and D-dimer as well as overexpression of tissue factor in lesional skin. These levels have been shown to return to normal with disease control. Together this evidence suggests a prothrombotic state exists in the BP patient. This may lead to a higher risk of thromboembolic events, including pulmonary embolism and stroke in comparison to age-matched controls.

Question 2 What are possible complications of bullous pemphigoid?

Question 3 Which of the following would be the first-line treatment in a patient diagnosed with mild to moderate BP?

A. Neurologic disorders B. Malignancy C. Thrombosis D. A and B E. All the above

A. Doxycycline B. Low potency topical corticosteroid C. High potency topical corticosteroid D. Oral prednisolone E. Methotrexate

Answer: E There have been studies that have demonstrated an association between BP and neurologic disorders. A recent ­systematic review with meta-analysis evaluated 14 studies. Results of the analysis indicated that individuals with BP were five times more likely to develop neurologic disorders such as dementia, epilepsy, multiple sclerosis, Parkinson’s disease, and stroke. This review also found that the neurologic disorder typically precedes the onset of BP by 5.5 years. The most common associated neurologic disorder is multiple sclerosis with a 5–12 time risk of development of BP. Unfortunately, the pathogenesis linking BP and neurologic disorders is still not completely understood. Another study found that bullous pemphigoid antigen (BP180 and BP230) are expressed both in the central ner-

Answer: C BP is a chronic disease that can persist for many years with a risk of relapse. The main purpose of treatment is to promote healing of the skin lesions as well as to decrease itching and prevent recurrence to improve patients’ quality of life. The first-line treatment in BP depends on the disease severity and spread. For localized and moderate forms of BP the current first-line treatment consists of super potent t­ opical corticosteroids (e.g., clobetasol propionate). The effectiveness of topical clobetasol propionate cream was proven in extensive BP with less mortality and side effects when compared with oral prednisone in a randomized control trial. Limitations of extensive topical steroid use include skin atrophy and difficult application for the elderly person.

J. Jeskey et al.

188

In the case of extensive and advanced BP, systemic corticosteroids are considered the first line of treatment. Other alternative therapies may be considered to counteract the systemic effects of oral steroids. For patients who fail to respond to corticosteroids or who develop side effects, other therapeutic options are available. Doxycycline has proven effective as an alternative to steroids. In a randomized control trial of 253 patients with BP, Doxycycline was noninferior to oral prednisolone for short-­ term blister control. The patient was started on an oral prednisone taper and empiric mycophenolic acid (Cellcept) 750  mg twice daily for treatment for BP. Despite starting mycophenolic acid, the patient had persistent symptoms and reported increased pruritus. He completed two 10-day prednisone tapers and started hydroxyzine 10  mg three times daily as needed for pruritus. There was minimal improvement in his rash and his blisters were in various stages of healing. After a 26-day trial of mycophenolic acid, the patient elected to stop treatment due to persistence of his rash. Alternative treatment options were considered and discussed with the patient. Question 4 Which of the following would be the next best step in management for a patient who has failed corticosteroids and corticosteroid-sparing therapy? A. Intravenous immunoglobulins (IVIG) B. Cyclosporine C. Methotrexate D. Doxycycline E. Topical urea Answer: A Addition of an oral immunosuppressive agent (cyclosporine, methotrexate, mycophenolic acid. Etc.) can be considered with severe disease. However, in cases refractory to corticosteroids and corticosteroid-sparing therapy, biological therapies should be considered. Various biologic therapies can be used including rituximab, omalizumab, dupilumab, and IVIG (Table  2). Therefore, the next most appropriate choice in treatment in this case would be IVIG.  Multiple treatment cycles are typically needed for disease improvement. Studies have shown that IVIG leads to a decline in serum levels of BP180 and BP230 antibodies. The patient was started on 70 g of IVIG daily for 2 consecutive days to treat his resistant disease state. Over the next 6  weeks, he had near complete resolution of his rash and associated symptoms. Monthly IVIG infusions were continued for disease maintenance.

Table 2  Biologic therapies available for refractory BP

Biologic agents Rituximab Omalizumab Dupilumab Intravenous immunoglobulin (IVIG)

Target CD20 Immunoglobulin E (IgE) IL-4 and IL-13 BP180 and BP230 antibodies

Average response rate based on case series (%) 85 84 92 86

Question 5 By what mechanism does IVIG affect autoimmune disorders in general? A. Modulation of pathogenic autoantibodies B. Inhibition of complement activation and interception of membranolytic attack complex (MAC) formation, an action relevant to the complement-mediated mechanisms C. Modulation of the inhibitory or activation Fc receptors on macrophages invading targeted tissues D. Downregulation of pathogenic cytokines and adhesion molecules E. All the above Answer: E Many mechanisms have been suggested to account for the beneficial action of IVIG in autoimmune and inflammatory disorders including blockade of Fc receptors on cells of the reticuloendothelial system, the neutralization of pathogenic autoantibodies, and the attenuation of complement-mediated tissue damage. The major role of IVIG in these disorders appears to be through its effects on Fc receptors. FcRn blockade leads to an accelerated neutralization and clearance of autoantibodies. The blockade of activating FcγRIII results in reduced opsonization of antigens and decreased pro-­ inflammatory responses from innate effector cells. Furthermore, sialylated IgG exerts anti-inflammatory effects by upregulated inhibitory FcγRIIB on macrophages. Still other mechanisms of IVIG may include the downregulation of pro-inflammatory genes in macrophages, modulation of dendritic cell maturation and function, and inhibition of lymphocyte autoreactivity. The pleiotropic role and effect of IVIG on autoimmune disease such as BP has yet to be completely elucidated. Case 2 A 35-year-old female presents to the outpatient clinic for evaluation of a new skin rash. The rash started 4 months prior with scaling, erythematous lesions on the upper trunk. She was seen at a local urgent care clinic and was given an oral

Autoimmunity

189

C. Psoriasis D. Lichen planus E. Polymorphic light eruption Answer: B Lupus erythematous (LE) is a multisystemic autoimmune disorder characterized by the formation of autoantibodies that cause tissue injury mainly through the deposition of immune complexes as well as binding of antibodies directly to cells. Cutaneous manifestations are frequently the initial presentation of LE and can occur with or without systemic disease. LE-specific skin disease includes acute cutaneous lupus erythematous (ACLE), subacute cutaneous lupus erythematous (SCLE), and chronic cutaneous lupus erythematosus (CCLE). Although all types share similar histologic features like interface dermatitis and deposits of immunoglobulin and complement, they differ in clinical presentation, association with systemic disease, and long-term complications. The clinical vignette above describes a case of SCLE. SCLE occurs most common in middle aged females. Fig. 2  Picture of the patient’s skin lesion Two morphologic variants of SCLE exist: annular and papulosqamous. In the annular variant, scaling, erythematous, prednisone taper with temporary resolution of her symp- plaques with central clearing coalesce to form polycyclic toms. The lesions have since progressed with involvement of configurations. The papulosqamous variant resembles plaque the extremities. She has been working as a lifeguard for the psoriasis. SCLE is a photosensitive rash and exacerbations past year and notes the lesions are exacerbated by the sun. occur with sun exposure. In turn, the most affected areas are She denies any symptoms other than occasional pruritis. Her sun exposed skin like the neck, upper trunk, and extensor medical history is significant for Hashimoto’s thyroiditis for portion of the upper extremities; however, the face is usually which she takes levothyroxine. She has no known allergies. spared. SCLE heals without scarring or atrophy. Post inflamShe is a nonsmoker without any recent travel history or sick matory hypopigmentation may result, but usually resolves contacts. Her father has a history of psoriasis limited to the overtime. scalp. Discoid lupus erythematous (DLE) is the most common Her vital signs were stable. Skin examination revealed type of CCLE. It presents as well-defined, round, erythemaerythematous, annular plaques with central clearing forming tous plaques with an adherent scale that involves the hair folpolycyclic patterns. There are variable amounts of scale-­ licles. Unlike SCLE, the lesions scar as they evolve into crust at the margins as shown in Fig.  2. There are similar atrophic plaques with central hypopigmentation and periphlesions symmetrically distributed over the neck and trunk; eral hyperpigmentation, making this an incorrect answer however, the face is spared. There are no mucosal lesions or choice. In addition, the head and neck are common sites of joint swellings present. The remainder of the physical exam involvement. Lesions of the scalp can result in permanent was unremarkable. alopecia. Laboratory results at the time of evaluation revealed a Psoriasis is a chronic inflammatory disorder of the skin positive anti-neutrophilic antibody (ANA), but normal blood characterized by well-defined erythematous plaques with cell counts and urinalysis. A skin biopsy was performed and overlying silvery scales. It commonly affects the scalp, demonstrated lymphocytic infiltrate along the dermal-­ elbows, knees, and intergluteal cleft but any site may be epidermal junction with vacuolar basal cell degeneration. affected. The nails and joints may also be involved. DIF was performed and revealed granular deposits of IgG Psoriasis is not usually exacerbated by UV-radiation, but and C3 along the dermal-epidermal junction. instead may be beneficial. In fact, phototherapy may be used as a ­treatment modality in severe disease. Laboratory Question 1 evaluation in psoriatic patients would demonstrate a negaWhich of the following is the most likely diagnosis: tive ANA. The histological findings of psoriasis are characterized by parakeratosis, a decreased granular layer, and a A. Discoid lupus erythematous neutrophilic infiltrate in the dermis. DIF would also be B. Subacute cutaneous lupus erythematosus negative.

190

J. Jeskey et al.

Lichen planus is an idiopathic inflammatory disorder of the skin. The lesions manifest as pruritic, flat-topped papules and plaques with an overlying reticulated scale known as Wickham striae. The wrist and ankles are common sites of involvement, but any location may be affected. The rash is typically not photosensitive. Characteristic histological features of LP are described in Table  1. Both ANA and DIF would be negative. Polymorphic light eruption (PMLE) is an idiopathic inflammatory disorder of the skin. The lesions can take many morphologies including erythematous macules, papules, plaques, or vesicles that occur after sun exposure. PMLE is a diagnosis of exclusion that may resemble the cutaneous features of lupus erythematous; however, DIF would be negative in PMLE, making this answer choice incorrect. Question 2 Which of the following is most likely to be positive in this patient: A. B. C. D. E.

Anti-SSA/Ro antibody Anti-dsDNA antibody Anti-histone antibody Anti-Jo-1 antibody Anticentromere antibody

Answer: A Detection of serum autoantibodies can aid in the diagnosis of autoimmune diseases. SCLE is strongly associated with Anti-SSA/Ro antibodies. Studies have shown that 70% of patients are positive for anti-SSA/Ro and 70–80% are positive for ANA. In addition, half of the patients with SCLE will be positive for Anti-SSB/La. In contrast to systemic lupus erythematous, only 5% of patients are positive for anti-dsDNA. Anti-histone antibodies are associated with drug-induced lupus which presents with acute onset fever, arthralgias, and serositis after starting the causative drug. Common culprits are hydralazine, procainamide, isoniazid, and TNF-alpha inhibitors. Unlike SLE, Drug-induced lupus is less likely to have a cutaneous, central nervous system, or renal involvement. Anti-Jo-1 antibodies are specific for inflammatory myositis, which presents as progressive proximal muscle weakness and atrophy. Dermatomyositis can also present with similar skin manifestations as SLE. Anticentromere antibodies are associated with disease limited systemic sclerosis or CREST syndrome, which usually presents with fibrosis of the skin limited to fingers and face, calcinosis cutis, Raynaud phenomenon, esophageal dysmotility, sclerodactyly, and telangiectasias. The patient was started on an oral prednisone taper supplemented with 0.1% tacrolimus cream. She was advised to apply broad spectrum sunscreen (SPF of at least 50)

20–30 min before sun exposure. The rash resolved without scarring and minimal dyspigmentation. One year later the rash returned despite extensive photoprotection and continued topical maintenance therapy. In addition to the recurrence of the annular erythematous plaques, the patient also now complains of fatigue, joint pains, and oral ulcers. The laboratory evaluation revealed an elevated ANA, hemoglobin of 13, and platelet count of 120,000. Urinalysis was normal. Question 3 What percentage of patients will progress to systemic lupus erythematous? A. B. C. D. E.

5% 10% 25% 50% 90%

Answer: D Cutaneous lupus erythematosus is a common initial manifestation of LE and can occur with or without systemic disease. Fifty percent of patients with SCLE will eventually meet the criteria for systemic disease outlined by the American College of Rheumatology. However, systemic symptoms are mild and most commonly include a photosensitive rash, oral ulcers, arthritis, and positive serology. Most patients do not have central nervous involvement or lupus nephritis. The progression of chronic cutaneous lupus erythematosus to systemic disease is even rarer. In contrast, acute cutaneous lupus erythematosus is almost always associated with systemic disease. Question 4 Which of the following is the best next step in treatment for this patient? A. B. C. D. E.

Hydroxychloroquine Methotrexate Mycophenolate mofetil Rituximab Acitretin

Answer: A The first-line treatment for cutaneous lupus erythematous includes photoprotection, topical or oral steroids (depending on the extend of the disease), and hydroxychloroquine. For localized disease, topical steroids or topical calcineurin inhibitors can be used. For disease that is widespread, scarring or refractory to topicals, hydroxychloroquine is considered the drug of choice. Methotrexate, mycophenolate mofetil, rituximab, and acitretin may be considered in cases refractory to hydroxychloroquine.

Autoimmunity

Question 5 Which of the following should be monitored while taking this medication? A. Liver function tests B. Visual acuity C. Blood cell counts D. Thyroid function tests E. Lipid panel Answer: B Hydroxychloroquine is the drug of choice for systemic lupus erythematosus as well as cutaneous lupus erythematous that is widespread, severe or refractory to other first-line treatments. Although hydroxychloroquine is normally well tolerated, there is an increased risk of retinopathy after 5–7 years of use. The American Academy of Ophthalmology recommends a baseline examination for patients starting hydroxychloroquine with annual eye examinations after 5 years.

Bibliography 1. Kumar V, Abbas AK, Aster JC.  Robbins and Cotran pathologic basis of disease. 9th ed. Philadelphia, PA: Elsevier/Saunders; 2015. p. 211–7. 2. Miyamoto D, Santi CG, Aoki V, Maruta CW. Bullous pemphigoid. An Bras Dermatol. 2019;94(2):133–46. 3. Pratasava V, Sahni VN, Suresh A, Huang S, Are A, Hsu S, Motaparthi K. Bullous pemphigoid and other pemphigoid dermatoses. Medicina. 2021;57(10):1061. 4. Schmidt E, Della Torre R, Borradori L. Clinical features and practical diagnosis of bullous pemphigoid. Immunol Allergy Clin N Am. 2012;32(2):217–32. 5. Memis I, Andreadis D, Apessos I, Georgakopoulou E, Poulopoulos A.  Paraneoplastic autoimmune multi-organ syndrome and oral mucosa involvement: an intriguing disorder. Cancer Res Front. 2015;1(3):268–79. 6. Li N, Culton D, Diaz LA, Liu Z.  Modes of action of intravenous immunoglobulin in bullous pemphigoid. J Invest Dermatol. 2018;138(6):1249–51. 7. Ahmed AR. Intravenous immunoglobulin therapy for patients with bullous pemphigoid unresponsive to conventional immunosuppressive treatment. J Am Acad Dermatol. 2001;45(6):825–35.

191 8. Santoro FA, Stoopler ET, Werth VP. Pemphigus. Dent Clin N Am. 2013;57(4):597–610. 9. Trayes KP, Love G, Studdiford JS.  Erythema multiforme: recognition and management. Am Family Physician. 2019;100(2): 82–8. 10. Xu HH, Werth VP, Parisi E, Sollecito TP. Mucous membrane pemphigoid. Dent Clin N Am. 2013;57(4):611–30. 11. Lehman JS, Tollefson MM, Gibson LE.  Lichen planus. Int J Dermatol. 2009;48(7):682–94. 12. Joly P, Roujeau JC, Benichou J, et  al. A comparison of oral and topical corticosteroids in patients with bullous pemphigoid. N Engl J Med. 2002;346(5):321–7. 13. Williams HC, Wojnarowska F, Kirtschig G, et al. Doxycycline versus prednisolone as an initial treatment strategy for bullous pemphigoid: a pragmatic, non-inferiority, randomised controlled trial. Lancet. 2017;389(10079):1630–8. 14. Czernik A, Toosi S, Bystryn JC, Grando SA.  Intravenous immunoglobulin in the treatment of autoimmune bullous dermatoses: an update. Autoimmunity. 2012;45(1):111–8. 15. Sami N, Ali S, Bhol KC, Ahmed AR.  Influence of intravenous immunoglobulin therapy on autoantibody titres to BP AG1 and BP AG2  in patients with bullous pemphigoid. J Eur Acad Dermatol Venereol. 2003;17(6):641–5. 16. Kremer N, Snast I, Cohen ES, et  al. Rituximab and omalizumab for the treatment of bullous pemphigoid: a systematic review of the literature. Am J Clin Dermatol. 2019;20(2):209–16. 17. Abdat R, Waldman RA, De Bedout V, et al. Dupilumab as a novel therapy for bullous pemphigoid: a multicenter case series. J Am Acad Dermatol. 2020;83(1):46–52. 18. Galeotti C, Kaveri SV, Bayry J.  IVIG-mediated effector functions in autoimmune and inflammatory diseases. Int Immunol. 2017;29(11):491–8. 19. Grönhagen CM, Nyberg F.  Cutaneous lupus erythematosus: an update. Indian Dermatol Online J. 2017;5(1):7–13. 20. Gruber-Wackernagel A, Byrne SN, Wolf P.  Pathogenic mechanisms of polymorphic light eruption. Front Biosci (Elite Ed). 2009;1:341–54. 21. Trayes KP, Savage K, Studdiford JS. Annular lesions: diagnosis and treatment. Am Fam Physician. 2018;98(5):283–91. 22. Okon LG, Werth VP.  Cutaneous lupus erythematosus: diagnosis and treatment. Best Pract Res Clin Rheumatol. 2013;27(3):391–404. 23. Marmor MF, Kellner U, Lai TY, et al. Revised recommendations on screening for chloroquine and hydroxychloroquine retinopathy. Ophthalmology. 2011;118(2):415–22. 24. Wieczorek IT, Propert KJ, Okawa J, Werth VP. Systemic symptoms in the progression of cutaneous to systemic lupus erythematosus. JAMA Dermatol. 2014;150(3):291–6.

Vasculitis Nikita Patel and Stratos Christianakis

Abbreviations

1 Introduction

AAV ANCA-associated vasculitis ANCA Antineutrophil cytoplasmic antibodies CDC Centers for disease control CT Computed tomography DIF Direct immunofluorescence EM Electron microscopy EPGA Eosinophilic granulomatosis with polyangiitis FDG Fluoro-deoxyglucose GN Glomerulonephritis GPA Granulomatosis with polyangiitis HBV Hepatitis B virus HCV Hepatitis C virus HIV Human immunodeficiency virus IBD Inflammatory bowel disease IV Intravenous MPA Microscopic polyangiitis MPO myeloperoxidase MRI Magnetic resonance imaging PAN Polyarteritis nodosa PCR polymerase chain reaction PET Positron emission tomography PR 3 Proteinase 3 PUK Peripheral ulcerative keratitis RA Rheumatoid arthritis SAGN Streptococcal-associated glomerulonephritis TB Tuberculosis

Vasculitis is a broad topic and the clinical presentation can vary depending on the type of vessel and organ system affected. Large vessel vasculitis includes Giant Cell Arteritis and Takayasu Arteritis. Medium vessel vasculitis includes Polyarteritis nodosa and Kawasaki Disease. Small vessel vasculitis can be categorized as immune complex-mediated or pauci-immune such as ANCA-associated vasculitis. Immune complex small vessel vasculitis includes cryoglobulinemic vasculitis, IgA vasculitis, and hypocomplementemic urticarial vasculitis. ANCA-associated small vessel vasculitis includes MPA, GPA, and EPGA. For this chapter, due to the diversity of types of vasculitis, we will focus on ANCA-associated vasculitis and large vessel vasculitis. The patient cases and questions will highlight the complexity of organ involvement and how to recognize and diagnose vasculitis through clinical scenarios. The focus will be on laboratory, imaging, and pathology findings that would be useful in aiding the diagnosis. We will not focus on treatment, as this topic is more detailed and not pertinent to this chapter. The goal of this chapter is to make the non-­ Rheumatologist clinician aware of when to be concerned about vasculitis.

N. Patel (*) · S. Christianakis LAC-USC Division of Rheumatology, Los Angeles, CA, USA

Case 1 A 57-year-old man with a history of prior treatment for pulmonary tuberculosis (TB) presents with worsening right eye pain and acute on chronic renal failure. Patient’s social history includes tobacco use with smoking 5 cigarettes per day for 25 years. He denied any current alcohol use. He works as a mechanic. He lives with his wife and son. He is originally from Mexico, immigrated to the United States approximately 10 years ago. He was treated for pulmonary TB approximately 1 year prior to presentation with rifampin, isoniazid, ethambutol, pyrazinamide, and pyridoxin. At that time, he had symptoms of congestion, cough, shortness of breath, and fevers. He also noted some eye irritation however was given eye

© The Author(s), under exclusive license to Springer Nature Switzerland AG 2022 M. Mahmoudi (ed.), Absolute Allergy and Immunology Board Review, https://doi.org/10.1007/978-3-031-12867-7_20

193

N. Patel and S. Christianakis

194

drops for possible allergies or dryness. A preliminary chest radiograph showed left lung pneumonia with concerning features, so it was followed up with chest computed tomography (CT) (Fig. 1). Question 1 Which ANCA-associated vasculitis is most likely to have this type of pulmonary involvement? A. Eosinophilic Granulomatosis with Polyangiitis (EGPA) B. Microscopic Polyantiitis (MPA)

C. Granulomatosis with Polyangiitis (GPA) D. Polyarteritis Nodosa (PAN) Answer: C Discussion: There are many different causes of cavitary lung lesions that must be considered in the differential (see Table 1 and Fig. 2). Among autoimmune causes of cavitary lung lesions, ANCA-associated vasculitis is high on the differential. The main types of lung lesions in AAV include necrotizing granulomatous or cavitary lesions (lung nodules), tracheobronchial inflammation, pulmonary capillaritis manifesting as diffuse alveolar hemorrhage (DAH), interstitial lung disease (ILD), and asthma. ANCA-associated vasculitis variants include MPA, GPA, EGPA, and single organ limited AAV. GPA is characterized by necrotizing granulomatous inflammation. GPA can involve both the upper and lower respiratory tracts. Table 1  Causes of cavitary lung lesions Cancer Autoimmune Vascular Infection

Fig. 1  CT of the chest without contrast showing left lung with large consolidative mass in the upper lobe with cavitation in the anterior, inferior portion, and surrounding smaller nodules in bilateral lungs

Trauma Youth

Bronchogenic carcinoma Metastasis Granulomatosis with polyangiitis Rheumatoid arthritis Pulmonary emboli Tuberculosis Pulmonary abscess Traumatic pulmonary pseudocyst Congenital pulmonary airway malformation Pulmonary sequestration Bronchogenic cyst

C

Cancer

Bronchogenic carcinoma (most common SCC) Metastasis (most common SCC)

A

Autoimmune

Granulomatosis with polyangiitis Rheumatoid arthritis

V

Vascular

Pulmonary emboli

I

Infection

Tuberculosis pulmonary abscess

I

Trauma

Traumatic pulmonary pseudocyst

Y

Youth

Congenital pulmonary airway malformation pulmonary sequestration Bronchogenic cyst

SCC, squmous cell carcinoma. Fig. 2  Differential diagnosis for cavitary lung lesions with pneumonic “CAVITY”

195

Vasculitis

Upper tract involvement tends to be more symptomatic with hoarseness, dyspnea, stridor, or wheezing. Lower respiratory tract involvement in GPA can present as any of the lung lesions listed above; however, lung nodules can be found in up to 50% of the patients with GPA. Cavities are usually thick-­ walled, with irregular inner margins and absence of calcification. They are often in relation to vessels but do not necessarily have a predilection for specific lung zones. Depending on the degree of lung involvement patients may experience cough or development of hemoptysis. Chest X-ray must be followed up by CT of the chest if abnormalities are noted. Interstitial lung disease (ILD) in AAV mostly occurs in the context of MPA and in patients with anti-­myeloperoxydase (MPO)-ANCA. The most frequent high-resolution computed tomography (HRCT) findings of ILD associated with MPA include ground-glass and reticular opacities, interlobular septal thickening, parenchymal consolidations, and honeycombing. Question 2 If this gentleman were to have a history of adult-onset asthma, which of the following vasculitides should be suspected? A. Eosinophilic Granulomatosis with Polyangiitis (EGPA) B. Microscopic Polyangiitis (MPA) C. Granulomatosis with Polyangiitis (GPA) D. Polyarteritis Nodosa (PAN) Answer: A Asthma is the main clinical manifestation of EGPA and occurs in 95–100% of patients. Asthma in EGPA is also associated with chronic rhinosinusitis, atopic background, blood eosinophilia, and the presence of ANCA (though can be absent in up to 60% of patients) most often directed against MPO antigen. Radiological abnormalities like peripheral ground-glass opacities, consolidation, bronchial thickening, or pleural effusions can be found in the most severe cases. Transbronchial or surgical biopsies may be considered to rule out differential diagnoses (mainly tumors or infections, especially fungal caused by histoplasmosis, coccidioidomycosis, blastomycosis, or nocardia) or to confirm the diagnosis of vasculitis in cases of isolated lung nodules without other organ involvement. Further workup along with infectious considerations should include inflammatory markers (ESR, CRP), ANCA with MPO and PR3 titers, as well as ruling out other possible inflammatory causes such as RA and Sarcoidosis in the ­correct clinical scenario. Lung nodules usually respond to immunosuppressive therapies.

Case Continued The patient continued to complain of eye irritation and 2  months after completing TB treatment presented to the Emergency Department (ED) with bilateral eye redness, pain, and blurring of the vision. He had two subsequent visits to the ED with cough, rhinorrhea, and chest discomfort. Each time he was treated with supportive care for presumed viral etiologies with CXR showing the previously seen findings on the left lung field. Antineutrophil cytoplasmic antibodies were sent which returned with results below (Table 2 ANCA testing). Question 3 Which of the following is the most likely diagnosis for this patient? A. B. C. D.

Episcleritis Anterior scleritis Scleromalacia perforans Nodular scleritis

Answer: B Discussion: The sclera is the outer type I collage of the eyes with scleritis representing inflammation of the sclera. The differential diagnosis of scleritis includes infection, malignancy, inflammatory arthritides, systemic autoimmune disease, and primary systemic vasculitides. Clinically scleritis can present with erythema, decreased visual acuity, pain, and photophobia. Ocular pain typically helps differentiate episcleritis from scleritis. Scleritis can be characterized based on anatomical distribution, as anterior or posterior to recti muscle insertion site. The subtypes of anterior scleritis include diffuse, nodular, and necrotizing. Scleromalacia perforans is a type of necrotizing scleritis in which patients do not experience pain. An Ophthalmologic emergency can lead to blindness if not quickly treated. Necrotizing scleritis has a poor prognosis and the highest association with systemic disease. After ruling out infectious, malignant, and drug-­induced causes of scleritis, autoimmune etiologies must be further investigated. The most common autoimmune associations in order of prevalence include rheumatoid arthritis, GPA, relapsing polychondritis, inflammatory bowel disease, and systemic lupus erythematosus. Ocular involvement in GPA occurs in approximately 38% of patients with scleritis being the common manifestation. Table 2  ANCA testing Variable ANCA screen C-ANCA titer P-ANCA titer Proteinase 3 antibody (antibody index) Myeloperoxidase antibody (antibody index)

Patient value C-ANCA positive 1:160 Not detected 42.1